summaryrefslogtreecommitdiff
path: root/info/digests/texhax/96
diff options
context:
space:
mode:
authorNorbert Preining <norbert@preining.info>2019-09-02 13:46:59 +0900
committerNorbert Preining <norbert@preining.info>2019-09-02 13:46:59 +0900
commite0c6872cf40896c7be36b11dcc744620f10adf1d (patch)
tree60335e10d2f4354b0674ec22d7b53f0f8abee672 /info/digests/texhax/96
Initial commit
Diffstat (limited to 'info/digests/texhax/96')
-rw-r--r--info/digests/texhax/96/texhax.01431
-rw-r--r--info/digests/texhax/96/texhax.02359
-rw-r--r--info/digests/texhax/96/texhax.03594
-rw-r--r--info/digests/texhax/96/texhax.04257
-rw-r--r--info/digests/texhax/96/texhax.05224
-rw-r--r--info/digests/texhax/96/texhax.06432
-rw-r--r--info/digests/texhax/96/texhax.07433
-rw-r--r--info/digests/texhax/96/texhax.08372
-rw-r--r--info/digests/texhax/96/texhax.09493
-rw-r--r--info/digests/texhax/96/texhax.10366
-rw-r--r--info/digests/texhax/96/texhax.11416
-rw-r--r--info/digests/texhax/96/texhax.12352
-rw-r--r--info/digests/texhax/96/texhax.13429
-rw-r--r--info/digests/texhax/96/texhax.14344
-rw-r--r--info/digests/texhax/96/texhax.15233
15 files changed, 5735 insertions, 0 deletions
diff --git a/info/digests/texhax/96/texhax.01 b/info/digests/texhax/96/texhax.01
new file mode 100644
index 0000000000..5705be5c75
--- /dev/null
+++ b/info/digests/texhax/96/texhax.01
@@ -0,0 +1,431 @@
+From texhax-digest-outgoing-request@nottingham.ac.uk Mon Jan 8 21:25:30 1996
+Received: from jess.ccc.nottingham.ac.uk (jess.ccc.nottingham.ac.uk [128.243.40.193]) by granby.ccc.nottingham.ac.uk (8.6.12/8.6.12) with ESMTP id VAA14383 for <cczdao@unix.ccc.nottingham.ac.uk>; Mon, 8 Jan 1996 21:25:28 GMT
+Message-Id: <199601082125.VAA14383@granby.ccc.nottingham.ac.uk>
+Received: from nottingham.ac.uk by jess.ccc.nottingham.ac.uk
+ id <11536-0@jess.ccc.nottingham.ac.uk>;
+ Mon, 8 Jan 1996 20:43:04 +0000
+From: Majordomo list server <owner-texhax-digest@nottingham.ac.uk>
+To: texhax-digest@nottingham.ac.uk
+Subject: TeXhax Digest V96 #1
+Reply-To: TeXhax@tex.ac.uk
+Errors-To: owner-texhax-digest@nottingham.ac.uk
+Precedence: bulk
+Date: Mon, 8 Jan 1996 20:43:04 +0000
+Sender: owner-texhax-digest@nottingham.ac.uk
+
+
+TeXhax Digest Monday, 8 January 1996 Volume 96 : Number 001
+
+(incorporating UKTeX Digest)
+
+Today's Topics:
+
+ Re: TeXhax Digest V95 #18
+ Re: TeXhax Digest V95 #18
+ some questions from a newcomer
+ more questions!
+ LaTeX2e
+ modes.mf 3.0 available
+
+----------------------------------------------------------------------
+
+From: Sebastian Rahtz <s.rahtz@elsevier.co.uk>
+Date: Mon, 18 Dec 1995 09:16:38 GMT
+Subject: Re: TeXhax Digest V95 #18
+
+ > From: Andre HECK <heck@cdsxb6.u-strasbg.fr>
+ > Date: Tue, 05 Dec 95 17:25:31 +0100
+ > Subject: Chemical TeX/LaTeX
+ >
+ > My daughter heard that specific chemistry-oriented TeX/LaTeX packages
+ > were available, without more details unfortunately. My search in the
+ > ctan archives were unsuccessful.
+ > Anyone knows more about this matter? If so, is there any shareware
+ > available?
+there are several; look for chemtex, and xymtex; the latter, from
+Japan, is described in an article in the next issue of
+TUGboat. Another package, described by J Hagen at this years EuroTeX,
+is published in the EuroTeX 95 proceedings, an article which will be
+reprinted in TUGboat early next year
+
+Sebastian Rahtz
+Secretary, TUG
+
+------------------------------
+
+From: lamport@pa.dec.com
+Date: Sun, 17 Dec 95 13:58:13 -0800
+Subject: Re: TeXhax Digest V95 #18
+
+J. Greg Davidson, who hates being misquoted, writes
+
+ I find that most people I show TeX to dislike it on sight and decline
+ my offer to help them learn to use it. At the same time, they're
+ interested in SGML and planning to convert existing documents to and
+ write new documents in that form. This is the source of our big
+ opportunity.
+
+ My biggest gripe about TeX is that I find writing and understanding
+ TeX macros to be difficult...
+
+ As people move towards keeping documents in SGML form, they will be
+ looking for a good text formatter to use in conjunction with their
+ SGML documents.
+
+Many people think that SGML offers a solution to the problems of
+document preparation. They're wrong. SGML is just a statement of
+the problem. The goals of SGML--a markup language that separates
+logical structure from formatting directions--are precisely the goals
+of LaTeX. However, LaTeX is a program to produce real output--not just
+a syntax for wishing what the document should look like. It is this
+difference that is the primary reason why LaTeX input is a lot uglier
+than SGML; the difference between TeX syntax and SGML syntax has
+little to do with it.
+
+SGML syntax has some nice features. However, if you look at the
+HTML3.0 syntax for describing equations, you will discover that no-one
+in his right mind would want to type any significant amount of
+mathematics in SGML. Should HTML ever become expressive enough for
+mathematical documents (at the moment, the only math symbols HTML3.0
+provides are \sqrt, \lnot, and \pm), one would have to create those
+documents either with a WYSIWYG editor or by translating from a more
+TeX-like syntax.
+
+We've learned a lot in the last dozen years, and we know how to do
+some things better than Don did in TeX. But it's by no means clear
+that we know how to create a system with the power of TeX that would
+make adding new features very much easier. Think of all the work
+that's been done to give unsophisticated users the power of a
+programming language. After 30 years, there still doesn't seem to be
+anything better than Basic.
+
+Leslie Lamport
+
+
+------------------------------
+
+From: gabriele@dtovf1.roma2.infn.it (gabriele migliorini)
+Date: Wed, 27 Dec 95 15:27:26 +0100
+Subject: some questions from a newcomer
+
+ Dear readers,
+ i just began working with LaTeX, and I am waiting for
+a friend of mine to take me back from England a copy of the Companion
+(here in Italy, English and american books cost twice their price, for
+you get a very unfair exchange rate from bookshops) I'd like to get
+some little help before getting the book, even if I fear my questions
+could seem too much stupid for the expert ones. Two kind of problems:
+
+Text: how do I do to write a paragraph (or some paragraphs) and keep
+all of them on the same page? I remember that (a long time ago) in
+word perfect there was a function to put in a couple of markers
+telling to keep all the text in beetween on the same page. By now, I
+tried to resolve my problem by putting the paragraph in a minipage,
+but this cannot function with in paragraph stuff.
+
+Graphics: I poked around to understand how to include graphics in my
+documents, and I found many suggestions. Now, I've seen that (to me)
+the best format are those whose commands ger embedded right into the
+dvi file. I've seen that pstricks and eepic do thins kind of job, but
+both are written for LaTeX209; I cannot let pst function, but eepic
+seems great, with both XFIG and gnuplot. Now, my question: is there
+anybody who knows if there are incompatibilities w/epic-eepic other
+than the ones I didn't see? And what about PSTricks?
+
+Now, I have to thank everybody, and tell an happy new year to all the
+readers, those that will help me, and those who will be bored by my English
+an my questions. I apologize!.
+
+Sincerely,
+ gabriele
+
+gabriele@dtovf1.roma2.infn.it
+
+
+------------------------------
+
+From: gabriele@dtovf1.roma2.infn.it (gabriele migliorini)
+Date: Wed, 27 Dec 95 16:17:26 +0100
+Subject: more questions!
+
+I remembered just now that I need one more information about LaTeX:
+I'm writing about rotary motions, and I enocounter many rotations
+vectors; the standard practice is to use the normal greek letters
+you'd use for an angle, and tell the vector character typing the name
+in bold face. I can do that with standard {\bf a}, but the command
+{\bf \omega} doesn't work. How can I fix this behavior? Now, I'm to
+ask if there is someone willing to tell me the reason why the \left
+and \right verbs before opening and closing brackets and braces cannot
+be implicitly told by LaTeX: if i'm writing a fraction like (\frac
+{abc}{def}), I get small brackets; don't you think the standard
+behavior should be to prepone an mplicit \left or \right to any
+opening and closing bracket? (I see that |a|b|c| could break this
+semantics, but I think you could make the bars use the explicit
+notations and the other delimitations the implicit one) I thank you,
+sincerely.
+
+ gabriele
+
+
+
+------------------------------
+
+From: Andrew Jones $STAFF <JONEAN@trentcollege.nott.sch.uk>
+Date: Wed, 20 Dec 1995 16:29:11 +0000
+Subject: LaTeX2e
+
+To whom it may concern, I have a few problems which need solving, and
+do not know where to go for advice:
+
+I currently possess just LaTeX, but wiped it off my system, when
+trying to make room for something else. Stupidly, I did not back it
+up and recall that I had some problems trying to install it last
+time, having to change various lines in various files, both in emtex,
+and in my Config.sys and autoexec.bat files. As you may gather, I am
+pretty well a beginner with TeX. Does anyone know of anyone who would
+be able guide me through the reinstallation?
+
+Secondly, when I originally installed it, I had it set up to print on
+a Star LC10 9-pin dot matrix. I now possess a Canon BJ-10ex. I
+managed to get various printer drivers, none written speciically for
+this model, but none of them seem to work. Any suggestions?
+
+Finally, I was given your address from a visiting lecturer who uses
+LaTeX enormously. It rekindled my interest in Latex, and I would
+dearly love to get it properly re-installed once again. However, I
+notice that LaTeX2e is mentioned a great deal. What are the
+advantages, if any, of switching to this? I am currently using a
+386sx-20 with a 70 MB HDD, but am hoping to upgrade to a 486DX2-66
+with a 400MB HDD and CD-ROM in the near future.
+
+I would gratefully appreciate any information that you can give me.
+
+Yours sincerely Andrew Jones at Trent College
+the near future
+
+
+------------------------------
+
+From: "K. Berry" <kb@cs.umb.edu>
+Date: Mon, 8 Jan 1996 14:55:22 -0500
+Subject: modes.mf 3.0 available
+
+I have released version 3.0 of modes.mf. You can get it by anonymous ftp from
+
+ ftp.cs.umb.edu:pub/tex/modes.mf
+
+and shortly from ftp.cdrom.com:pub/tex/modes.mf and the ctan sites in
+/tex-archive/fonts/modes/modes-3.0.mf. finger ctan@ftp.shsu.edu for a
+list of all the CTAN sites and mirrors. The mailing list
+tex-archive@math.utah.edu receives most TeX-related announcements; email
+tex-archive-request@math.utah.edu to join.
+
+You can also get it by email from if you cannot ftp: email
+fileserv@shsu.edu with a body of `sendme modes'.
+
+News:
+- - most importantly, a new and much cleaner way of doing write/white
+ changes, compatible with the DC fonts as well as CM, etc. This *does*
+ affect the rasterization to some extent, but in the cases I tried, it
+ was no worse, and sometimes better.
+- - a small test file, modetest.tex, included.
+- - new modes for the Amiga ShowDVI previewer, PC previewing,
+ high-resolution fax, LJ 5, Canon BJC, Lexmark Optra 4049 R, Agfa 3400PS.
+- - default localfont now ljfour.
+- - screen_rows, screen_cols interchanged to match plain.mf.
+- - TeX formatting improved, but still not completely working.
+
+As always, thanks to the contributors, and further additions and
+improvements are welcome. Please send bug reports or suggestions to
+tex-fonts@math.utah.edu (email tex-fonts-request to join the mailing list).
+
+
+General information:
+
+modes.mf is a collection of Metafont mode_def's. It also makes common
+definitions for write/white printers, `special' information, and
+landscape mode. It uses up too much memory for the table sizes in the
+original mf.web, so you either have to increase the sizes (as in Web2c)
+or rename the file and remove unneeded modes. I can't decipher mf.web
+well enough to understand how to make the modes use less memory; if some
+Metafont hacker can tell me, I'd very much like to hear it.
+
+If you have mode_def's which are not listed below, or corrections to the
+existing ones, please send them to me. Improvements to the exposition,
+particularly in how to create a new mode_def, are also welcome.
+
+kb@cs.umb.edu
+
+mode_def agfafzz = % AGFA 400PS
+mode_def agfatfzz = % AGFA P3400PS
+mode_def amiga = % Commodore Amiga
+mode_def aps = % Autologic APS-Micro5
+mode_def apssixhi = % Autologic APS-Micro6
+mode_def atariezf = % Atari ST SLM 804 printer
+mode_def atarinf = % Atari 95dpi previewer
+mode_def atarins = % Atari 96x96 previewer
+mode_def atariotf = % Atari ST SM 124 screen
+mode_def bitgraph = % BBN Bitgraph at 118dpi
+mode_def bjtenex = % Canon BubbleJet 10ex
+mode_def boise = % HP 2680A
+mode_def canonbjc = % Canon BJC-600, 360x360dpi
+mode_def canonex = % CanonEX in LaserWriter Pro 630
+mode_def canonlbp = % e.g., Symbolics LGP-10
+mode_def cg = % Compugraphic 8600
+mode_def cgl = % Compugraphic 8600 landscape
+mode_def cgnszz = % Compugraphic 9600
+mode_def crs = % Alphatype CRS
+mode_def cx = % Canon CX, SX, LBP-LX
+mode_def datadisc = % DataDisc
+mode_def newdd = % DataDisc with special aspect ratio
+mode_def declarge = % DEC 19-inch, 1280 x 1024
+mode_def decsmall = % DEC 17-inch, 1024 x 768
+mode_def deskjet = % HP DeskJet 500
+mode_def docutech = % Xerox 8790 or 4045
+mode_def dover = % Xerox Dover
+mode_def eighthre = % EightThree (83x83)
+mode_def epsdrft = % Epson at 120x72dpi
+mode_def epsdrftl = % Epson at 120x72dpi landscape
+mode_def epsfast = % Epson at 60x72dpi
+mode_def epsfastl = % Epson at 60x72dpi landscape
+mode_def epson = % 9-pin Epson MX/FX family
+mode_def epsonl = % 9-pin Epson MX/FX family landscape
+mode_def epsonact = % Epson Action Laser 1500
+mode_def epsonlo = % Epson at 120x216dpi
+mode_def epsonlol = % Epson at 120x216dpi landscape
+mode_def epstylus = % Epson Stylus
+mode_def fourfour = % FourFour (44x44) (really low-res)
+mode_def gtfax = % 204 x 196dpi G3fax
+mode_def gtfaxl = % 204 x 196dpi G3fax landscape
+mode_def gtfaxlo = % 204 x 98dpi G3fax
+mode_def gtfaxlol = % 204 x 98dpi G3fax landscape
+mode_def highfax = % 200 x 200dpi G3fax
+mode_def hprugged = % HP RuggedWriter 480
+mode_def ibm_a = % IBM 38xx (\#1)
+mode_def ibmd = % IBM 38xx (\#2)
+mode_def ibmega = % IBM EGA monitor
+mode_def ibmegal = % IBM EGA monitor landscape
+mode_def ibmfzon = % IBM 4019
+mode_def ibmfztn = % IBM 4029-30, 4250
+mode_def ibmpp = % IBM ProPrinter
+mode_def ibmppl = % IBM ProPrinter landscape
+mode_def ibmsoff = % IBM 6154 display
+mode_def sherpa = % IBM 6670 (Sherpa)
+mode_def ibmteot = % IBM 3812
+mode_def ibmtetz = % IBM 3820
+mode_def ibmtont = % IBM 3193 screen
+mode_def ibmtosn = % IBM 3179 screen
+mode_def ibmtosnl = % IBM 3179 screen landscape
+mode_def ibmvga = % IBM VGA monitor
+mode_def ibx = % Chelgraph IBX
+mode_def itoh = % CItoh 8510A
+mode_def itohl = % CItoh 8510A landscape
+mode_def itohtoz = % CItoh 310
+mode_def itohtozl = % CItoh 310 landscape
+mode_def iw = % Apple ImageWriter
+mode_def jetiiisi = % HP Laser Jet IIISi
+mode_def lasf = % DEC LA75
+mode_def lexmarkr = % IBM (Lexmark) Optra R (4049)
+mode_def linolo = % Linotype Linotronic [13]00 at 635dpi
+mode_def linolttz = % Linotype Linotronic L-300 with RIP-50
+mode_def linoone = % Linotype Linotronic [13]00 at 1270dpi
+mode_def linotzzh = % Linotype Linotronic 300 at 2540dpi
+mode_def ljfive = % HP LaserJet 5
+mode_def ljfour = % 600dpi HP LaserJet 4
+mode_def ljlo = % HP LaserJet at 150dpi
+mode_def lmaster = % 1000dpi LaserMaster
+mode_def lnzo = % DEC LN01
+mode_def lpstz = % DEC lps20
+mode_def lqlores = % Epson LQ-500, 180x180dpi
+mode_def lqmed = % Epson LQ-500, 360x180dpi
+mode_def lqmedl = % Epson LQ-500, 360x180dpi landscape
+mode_def lview = % Sigma L-View monitor
+mode_def lwpro = % Apple LaserWriterPro 810
+mode_def macmag = % Mac screens at magstep 1
+mode_def mactrue = % Mac screens at 72dpi
+mode_def ncd = % NCD 19-inch
+mode_def nec = % NEC 180dpi
+mode_def nechi = % NEC-P6 at 360x360dpi
+mode_def neclm = % NEC PC-PR406LM 320dpi
+mode_def nectzo = % NEC PC-PR201 series 160dpi
+mode_def nexthi = % NeXT 400dpi, Newgen
+mode_def nextscrn = % 100dpi NeXT monitor
+mode_def nineone = % NineOne (91x91)
+mode_def nullmode = % TFM files only
+mode_def onetz = % OneTwoZero (120/120)
+mode_def onezz = % OneZeroZero (100x100)
+mode_def ocessfz = % OCE 6750-PS
+mode_def okidata = % Okidata
+mode_def okidatal = % Okidata landscape
+mode_def okifte = % Okidata 410e in 600 DPI mode
+mode_def pcscreen = % also, e.g., high-resolution Suns
+mode_def pcprevw = % preview on pc screen
+mode_def phaser = % Tektronix Phaser PXi
+mode_def prntware = % Printware 720IQ
+mode_def qms = % QMS (Xerox engine)
+mode_def qmsostf = % QMS 1725
+mode_def qmsoszz = % QMS 1700
+mode_def ricoh = % e.g., TI Omnilaser
+mode_def ricoha = % e.g., IBM 4216
+mode_def ricohlp = % e.g., DEC LN03
+mode_def sparcptr = % Sun SPARCprinter
+mode_def starnlt = % Star NL-10
+mode_def starnltl = % Star NL-10 landscape
+mode_def sun = % Sun and BBN Bitgraph at 85dpi
+mode_def supre = % Ultre*setter at 2400dpi
+mode_def toshiba = % Toshiba 13XX, EpsonLQ
+mode_def ultre = % Ultre*setter at 1200dpi
+mode_def vs = % VAXstation monitor
+mode_def vtftzz = % Varityper 4200 B-P
+mode_def vtftzzhi = % Varityper 4300P at 2400dpi
+mode_def vtftzzlo = % Varityper 4300P at 1200dpi
+mode_def vtfzszw = % Varitype 5060W, APS 6
+mode_def vtszz = % Varityper Laser 600
+mode_def xrxesnz = % Xerox 8790 or 4045
+mode_def xrxfzfz = % Xerox 4050/4075/4090/4700
+mode_def xrxnszz = % Xerox 9700
+mode_def xrxtszz = % Xerox 3700
+
+------------------------------
+
+End of TeXhax Digest V96 #1
+***************************
+
+
+About TeXhax...
+
+Please send contributions to: TeXhax@tex.ac.uk
+
+Subscription and unsubscription requests:
+ send a one line mail message to TeXhax-Request@tex.ac.uk
+ containing either subscribe texhax
+ or unsubscribe texhax
+If you have problems with un/subscribing, please mail owner-texhax@nott.ac.uk
+
+To obtain the Frequently Asked Questions (FAQ) lists for TeX, send a
+message with no subject to fileserv@shsu.edu, consisting of
+SENDME FAQ
+
+For information on the TeX Users Group, please send a message to
+TUG@TUG.org, or write TeX Users Group, 1850 Union Street, #1637
+San Francisco CA 94123 (phone: 1 415 982 8449, fax: 1 415 982 8559)
+
+Backnumbers of all the digests are stored in the Comprehensive TeX
+Archive Network (CTAN) and can be retrieved on the Internet by
+anonymous ftp. The hosts comprising CTAN include, among others,
+ ftp.dante.de (129.69.1.12) -- Germany
+ ftp.shsu.edu (192.92.115.10) -- USA
+ ftp.tex.ac.uk (128.232.1.87) -- UK
+Please use your nearest server, to keep network load down.
+The file /tex-archive/CTAN.sites on each of these hosts gives a
+list of other sites which maintain full or partial mirrors of the CTAN.
+Alternatively, finger ctan_us@ftp.shsu.edu for full details.
+
+TeXhax Digest back issues are filed below /tex-archive/digests/texhax/
+Keyword-In-Context indexes are filed in /tex-archive/digests/indexes/
+
+A Hypermail version of TeXhax is also available on the World-Wide Web at URL
+http://www.tex.ac.uk/tex-archive/digests/hyper/
+
+\bye
+
diff --git a/info/digests/texhax/96/texhax.02 b/info/digests/texhax/96/texhax.02
new file mode 100644
index 0000000000..806e3b9900
--- /dev/null
+++ b/info/digests/texhax/96/texhax.02
@@ -0,0 +1,359 @@
+From texhax-digest-outgoing-request@nottingham.ac.uk Wed Jan 17 16:13:03 1996
+Received: from jess.ccc.nottingham.ac.uk (jess.ccc.nottingham.ac.uk [128.243.40.193]) by granby.ccc.nottingham.ac.uk (8.6.12/8.6.12) with ESMTP id QAA10736 for <cczdao@unix.ccc.nottingham.ac.uk>; Wed, 17 Jan 1996 16:12:53 GMT
+Message-Id: <199601171612.QAA10736@granby.ccc.nottingham.ac.uk>
+Received: from nottingham.ac.uk by jess.ccc.nottingham.ac.uk
+ id <06297-0@jess.ccc.nottingham.ac.uk>;
+ Wed, 17 Jan 1996 15:54:48 +0000
+From: Majordomo list server <owner-texhax-digest@nottingham.ac.uk>
+To: texhax-digest@nottingham.ac.uk
+Subject: TeXhax Digest V96 #2
+Reply-To: TeXhax@tex.ac.uk
+Errors-To: owner-texhax-digest@nottingham.ac.uk
+Precedence: bulk
+Date: Wed, 17 Jan 1996 15:54:48 +0000
+Sender: owner-texhax-digest@nottingham.ac.uk
+
+
+TeXhax Digest Wednesday, 17 January 1996 Volume 96 : Number 002
+
+(incorporating UKTeX Digest)
+
+Today's Topics:
+
+ TeX/LaTeX on Windows
+ Re: LaTeX2e
+ latex2e IEEEtran macro wanted.
+ EmTeX printing to Canon BJ200ex printer
+ gtex installation help
+ TUGboat 16 #3 is being mailed
+ New problems from a newcomer...
+
+----------------------------------------------------------------------
+
+From: "Sam Nelson (CO)" <Sam.Nelson@cs.stir.ac.uk>
+Date: Tue, 9 Jan 1996 10:27:33 +0000 (GMT)
+Subject: TeX/LaTeX on Windows
+
+Not being a beginner, this is going to sound a bit weird, I suppose, but
+despite having used TeX and LaTeX around here for more than a decade, I've
+never been asked (very much) to put them up on PC platforms, which are in
+little use in this department. However, I've just been asked about running
+LaTeX on a Windows platform, and it occurs to me that I know nothing whatever
+about this.
+
+Starting from scratch then, in an early-1996 Windows environment, where should
+one go to get the ideal TeX/LaTeX setup? I assume there are `commercial'
+solutions (we use Textures on Macintosh already) but I don't know how much
+money might be available, so a price spectrum from zero to lottery-winner is
+appropriate, I think.
+
+Thanks for any suggestions,
+
+Sam.
+- --
+Sam Nelson, Comp Sci, Stirling U, FK9 4LA, Scotland ,->0->M
+Email: sam@cs.stir.ac.uk Pager: 0839 456640 I->3-+->2->R=->-+->4->O
+Tel: +44 1786 467443 Fax: +44 1786 464551 `->1->S=->-'
+URL: http://www.cs.stir.ac.uk/~sam R$+@$+ $@smtp$#$2$:$1@$2
+
+------------------------------
+
+From: David Carlisle <carlisle@ma.man.ac.uk>
+Date: Tue, 9 Jan 96 17:33:17 GMT
+Subject: Re: LaTeX2e
+
+> I currently possess just LaTeX,
+> ...
+> However, I notice that LaTeX2e is mentioned a great deal. What are
+> the advantages, if any, of switching to this?
+
+I think this question is the result of a misunderstanding. LaTeX2e is
+not an `alternative to LaTeX' but rather is just the current version
+of LaTeX. The previous version (2.09) is now obsolete.
+
+Specifically if you should decide to re-install a complete emtex on
+your PC (This is probably a good idea anyway, as emtex was
+significantly re-organised and improved earlier this year) then the
+LaTeX version that you will get by default will be LaTeX2e.
+
+emtex (including LaTeX, and suitable previewers etc) is available from
+ctan sites such as ftp.tex.ac.uk in tex-archive/systems/msdos/emtex.
+You can make a basic installation by following QUICK.ENG (quick
+English instructions) to be found in that directory.
+
+David
+
+
+------------------------------
+
+From: "Dr. Ke Chen" <chen@cis.pku.edu.cn>
+Date: Fri, 12 Jan 1996 17:11:18 +0800 (GMT+0800)
+Subject: latex2e IEEEtran macro wanted.
+
+I found the previous IEEEtran.sty did not work in LaTex2e. Some days ago,
+a person sent me his own new IEEEtran.sty but there are a few problems
+such as that the title of section is always typeset with a small font,
+etc. As a result, I would appreciate it if anyone could tell me where
+a complete IEEEtran macro in latex2e is available.
+
+Ke Chen
+
+#========================================================================#
+# Dr. Ke Chen, Associate Professor Tel: +86-10-2751935(O) #
+# National Lab of Machine Perception +86-10-2585703(H) #
+# The Center of Information Science Fax: +86-10-2563883,2552779 #
+# Peking University Email: chen@cis.pku.edu.cn #
+# Beijing 100871, China chenke@pku.edu.cn #
+#========================================================================#
+
+
+------------------------------
+
+From: maues@csv.warwick.ac.uk
+Date: Thu, 11 Jan 1996 20:51:00 +0000 (GMT)
+Subject: EmTeX printing to Canon BJ200ex printer
+
+I am absolutely a beginner as you can tell.
+I have successfully installed emTex onto my PC and now is the Printer.
+(use under DOS)
+
+I have a Canon BJ200ex printer.
+
+Q1. When the time I use the commands:
+1) latex2e sample2e OKAY (where sample2e is the file name)
+2) v @bj samples2e OKAY
+3) prtbj10e samples2e ===> 6 blank pages
+4) prtbj300 sample2e ==> 6 blank pages
+5) prtp6h samples2e ===> there are some pages print out, which is the
+ original BUT for each line of the orginal
+ document, it comes out twice and into four
+ separate parts.
+
+
+By the way, p6h is not the printer I am using, but every time I used the
+print command, a file for the p6h fonts created under the font directory.
+
+Do you know what is it about?
+And what should I do now?
+What command should I use?
+
+Thanks
+ __\/__
+ . / ^ _ \ .
+ |\| (o)(o) |/|
+ #---------------.OOOo--oo--oOOO.--------------#
+ # Trevor Jonathan Yuen #
+ # Mathematics Institute #
+ # University of Warwick #
+ # Phone: (0973) 345 457 #
+ #_________________________Oooo._______________#
+ .oooO ( )
+ ( ) ) /
+ \ ( (_/
+ \_)
+
+
+------------------------------
+
+From: Joao Luis Cardoso Soares <jls55@columbia.edu>
+Date: Mon, 15 Jan 1996 09:59:41 -0500 (EST)
+Subject: gtex installation help
+
+I've installed gtex 2.2 (by Young U. Ryu) on a Pentium machine running
+windows 95, but I keep getting two error messages which might be related:
+1) After running \tex\set-tex.bat I get
+ Sharing violation - c:\tex\set-tex.bat
+2) When trying to open a DVI file whose fonts are to be created, I get
+ Cannot find the file ".
+ Make sure that the file exists on your system and that the path
+ and filename are correct.
+
+Does anyone have a suggestion, or at least let me know the email address of
+the author. Many thanks, Joao
+- --
+Joao Luis Cardoso Soares E-mail: jsoares@groucho.gsb.columbia.edu
+Columbia University Tel : (212) 749-6222 (Home)
+804 Uris Hall Fax : (212) 864-4857 (PhD student)
+New York, NY 10027 http://www.cc.columbia.edu/~jls55/
+
+
+------------------------------
+
+From: bbeeton <BNB@MATH.AMS.ORG>
+Date: Tue, 16 Jan 1996 17:44:10 -0500 (EST)
+Subject: TUGboat 16 #3 is being mailed
+
+i'm pleased to announce that tugboat 16#3 will be mailed from the
+printer tomorrow. the table of contents for this issue is attached.
+
+tugboat 16#4 will be sent to the printer within the next few days.
+the schedule calls for mailing three weeks after the printer receives
+the copy.
+
+the next issue, 17#1, will be dated march 1996 and should be delivered
+to the printer within that month. thanks to all the members of the
+production team who have made very great efforts to get this back on
+schedule.
+ -- barbara beeton
+
+ --------------------
+
+ TUGboat
+
+ Volume 16, Number 3 / September 1995
+ ================================
+
+Contents (128 pages)
+
+
+Robin Fairbairns
+ Production notes 222
+
+Opening Address
+ Michel Goossens
+ President's words 223
+
+Fonts
+ Jiri Zlatuska 227
+ When MF does it alone
+ Richard J. Kinch
+ MetaFog: converting MF shapes to contours 233
+ Alan Hoenig
+ The Poetica family: fancy fonts with TeX and LaTeX 244
+ Michel Goossens
+ Using Adobe Type 1 Multiple Master fonts with TeX 253
+ Jeremy Gibbons
+ Dotted and dashed lines in MF 259
+ Sergey Lesenko
+ Printing TeX documents with partial Type 1 fonts 265
+
+LaTeX
+ Matthew Swift
+ Modularity in LaTeX 269
+ Dennis Kletzing
+ A multienumerate package 276
+
+Hyphenation
+ Petr Sojka and Pavel Sevecek
+ Hyphenation in TeX --- Quo Vadis? 280
+ Petr Sojka
+ Notes on compound word hyphenation in TeX 290
+
+Literate programming
+ Wlodek Bzyl
+ Literate Plain source is available! 297
+ Bart Childs, Deborah Dunn and William Lively
+ Teaching CS/1 courses in a literate manner 300
+
+Methods
+ T.V. Raman
+ An audio view of (La)TeX documents --- part II 310
+ Sebastian Rahtz
+ Another look at LaTeX to SGML conversion 315
+ Robin Fairbairns
+ Omega --- Why bother with Unicode? 325
+ Gabriel Valiente Feruglio
+ Modern Catalan typographical conventions 329
+
+News & Announcements
+ TUG'96 Announcement 339
+ Calendar 340
+
+TUG Business
+ TUG'95 --- List of Attendees 341
+ Institutional members 344
+
+Advertisements
+ TeX consulting and production services 345
+
+%% END OF FILE
+
+------------------------------
+
+From: gabriele@dtovf1.roma2.infn.it (gabriele migliorini)
+Date: Wed, 17 Jan 96 13:12:34 +0100
+Subject: New problems from a newcomer...
+
+Dear readers,
+ at the very beginning, I have to thank all of you for reading
+my previous post, an trying to give me sensible replies. To summarize,
+the best answer to the bold greek letters is to simply get and run
+amsbsy, that's part of the amslatex distribution. It gives two new
+commands: a \boldsymbol that embolden a char in a font having a
+correspondent bold-face, and a 'ppor men's bold', that overstrikes a
+normal font, emboldening it even if there is not a bold typeface for
+printing it. As I got asmlatex, I got many new useful functions, like
+an extendable arrow symbol for writing limits, and a couple of new
+strange (?bugs?): in the manual, I read that there are a couple of
+operator redefining commands, that should make a log-like operator,
+with eventual _{text} printed at bottom right of the operator, and a
+lim like one, with text printed under the operator. But, in my printed
+version, they look exactly the same, and therefore, something is not
+doing it job. Furthermore, when I latexed the testmath.tex document, I
+got some \i command undefined in math mode, about lines containing a
+\begin or a \end... Are those signs of an incompatible behavior of
+
+a) the latex release of december'95 and amslatex
+b) TeX 3.1415 and the other programs
+c) the test documents, and amslatex (sure enough, I'm joking here!)
+
+Now, I'm crying for help another time: You know, I was writing down
+some lecture notes for a course I attended In march, and I started
+using latex after a disaster with the copy of word on the computer I
+used: the equation editor suddenly decided not to run anymore. Now, I
+have half of my work in Latex, and another half in Word 6 format. Is
+there anybody who does know how to convert the latter? I read, in the
+documentation about programs on ctan, that such a program cannot be
+public domain... But does there exist a commercial program doing the
+job? If it does exist, does some of you own a copy of it, and can be
+so nice to let me send him my document, and convert it for me?
+
+I'll be, as usual, grateful 'till the end of my life to anybody
+helping me. Sincerely,
+
+l.
+
+
+------------------------------
+
+End of TeXhax Digest V96 #2
+***************************
+
+
+About TeXhax...
+
+Please send contributions to: TeXhax@tex.ac.uk
+
+Subscription and unsubscription requests:
+ send a one line mail message to TeXhax-Request@tex.ac.uk
+ containing either subscribe texhax
+ or unsubscribe texhax
+If you have problems with un/subscribing, please mail owner-texhax@nott.ac.uk
+
+To obtain the Frequently Asked Questions (FAQ) lists for TeX, send a
+message with no subject to fileserv@shsu.edu, consisting of
+SENDME FAQ
+
+For information on the TeX Users Group, please send a message to
+TUG@TUG.org, or write TeX Users Group, 1850 Union Street, #1637
+San Francisco CA 94123 (phone: 1 415 982 8449, fax: 1 415 982 8559)
+
+Backnumbers of all the digests are stored in the Comprehensive TeX
+Archive Network (CTAN) and can be retrieved on the Internet by
+anonymous ftp. The hosts comprising CTAN include, among others,
+ ftp.dante.de (129.69.1.12) -- Germany
+ ftp.shsu.edu (192.92.115.10) -- USA
+ ftp.tex.ac.uk (128.232.1.87) -- UK
+Please use your nearest server, to keep network load down.
+The file /tex-archive/CTAN.sites on each of these hosts gives a
+list of other sites which maintain full or partial mirrors of the CTAN.
+Alternatively, finger ctan_us@ftp.shsu.edu for full details.
+
+TeXhax Digest back issues are filed below /tex-archive/digests/texhax/
+Keyword-In-Context indexes are filed in /tex-archive/digests/indexes/
+
+A Hypermail version of TeXhax is also available on the World-Wide Web at URL
+http://www.tex.ac.uk/tex-archive/digests/hyper/
+
+\bye
+
diff --git a/info/digests/texhax/96/texhax.03 b/info/digests/texhax/96/texhax.03
new file mode 100644
index 0000000000..748c16e714
--- /dev/null
+++ b/info/digests/texhax/96/texhax.03
@@ -0,0 +1,594 @@
+From texhax-digest-outgoing-request@nottingham.ac.uk Thu Mar 7 17:53:49 1996
+Received: from jess.ccc.nottingham.ac.uk (jess.ccc.nottingham.ac.uk [128.243.40.193]) by granby.ccc.nottingham.ac.uk (8.6.12/8.6.12) with ESMTP id RAA09784 for <cczdao@unix.ccc.nottingham.ac.uk>; Thu, 7 Mar 1996 17:53:47 GMT
+Message-Id: <199603071753.RAA09784@granby.ccc.nottingham.ac.uk>
+Received: from nottingham.ac.uk by jess.ccc.nottingham.ac.uk
+ id <26648-0@jess.ccc.nottingham.ac.uk>;
+ Thu, 7 Mar 1996 17:29:03 +0000
+From: Majordomo list server <owner-texhax-digest@nottingham.ac.uk>
+To: texhax-digest@nottingham.ac.uk
+Subject: TeXhax Digest V96 #3
+Reply-To: TeXhax@tex.ac.uk
+Errors-To: owner-texhax-digest@nottingham.ac.uk
+Precedence: bulk
+Date: Thu, 7 Mar 1996 17:29:03 +0000
+Sender: owner-texhax-digest@nottingham.ac.uk
+
+
+TeXhax Digest Thursday, 7 March 1996 Volume 96 : Number 003
+
+(incorporating UKTeX Digest)
+
+Today's Topics:
+
+ TeX/LaTeX on Windows
+ Re: TeXhax Digest V96 #2
+ Re: TeXhax Digest V96 #2
+ spell checker for sun(UNIX)
+ permille, fractions, space, \afterpage and procs
+ Table of contents, TUGboat 16(2)
+ Announcing TUG'96 in Dubna (Russia) July 28-Aug 2 1996
+
+----------------------------------------------------------------------
+
+From: bkph@ai.mit.edu (Berthold K.P. Horn)
+Date: Wed, 17 Jan 1996 16:11:04 -0500
+Subject: TeX/LaTeX on Windows
+
+ From: "Sam Nelson (CO)" <Sam.Nelson@cs.stir.ac.uk
+ Date: Tue, 9 Jan 1996 10:27:33 +0000 (GMT)
+ Subject: TeX/LaTeX on Windows
+
+ Not being a beginner, this is going to sound a bit weird, I suppose, but
+ despite having used TeX and LaTeX around here for more than a decade, I've
+ never been asked (very much) to put them up on PC platforms, which are in
+ little use in this department. However, I've just been asked about running
+ LaTeX on a Windows platform, and it occurs to me that I know nothing whatever
+ about this.
+
+ Starting from scratch then, in an early-1996 Windows environment, where
+ should one go to get the ideal TeX/LaTeX setup? I assume there are
+ `commercial' solutions (we use Textures on Macintosh already) but I don't
+ know how much money might be available, so a price spectrum from zero to
+ lottery-winner is appropriate, I think.
+
+Check out http://www.YandY.com (or contact sales-help@Yandy.com).
+
+ Thanks for any suggestions,
+
+ Sam.
+
+> Sam Nelson, Comp Sci, Stirling U, FK9 4LA, Scotland ,->0->M
+> Email: sam@cs.stir.ac.uk Pager: 0839 456640 I->3-+->2->R=->-+->4->O
+> Tel: +44 1786 467443 Fax: +44 1786 464551 `->1->S=->-'
+> URL: http://www.cs.stir.ac.uk/~sam R$+@$+ $@smtp$#$2$:$1@$2
+
+DISCLAIMER: respondent has connections with Y&Y
+
+------------------------------
+
+From: Jonathan Dixon <dixonj@rococo.Colorado.EDU>
+Date: Fri, 19 Jan 96 13:59:29 MST
+Subject: Re: TeXhax Digest V96 #2
+
+You wrote:
+> From: "Dr. Ke Chen" <chen@cis.pku.edu.cn>
+> Date: Fri, 12 Jan 1996 17:11:18 +0800 (GMT+0800)
+> Subject: latex2e IEEEtran macro wanted.
+>
+> I found the previous IEEEtran.sty did not work in LaTex2e. Some days ago,
+> a person sent me his own new IEEEtran.sty but there are a few problems
+> such as that the title of section is always typeset with a small font,
+> etc. As a result, I would appreciate it if anyone could tell me where
+> a complete IEEEtran macro in latex2e is available.
+>
+> Ke Chen
+>
+> #========================================================================#
+> # Dr. Ke Chen, Associate Professor Tel: +86-10-2751935(O) #
+> # National Lab of Machine Perception +86-10-2585703(H) #
+> # The Center of Information Science Fax: +86-10-2563883,2552779 #
+> # Peking University Email: chen@cis.pku.edu.cn #
+> # Beijing 100871, China chenke@pku.edu.cn #
+> #========================================================================#
+
+I have a version I modified to make an IEEEtran.cls. I haven't tested it
+extensively, but for what I've done it seems to work. If someone can
+provide a place to post it to, I'd be more than willing to have it
+distributed.
+
+- --
+Jon Dixon
+dixonj@colorado.edu
+http://spot.colorado.edu/~dixonj/
+
+
+------------------------------
+
+From: "Dr. Ke Chen" <chen@cis.pku.edu.cn>
+Date: Mon, 22 Jan 1996 09:20:40 +0800 (GMT+0800)
+Subject: Re: TeXhax Digest V96 #2
+
+On Fri, 19 Jan 1996, Jonathan Dixon wrote:
+
+> Date: Fri, 19 Jan 96 13:59:29 MST
+> From: Jonathan Dixon <dixonj@rococo.Colorado.EDU>
+> To: TeXhax@tex.ac.uk
+> Cc: chen@cis.pku.edu.cn
+> Subject: Re: TeXhax Digest V96 #2
+>
+> You wrote:
+> > From: "Dr. Ke Chen" <chen@cis.pku.edu.cn>
+> > Date: Fri, 12 Jan 1996 17:11:18 +0800 (GMT+0800)
+> > Subject: latex2e IEEEtran macro wanted.
+> >
+> > I found the previous IEEEtran.sty did not work in LaTex2e. Some days ago,
+> > a person sent me his own new IEEEtran.sty but there are a few problems
+> > such as that the title of section is always typeset with a small font,
+> > etc. As a result, I would appreciate it if anyone could tell me where
+> > a complete IEEEtran macro in latex2e is available.
+> >
+> > Ke Chen
+> >
+> I have a version I modified to make an IEEEtran.cls. I haven't tested it
+> extensively, but for what I've done it seems to work. If someone can
+> provide a place to post it to, I'd be more than willing to have it distributed.
+>
+> --
+> Jon Dixon
+> dixonj@colorado.edu
+> http://spot.colorado.edu/~dixonj/
+>
+
+Thanks very much for your kind response. I think you might upload it
+to CTAN archives. In addition, I would appreciate it if you would send
+this macro to me by email for my immediate use.
+
+Ke Chen
+#========================================================================#
+# Dr. Ke Chen, Associate Professor Tel: +86-10-2751935(O) #
+# National Lab of Machine Perception +86-10-2585703(H) #
+# The Center of Information Science Fax: +86-10-2563883,2552779 #
+# Peking University Email: chen@cis.pku.edu.cn #
+# Beijing 100871, China chenke@pku.edu.cn #
+#========================================================================#
+
+
+------------------------------
+
+From: dhlee@pearl.cs.pusan.ac.kr (DoHoon Lee)
+Date: Tue, 30 Jan 96 19:02:12 KST
+Subject: spell checker for sun(UNIX)
+
+I am looking for spell checker utility for LaTeX.
+Does anybody know where this be found ? Or
+If you have spell checker for LaTeX, please let me got it.
+Thanks in advance,
+
+Dohoon Lee
+============================================================
+Dept. of Computer Science
+Pusan National University
+Pusan 609-735, Korea
+dhlee@pearl.cs.pusan.ac.kr
+============================================================
+
+
+------------------------------
+
+From: Kris Lockyear <K.Lockyear@soton.ac.uk>
+Date: Tue, 30 Jan 1996 16:17:08 PST
+Subject: permille, fractions, space, \afterpage and procs
+
+Could anyone help me with five queries? The first two are perhaps simple, the last two I am unsure about. FYI I am using
+LaTeX2e with emTeX on a PC.
+
+1) How can I get a permille sign (like a percent sign but with two zeros -- 0/00). I scanned the tables in The Companion and
+The Book but don't see it.
+
+2) How can I get nice in-text fractions? By that I mean not 1/2 which $1/2$ would give you, or \frac{1}{2}, but a fraction
+where there is a small 1, a diagonal line and a small 2. (I usually use the Times package and a PS laserprinter).
+
+3) I have a chapter which is nearly all text with some headings/sub-headings and footnotes. In places, LaTeX has increased
+the white space between paragraphs or around sub-headings in order to place a \section at the top of a page. This looks very
+ugly and I would rather have a little extra white space at the bottom of the page, than extra white space between paragraphs.
+ Is there any way I can stop LaTeX adding this extra space?
+
+4) In another chapter I have a lot of tables and figures. Following advice in The Companion, I often use
+\afterpage{\clearpage} to flush out the figures etc. Unfortunately, this occasionally results in a blank page with the header
+at the top, and a footnote at the bottom. Any suggestions?
+
+5) Lastly, later this year I have to produce a set of conference proceedings. Could I have some pointers to classes/packages
+and/or articles which would enable me to do this in LaTeX2e. Specifically, including authors names and addresses in paper
+titles and the TOC etc.
+
+Many thanks in advance,
+
+Kris Lockyear.
+
+
+
+------------------------------
+
+From: Robin Fairbairns <Robin.Fairbairns@cl.cam.ac.uk>
+Date: Thu, 01 Feb 1996 14:36:53 +0000
+Subject: Table of contents, TUGboat 16(2)
+
+It appears that we (the TUGboat production team) did not post this
+table of contents. Apologies to all ...
+
+ TUGboat
+
+ Volume 16, Number 2 / June 1995
+ Guest Editor: Malcolm Clark
+ ================================
+
+Addresses 99
+
+General Delivery
+ Michel~Goossens 101
+ Opening words
+ Malcolm~Clark
+ Introduction 101
+
+Theme Issue on Portability of Electronic Documents
+
+ Michel~Goossens and Janne~Saarela
+ A practical introduction to SGML 103
+ Peter~Flynn
+ HTML & TeX: Making them sweat 146
+ Geeti~Granger
+ The inside story of life at Wiley with SGML, 151
+ LaTeX and Acrobat
+ Mark~D.~Doyle
+ The Los Alamos e-print archives:
+ HyperTeX in action 154
+ Otfried~Schwarzkopf
+ The Hyperlatex story 159
+ Yannis~Haralambous and Sebastian~Rahtz
+ LaTeX, hypertext and PDF, or the entry of TeX
+ into the world of hypertext 162
+ Michel~Goossens and Janne~Saarela
+ TeX to HTML and back 174
+
+News & Announcements
+
+ Calendar 215
+ Late-Breaking News
+ Mimi~Burbank and Barbara Beeton
+ Production notes 100
+ Coming next issue
+
+TUG Business
+ Institutional members 217
+
+Forms
+ TUG membership application
+
+Advertisements
+ TeX consulting and production services 218
+ Index of advertisers 218
+
+%%% end of file
+
+
+------------------------------
+
+From: Michel Goossens <Michel.Goossens@cern.ch>
+Date: Wed, 7 Feb 1996 00:44:11 +0100 (MET)
+Subject: Announcing TUG'96 in Dubna (Russia) July 28-Aug 2 1996
+
+======================================================================
+
+ The 17th Annual TeX Users Group Meeting
+
+
+ polyglot polytechnic
+
+ polymath POLY-TeX polymath
+
+ polytechnic polyglot
+
+
+ July 28-- August 2, 1996
+
+ in JINR, Dubna, Russia
+
+ Organized jointly by: TUG, CyrTUG, JINR
+
+ CALL FOR PAPERS
+
+======================================================================
+Dear TeX Friends!
+
+So the time has arrived when TeX, the Polyglot, who has for many years
+happily ``spoken'' many different languages written in the Latin
+alphabet, extends its knowledge in the field of other alphabets.
+During the Summer of 1996, a visit is planned to Russia where TeX
+will have its first practical session in Cyrillic.
+
+Russia is that large country, with its enormous spaces, inhabited by
+those enigmatic Russians, who started the century with a Revolution,
+and ended it with ``Perestroyka''. A country who founded a brilliant
+mathematical school, colonized the Cosmos, and conquered the world
+with its literature, music, and ballet.
+
+So what awaits the TeX user who plans to attend TUG'96?
+
+At Moscow's international Sheremetevo-2 Airport, conference
+participants will be met by a member of the organizing committee and
+escorted by bus to Dubna, where from July 28 to August 2, the 17th TUG
+conference will take place (see below for more about Dubna). If, for
+some reason or another, you envision cold, the taiga, and white bears,
+then we can reassure you, Dubna is in the European part of Russia, and
+if you want to see the ice on the nordic oceans, you will have to
+travel further than you would to reach the subtropics of the Black Sea
+shore. Indeed, you will soon find that the pine forest in which the
+comfortable town of Dubna is situated will remind you of a park. In
+July and August, the weather is mostly warm and sunny, with an average
+temperature of 28 degrees Centigrade. Guests will be housed in a
+comfortable hotel on the banks of the Volga river in single or double
+rooms (hot water, shower, telephone, and television). The Russian
+``cuisine'' is characterized by its abundance, so one can forget about
+slimming.
+
+The social program includes a picnic on the picturesque banks of the
+Volga, where we will be taken by boat, a bus excursion to Sergiev
+Posad (the center of the Russian Orthodox Church, where Andrey
+Slephkhin works--- see his article on page 373 of TUGboat 16#4 or the
+EuroTeX'95 proceedings page 331), and, on the last day a visit to
+Moscow, following which the participants can be dropped of at the
+airport to fly home, or at one of the railway stations, if they want
+to prolong their visit.
+
+The conference's preliminary program will be announced as soon as we
+receive from you, dear readers, proposals for presentations, courses
+that you would like to teach or attend, poster sessions, or any
+problem(s) or subject(s) that interest you. Please send your
+suggestions to the conference electronic address
+
+TUG96@pds.jinr.ru
+=================
+
+The theme of the Conference is: PolyTeX, TeX or the art of
+multi-lingual, maths and technical typesetting.
+
+
+ polyglot polytechnic
+
+ polymath POLY-TeX polymath
+
+ polytechnic polyglot
+
+
+===================
+PROGRAM COMMITTEE
+===================
+
+Evgeniy Vasilievitch Pankratiev Moscow, Russia
+ Email: pankrat@shade.msu.ru
+
+Michel Goossens Geneva, Switzerland
+ Email: goossens@cern.ch
+
+Mimi Burbank Florida, USA
+ Email: mimi@scri.fsu.edu
+
+
+===========
+DEADLINES
+===========
+
+Submission of abstracts February 20 <<<<<<<
+
+Acceptance signified to authors February 29 <<<<<<<
+
+Preliminary articles March 31 <<<<<<<
+
+Proposals for workshops, demos,
+ and poster sessions April 20
+
+Registration and transfer of a
+non-refundable sum of $100
+per person to a Dubna bank May 31
+
+Visa supporting information June 5
+
+Revised articles June 10 <<<<<<<
+
+Start of Conference July 28
+
+
+
+======================
+ORGANIZING COMMITTEE
+======================
+
+Vladimir Vasilievitch Korenkov Dubna, Russia
+ Email: korenkov@cv.jinr.ru
+
+Irina Anatolievna Makhovaya Moscow, Russia
+ Email: irina@mir.msk.su
+
+Sebastian Rahtz Oxford, UK
+ Email: s.rahtz@elsevier.co.uk
+
+
+======================
+PRACTICAL INFORMATION
+=======================
+
+- - ----------------
+Conference costs
+- - ----------------
+
+The Conference Committee foresees a cost in the range 550--600 USD.
+This sum includes the complete cost of the conference, namely the
+registration fee, lodging (6 nights with six breakfasts, lunches, and
+dinners), coffee/tea breaks, social events, and transport from
+Sheremetevo Airport to Dubna. The payment should be made in the
+following way: a `non-refundable' sum of $100 per person should be
+transferred to a bank account (to be announced) before June 1st.
+After receipt of that sum an official invitation, necessary for
+obtaining a visa (see below), will be faxed to the participant. The
+rest will be payable in cash upon arrival at the Conference (no credit
+cards or cheques can be used in Dubna). %For participants of
+economically weaker countries or for students %we hope to arrange
+partial support via sponsors. We hope to arrange bursary funds for
+support of students and those participants who demonstrate need.
+
+- - -----
+Visas
+- - -----
+
+Most of the visitors from outside Russia will need a visa to attend
+the Conference. Therefore, for arranging a visa into Russia,
+participants should inform the
+
+ Mrs. N. Dokalenko, Conference Secretariat
+ E-mail nataly@ypr.jinr.dubna.su;
+ Fax 7 095 975 2381 or 7 09621 65 891
+
+of their and (possibly) the accompanying person(s)'s full name, date
+of birth, citizenship, passport number, arrival and departure dates.
+The Secretariat will forward by fax the visa support message to the
+participants with which they should apply for visas to the nearest
+Russian Embassy or Consulate. Please note that you should apply for a
+visa valid for Dubna, Moscow, and Sergiev Posad. (If you plan to visit
+other cities in Russia you should obtain the relevant documents and
+join them to the visa application, so that the names of all places to
+be visited can be entered on the visa form as required.)
+
+- - --------------
+Transportation
+- - --------------
+
+The Organizing Committee will arrange direct transportation by bus
+from the Sheremetevo-2 Airport to Dubna (130 km north of Moscow). The
+Secretariat should be informed of the flight number, precise date and
+time of arrival (Moscow time) `no later than' four working days before
+a participant wishes to be met at the airport. It is our intention to
+have each participant met by a member of Organizing Committee.
+Details will be available later.
+
+
+====================
+WELCOME TO DUBNA !
+====================
+
+Dubna was founded in 1956 when the Convention establishing the Joint
+Institute for Nuclear Research was signed. The town is situated on the
+picturesque banks of the Volga river and the Moscow sea 120 km to the
+north of Moscow. One can reach Dubna from Moscow within 2 hours going
+by car, by bus or by express train. It will take you 1.5 hours to go
+to Dubna from the Sheremetevo-2 International Airport. Waterways
+connect Dubna not only to the Russian Volga cities, but also to the
+waters of the Black, the Caspian, the Baltic and the White seas.
+
+There is no harmful environmental impact from the industrial plants;
+this together with the large tracts of forest in the environs of
+Dubna, and the vast water area dotted with small islands, makes the
+area quite attractive for tourism and rest. The Volga embankment is
+one of the prettiest parts of the town. In springtime, the streets of
+Dubna are full of the odour of lilacs, the apple trees are pink-white;
+in summer, lime trees, maples, birch trees and poplars make the town
+seem totally green; in autumn the town is all golden excepting the
+evergreen of old pine trees. The town's modern look harmonizes with
+the quietness of the surrounding forest. The town was built in the
+midst of a forest. There are separate patches of trees in the town
+itself, and the town park is just a part of the forest. It takes just
+a few minutes to get to the forest from the shopping centre on foot. A
+few minutes' walk and you are outside the city limits!
+
+Small as it is, Dubna is a real metropolis. It is a scientific
+metropolis. It is a ``big little city'', as a visiting American
+scientist called it many years ago. Since the foundation of the Joint
+Institute for Nuclear Research (JINR), the name of Dubna has
+constantly been in the pages of the world's newspapers and journals.
+Dubna is one of the world centres for fundamental research in nuclear
+physics. The Joint Institute plays an important role as a coordinator
+of investigations of the scientists from 18 JINR member-state
+institutes. Wide international scientific and technical cooperation
+is one of the fundamental concepts of the JINR.
+
+Dubna is indeed a town of international friendship. Foreign speech can
+be heard everywhere. But the words, no matter in which language they
+are pronounced, are clear to everybody: friendly cooperation and
+fraternity unite all the physicists and mathematicians living and
+working in Dubna into an international scientific community.
+
+On October 3, 1994, Dubna opened its doors to its first university,
+``International University of Dubna: Nature, Society and Man''. The
+university is composed of five "cathedra" or faculties, including
+socioeconomic sciences, ecology and earth science, computer education,
+linguistics, and health and physical education. Two more faculties---
+law and government and technology--- are also being contemplated.
+
+The town has great experience in holding international conferences,
+and exchanges of delegations between countries in the sphere of
+science, education and culture. Dubna and La Crosse, Wisconsin, USA,
+are sister cities. Dubna is famous for its hospitality. Famous
+scientists, public figures and statesmen from different countries
+visit Dubna. They are always impressed by the gracious welcome they
+receive in Dubna and warm generosity which Dubna residents
+demonstrate.
+
+=======================
+A FEW WORDS ON MOSCOW
+=======================
+
+Moscow, the capital of Russia, has a population of some nine million
+people. It is a city rich in cultural, architectural and historical
+monuments, and, at the same time, boasts a rapidly developing modern
+urban community with brand new blocks of flats, long, straight and
+broad avenues, parks, gardens, stadiums, schools, cinemas, department
+stores, recreation centres, bridges and highways. Though
+forward-looking, it cherishes the memory of its past, and its old
+sections lend it a special charm.
+
+
+Michel Goossens / TUG President
+
+
+------------------------------
+
+End of TeXhax Digest V96 #3
+***************************
+
+
+About TeXhax...
+
+Please send contributions to: TeXhax@tex.ac.uk
+
+Subscription and unsubscription requests:
+ send a one line mail message to TeXhax-Request@tex.ac.uk
+ containing either subscribe texhax
+ or unsubscribe texhax
+If you have problems with un/subscribing, please mail owner-texhax@nott.ac.uk
+
+To obtain the Frequently Asked Questions (FAQ) lists for TeX, send a
+message with no subject to fileserv@shsu.edu, consisting of
+SENDME FAQ
+
+For information on the TeX Users Group, please send a message to
+TUG@TUG.org, or write TeX Users Group, 1850 Union Street, #1637
+San Francisco CA 94123 (phone: 1 415 982 8449, fax: 1 415 982 8559)
+
+Backnumbers of all the digests are stored in the Comprehensive TeX
+Archive Network (CTAN) and can be retrieved on the Internet by
+anonymous ftp. The hosts comprising CTAN include, among others,
+ ftp.dante.de (129.69.1.12) -- Germany
+ ftp.shsu.edu (192.92.115.10) -- USA
+ ftp.tex.ac.uk (128.232.1.87) -- UK
+Please use your nearest server, to keep network load down.
+The file /tex-archive/CTAN.sites on each of these hosts gives a
+list of other sites which maintain full or partial mirrors of the CTAN.
+Alternatively, finger ctan_us@ftp.shsu.edu for full details.
+
+TeXhax Digest back issues are filed below /tex-archive/digests/texhax/
+Keyword-In-Context indexes are filed in /tex-archive/digests/indexes/
+
+A Hypermail version of TeXhax is also available on the World-Wide Web at URL
+http://www.tex.ac.uk/tex-archive/digests/hyper/
+
+\bye
+
diff --git a/info/digests/texhax/96/texhax.04 b/info/digests/texhax/96/texhax.04
new file mode 100644
index 0000000000..e02a32552d
--- /dev/null
+++ b/info/digests/texhax/96/texhax.04
@@ -0,0 +1,257 @@
+From texhax-digest-outgoing-request@nottingham.ac.uk Mon Mar 11 16:50:38 1996
+Received: from jess.ccc.nottingham.ac.uk (jess.ccc.nottingham.ac.uk [128.243.40.193]) by granby.ccc.nottingham.ac.uk (8.6.12/8.6.12) with ESMTP id QAA11662 for <cczdao@unix.ccc.nottingham.ac.uk>; Mon, 11 Mar 1996 16:50:36 GMT
+Message-Id: <199603111650.QAA11662@granby.ccc.nottingham.ac.uk>
+Received: from nottingham.ac.uk by jess.ccc.nottingham.ac.uk
+ id <03918-0@jess.ccc.nottingham.ac.uk>;
+ Mon, 11 Mar 1996 16:12:56 +0000
+From: Majordomo list server <owner-texhax-digest@nottingham.ac.uk>
+To: texhax-digest@nottingham.ac.uk
+Subject: TeXhax Digest V96 #4
+Reply-To: TeXhax@tex.ac.uk
+Errors-To: owner-texhax-digest@nottingham.ac.uk
+Precedence: bulk
+Date: Mon, 11 Mar 1996 16:12:56 +0000
+Sender: owner-texhax-digest@nottingham.ac.uk
+
+
+TeXhax Digest Monday, 11 March 1996 Volume 96 : Number 004
+
+(incorporating UKTeX Digest)
+
+Today's Topics:
+
+ UKTUG Meeting: March 20 1996 --- TeX and the Internet
+ revised dealines for TUG'96
+ Memo Class?
+ startsection
+
+----------------------------------------------------------------------
+
+From: David Carlisle <carlisle@ma.man.ac.uk>
+Date: Wed, 21 Feb 1996 15:07:21 GMT
+Subject: UKTUG Meeting: March 20 1996 --- TeX and the Internet
+
+The following message is a courtesy copy of an article
+that has been posted as well.
+
+UKTUG is holding a one day meeting at Warwick on `TeX and the Internet'
+
+Speakers include:
+
+Michel Goossens: LaTeX2HTML
+Malcolm Clark HTML3 maths
+Yannis Haralambous (To be announced)
+Robin Fairbairns The CTAN Archives
+Sebastian Rahtz Acrobat, TeX and the Web
+
+Details and a booking form are available on WWW
+
+http://www.ma.man.ac.uk/~carlisle/form2.html
+
+(They are in the post to all UKTUG members already.)
+
+David Carlisle
+For UKTUG
+
+
+------------------------------
+
+From: Michel Goossens <Michel.Goossens@cern.ch>
+Date: Tue, 20 Feb 1996 17:16:38 +0100 (MET)
+Subject: revised dealines for TUG'96
+
+ ============================
+ REVISED DEADLINES for TUG'96
+ ============================
+
+Submission of abstracts March 11 << new date
+
+Acceptance signified to authors March 20 << new date
+
+Preliminary articles April 12 << new date
+
+Proposals for workshops, demos,
+ and poster sessions April 20
+
+Registration and transfer of a
+non-refundable sum of $100
+per person to a Dubna bank May 31
+
+Visa supporting information June 5
+
+Revised articles June 10 << firm date
+
+Start of Conference July 28
+
+======================================================================
+
+Please send your proposals for contributions to the TUG'96 Organizing
+Committee at their email address TUG96@pds.jinr.ru.
+
+The date of June 10th is a "firm date", since we want to have all
+articles in a final form ready to be printed by the time of the
+Conference, and therefore need them to go to the printer by late
+June. As a consequence final papers not received by June 10 will not
+appear in the Proceedings Issue of TUGboat.
+
+The most up-to-date information about TUG'96 will be available at the
+URL: http://www.scri.fsu.edu/~mimi/tug96/tug96.html
+or, by ftp from the CTAN archives in
+ tex-archive/usergrps/tug/tug96/news-mon.asc
+
+The TUG'96 Organizing Committee
+
+
+------------------------------
+
+From: rpreuss@bbn.com (Rob Preuss)
+Date: Fri, 8 Mar 1996 17:07:58 -0500
+Subject: Memo Class?
+
+Q: Regarding Memorandum Class(es) for LaTeX2e.
+
+ (short form)
+ Is there a popular/standard document class for creating memos
+ with LaTeX2e (and the AMS-LaTeX 1.2 add-ons) using TeXtures?
+
+ (long form)
+ 10 years ago I wrote my own "memo" style using the standard
+ "article" style as a starting point. Main features included:
+
+ - double column option
+ - 10pt, 11pt, 12pt options
+ - titlesection (created by \maketitle) sorta like
+
+ MEMORANDUM
+
+ To: ...
+ From: ...
+ Subject: ...
+ Date: ...
+ Copies To: ...
+ _______________________
+
+ - RHS header with
+ + short form contents of the "To:" field,
+ + contents of "Date:" field, and
+ + Page x of xx.
+ - footer with copyright info.
+
+ Its been great, but now I'd like to upgrade to LaTeX2e while
+ taking advantage of all the nifty stuff offered by AMS-LaTeX.
+
+ 1. Do I have to reinvent this or does something similar exist?
+ 2. If so, should I write a new class or just a package?
+ 3. Should this new class/package be more related to the
+ "article" class or the "amsart" class?
+
+Thanks very much!
+
+______________
+ Dr. Robert D. Preuss (Rob)
+ BBN Acoustic Technologies Voice: 617-873-3773
+ 70 Fawcett Street Fax: 617-873-2894 or 2918
+ Cambridge, MA 02138 USA Internet: rpreuss@bbn.com
+
+
+
+------------------------------
+
+From: Mr Marcilo Alves <marcilio@liverpool.ac.uk>
+Date: Mon, 11 Mar 1996 11:26:32 +0000 (GMT)
+Subject: startsection
+
+Hi,
+
+I'm trying to redefine the \section, \chapter look by
+using the \@startsection command (Companion, page 24).
+
+I always get the message
+
+ you can't use \spacefactor in vertical mode
+
+
+Coul you help me?
+
+Here goes my input
+
+
+\documentclass{book}
+\renewcommand{\section}{\@startsection
+ {\section}%
+ {1}%
+ {2mm}%
+ {2mm}%
+ {2mm}
+ {\itshape}}%
+\begin{document}
+\section{Hi}
+This is ...
+\end{document}
+
+
+and here the message
+
+This is TeX, Version 3.1415 (C version 6.1)
+(lixo.tex
+LaTeX2e <1994/12/01> patch level 1
+(/apps/TeX/inputs/tex/latex2e/base/book.cls
+Document Class: book 1994/12/09 v1.2x Standard LaTeX document class
+(/apps/TeX/inputs/tex/latex2e/base/bk10.clo))
+No file lixo.aux.
+! You can't use `\spacefactor' in vertical mode.
+\@->\spacefactor
+ \@m
+l.10 \section
+ {Hi}
+?
+
+Thanks
+
+
+marcilio
+
+
+------------------------------
+
+End of TeXhax Digest V96 #4
+***************************
+
+
+About TeXhax...
+
+Please send contributions to: TeXhax@tex.ac.uk
+
+Subscription and unsubscription requests:
+ send a one line mail message to TeXhax-Request@tex.ac.uk
+ containing either subscribe texhax
+ or unsubscribe texhax
+If you have problems with un/subscribing, please mail owner-texhax@nott.ac.uk
+
+To obtain the Frequently Asked Questions (FAQ) lists for TeX, send a
+message with no subject to fileserv@shsu.edu, consisting of
+SENDME FAQ
+
+For information on the TeX Users Group, please send a message to
+TUG@TUG.org, or write TeX Users Group, 1850 Union Street, #1637
+San Francisco CA 94123 (phone: 1 415 982 8449, fax: 1 415 982 8559)
+
+Backnumbers of all the digests are stored in the Comprehensive TeX
+Archive Network (CTAN) and can be retrieved on the Internet by
+anonymous ftp. The hosts comprising CTAN include, among others,
+ ftp.dante.de (129.69.1.12) -- Germany
+ ftp.shsu.edu (192.92.115.10) -- USA
+ ftp.tex.ac.uk (128.232.1.87) -- UK
+Please use your nearest server, to keep network load down.
+The file /tex-archive/CTAN.sites on each of these hosts gives a
+list of other sites which maintain full or partial mirrors of the CTAN.
+Alternatively, finger ctan_us@ftp.shsu.edu for full details.
+
+TeXhax Digest back issues are filed below /tex-archive/digests/texhax/
+Keyword-In-Context indexes are filed in /tex-archive/digests/indexes/
+
+A Hypermail version of TeXhax is also available on the World-Wide Web at URL
+http://www.tex.ac.uk/tex-archive/digests/hyper/
+
+\bye
+
diff --git a/info/digests/texhax/96/texhax.05 b/info/digests/texhax/96/texhax.05
new file mode 100644
index 0000000000..c1a77277bb
--- /dev/null
+++ b/info/digests/texhax/96/texhax.05
@@ -0,0 +1,224 @@
+From texhax-digest-outgoing-request@nottingham.ac.uk Mon Apr 15 11:53:11 1996
+Received: from jess.ccc.nottingham.ac.uk (jess.ccc.nottingham.ac.uk [128.243.40.193]) by granby.ccc.nottingham.ac.uk (8.6.12/8.6.12) with ESMTP id LAA15365 for <cczdao@unix.ccc.nottingham.ac.uk>; Mon, 15 Apr 1996 11:53:11 +0100
+Message-Id: <199604151053.LAA15365@granby.ccc.nottingham.ac.uk>
+Received: from nottingham.ac.uk by jess.ccc.nottingham.ac.uk
+ id <27588-0@jess.ccc.nottingham.ac.uk>;
+ Mon, 15 Apr 1996 11:52:28 +0100
+From: Majordomo list server <owner-texhax-digest@nottingham.ac.uk>
+To: texhax-digest@nottingham.ac.uk
+Subject: TeXhax Digest V96 #5
+Reply-To: TeXhax@tex.ac.uk
+Errors-To: owner-texhax-digest@nottingham.ac.uk
+Precedence: bulk
+Date: Mon, 15 Apr 1996 11:52:28 +0100
+Sender: owner-texhax-digest@nottingham.ac.uk
+
+
+TeXhax Digest Monday, 15 April 1996 Volume 96 : Number 005
+
+(incorporating UKTeX Digest)
+
+Today's Topics:
+
+ Re: fractions, space, \afterpage (TeXhax)
+ Re: startsection (TeXhax Digest V96 #4)
+ Plain TeX query
+ disc set and printer driver for LaTex 2e
+
+----------------------------------------------------------------------
+
+From: Donald Arseneau <asnd@triumf.ca>
+Date: Thu, 07 Mar 1996 20:47:44 PST
+Subject: Re: fractions, space, \afterpage (TeXhax)
+
+In TeXhax Digest V96 #3, Kris Lockyear <K.Lockyear@soton.ac.uk> wrote:
+
+% 2) How can I get ... 1/2 with small 1, a diagonal line and a small 2.
+
+It is in The TeXbook, exercise 11.6. You probably have manmac.tex and/or
+eplain.tex on your system somewhere; you can copy the definition out of
+there to avoid re-typing it.
+
+% 3) LaTeX increased white space between paragraphs to place a \section at
+the
+% top of a page. This looks very ugly.
+
+
+Declare \raggedbottom (described in the LaTeX manual)
+
+% 4) I often use \afterpage{\clearpage} to flush out the figures etc.
+% this occasionally results in a blank page with a footnote at the bottom.
+
+a) In my opinion, this is never necessary. It only helps when figures are
+*stuck* with impossible placement requirements. You are better off giving
+realistic placement options for your figures and setting higher values for
+\topfraction and \bottomfraction (say 0.8) and a lower \textfraction (0.2).
+Re-read the LaTeX manual discussion of these.
+
+b) Don't write long footnotes.
+
+c) You can avoid most split footnotes using:
+\raggedbottom \addtolength{\topskip}{0pt plus 1cm minus 0.2pt}
+(No, the top won't be ragged; just the bottom.)
+
+Donald Arseneau asnd@reg.triumf.ca
+
+
+------------------------------
+
+From: Robin Fairbairns <Robin.Fairbairns@cl.cam.ac.uk>
+Date: Tue, 12 Mar 1996 09:33:00 +0000
+Subject: Re: startsection (TeXhax Digest V96 #4)
+
+Mr Marcilo Alves writes:
+>I'm trying to redefine the \section, \chapter look by
+>using the \@startsection command (Companion, page 24).
+
+Having not read the Companion, page 16 (I think it is), where it tells
+you to:
+
+ \makeatletter
+>\renewcommand{\section}{\@startsection
+> {\section}%
+> {1}%
+> {2mm}%
+> {2mm}%
+> {2mm}
+> {\itshape}}%
+ \makeatother
+
+This is one of the most-FAQs. The Companion is _not_ a beginner's
+book; if you've never programmed LaTeX macros before *PLEASE*
+(everyone, not just Mr. Alves!) read the introductory pages first
+(there are only twenty or so of them).
+
+
+------------------------------
+
+From: cgm@ssci.liv.ac.uk (Colin Mason)
+Date: Thu, 28 Mar 1996 18:47:11 GMT
+Subject: Plain TeX query
+
+% Could someone please answer a question which has been puzzling me for some
+% while because, unless I misunderstand the TeXbook, TeX is not doing what I
+% think it should do. Consider the following input file:
+%-------------------------------------------------------------------------------
+\chardef\return=`\^^M
+%
+\def\dopling{%
+ \ifcat\space\noexpand\next
+ \wlog{<Pling><Space>}%
+ \else \ifcat\return\noexpand\next
+ \wlog{<Pling><Return>}%
+ \else \ifcat\bgroup\noexpand\next
+ \wlog{<Pling><Left brace>}%
+ \else
+ \wlog{<Pling><Other>}%
+ \fi \fi \fi}%
+%
+\catcode`\!=\active
+%
+\def!{\futurelet\next\dopling}%
+%
+! Hallo world.
+!
+!{Anyone for Tennis?}%
+!%
+! Just another line%
+!---and another part of the same line
+!
+! \bye
+%-------------------------------------------------------------------------------
+% Not a very sensible input file I know but it's just meant to illustrate the
+% problem. When run through TeX, I get the following log file (without the
+% leading `% ' of course):
+%-------------------------------------------------------------------------------
+% This is TeX, C Version 3.14t3 (format=plain 93.5.19) 28 MAR 1996 18:31
+% **test
+% (test.tex
+% <Pling><Space>
+% <Pling><Space>
+% <Pling><Left brace>
+% <Pling><Return>
+% <Pling><Space>
+% <Pling><Other>
+% <Pling><Space>
+% <Pling><Space>
+% [1] )
+% Output written on test.dvi (1 page, 308 bytes).
+%-------------------------------------------------------------------------------
+% I understand all of these except the <Pling><Return> generated by the input
+% line `!%'. Why should a comment character generate a <Return>?
+%
+% Thanks in anticipation.
+%
+% Colin Mason (cgm@sunserver.ssci.liv.ac.uk)
+
+------------------------------
+
+From: Andrew Jones $STAFF <JONEAN@trentcollege.nott.sch.uk>
+Date: Mon, 25 Mar 1996 09:12:44 +0000
+Subject: disc set and printer driver for LaTex 2e
+
+Please can anyone help me??
+
+I have used Latex in the past, but unfortunately had to remove it. I
+now possess a Canon BJ10ex printer, and I cannot get the exact driver for it.
+I've been given several to try, but to no avail.
+I also understand that I no longer have the latest version of LaTex ( I
+bought mine in 1990). How is LaTex 2e different?
+
+Could anyone please advise me on where I may get a proper printer
+driver for the BJ10ex, and possibly a disc set for LaTex 2e.
+
+I use Wordperfect 6.0 for word-processing. I recall reading somewhere
+that there is a LaTex converter for WP6.
+
+My own machine is not on the Internet, and therefore I cannot
+download, I would therefore need proper discs. Any help would be
+greatly appreciated.
+
+
+------------------------------
+
+End of TeXhax Digest V96 #5
+***************************
+
+
+About TeXhax...
+
+Please send contributions to: TeXhax@tex.ac.uk
+
+Subscription and unsubscription requests:
+ send a one line mail message to TeXhax-Request@tex.ac.uk
+ containing either subscribe texhax
+ or unsubscribe texhax
+If you have problems with un/subscribing, please mail owner-texhax@nott.ac.uk
+
+To obtain the Frequently Asked Questions (FAQ) lists for TeX, send a
+message with no subject to fileserv@shsu.edu, consisting of
+SENDME FAQ
+
+For information on the TeX Users Group, please send a message to
+TUG@TUG.org, or write TeX Users Group, 1850 Union Street, #1637
+San Francisco CA 94123 (phone: 1 415 982 8449, fax: 1 415 982 8559)
+
+Backnumbers of all the digests are stored in the Comprehensive TeX
+Archive Network (CTAN) and can be retrieved on the Internet by
+anonymous ftp. The hosts comprising CTAN include, among others,
+ ftp.dante.de (129.69.1.12) -- Germany
+ ftp.shsu.edu (192.92.115.10) -- USA
+ ftp.tex.ac.uk (128.232.1.87) -- UK
+Please use your nearest server, to keep network load down.
+The file /tex-archive/CTAN.sites on each of these hosts gives a
+list of other sites which maintain full or partial mirrors of the CTAN.
+Alternatively, finger ctan_us@ftp.shsu.edu for full details.
+
+TeXhax Digest back issues are filed below /tex-archive/digests/texhax/
+Keyword-In-Context indexes are filed in /tex-archive/digests/indexes/
+
+A Hypermail version of TeXhax is also available on the World-Wide Web at URL
+http://www.tex.ac.uk/tex-archive/digests/hyper/
+
+\bye
+
diff --git a/info/digests/texhax/96/texhax.06 b/info/digests/texhax/96/texhax.06
new file mode 100644
index 0000000000..6c0d93aed6
--- /dev/null
+++ b/info/digests/texhax/96/texhax.06
@@ -0,0 +1,432 @@
+From owner-texhax-l@IRLEARN.UCD.IE Wed May 29 14:47:00 1996
+Received: from listserv.rl.ac.uk (listserv.rl.ac.uk [130.246.132.23]) by granby.ccc.nottingham.ac.uk (8.6.12/8.6.12) with ESMTP id OAA09947 for <cczdao@UNICORN.CCC.NOTTINGHAM.AC.UK>; Wed, 29 May 1996 14:46:52 +0100
+Received: from listserv (listserv.rl.ac.uk [130.246.132.23]) by listserv.rl.ac.uk (8.7.1/8.7.1)
+ with SMTP id OAA15022; Wed, 29 May 1996 14:46:50 +0100 (BST)
+Received: from IRLEARN.UCD.IE by IRLEARN.UCD.IE (LISTSERV release 1.8b) with
+ NJE id 9757 for TEXHAX-L@IRLEARN.UCD.IE; Wed, 29 May 1996 13:44:20
+ +0000
+Received: from IRLEARN (NJE origin SMTP@IRLEARN) by IRLEARN.UCD.IE (LMail
+ V1.2a/1.8a) with BSMTP id 7220; Wed, 29 May 1996 13:44:19 +0000
+Received: from jess.ccc.nottingham.ac.uk by IRLEARN.UCD.IE (IBM VM SMTP V2R2)
+ with TCP; Wed, 29 May 96 13:44:15 GMT
+Received: from nottingham.ac.uk by jess.ccc.nottingham.ac.uk id
+ <22537-0@jess.ccc.nottingham.ac.uk>; Wed, 29 May 1996 13:24:21 +0100
+Errors-To: owner-texhax-digest@nottingham.ac.uk
+Precedence: bulk
+Message-ID: <TEXHAX-L%96052913442000@IRLEARN.UCD.IE>
+Date: Wed, 29 May 1996 13:24:21 +0100
+Reply-To: TeXhax@tex.ac.uk
+Sender: TeX Information Distribution List <TEXHAX-L@IRLEARN.UCD.IE>
+From: Majordomo list server <owner-texhax-digest@nottingham.ac.uk>
+Subject: TeXhax Digest V96 #6
+Comments: To: texhax-digest@nottingham.ac.uk
+To: Multiple recipients of list TEXHAX-L <TEXHAX-L@IRLEARN.UCD.IE>
+
+TeXhax Digest Wednesday, 29 May 1996 Volume 96 : Number 006
+
+(incorporating UKTeX Digest)
+
+Today's Topics:
+
+ "Standard" Font sizes
+ Re: TeXhax Digest V96 #5
+ TUGboat 17(1) Contents
+ Creating Adobe pdf files from LaTeX
+ excalibur
+ PostScript from other programs
+ [ANNOUNCE] TeX Live CDROM
+
+----------------------------------------------------------------------
+
+From: mclem@medphys.ucl.ac.uk (Matthew Clemence)
+Date: Mon, 15 Apr 96 15:52:03 BST
+Subject: "Standard" Font sizes
+
+I am maintaining a TeX system here at ucl, and like many others I suspect
+am allowing the system to generate all the fonts as needed (via MakeTeXpk).
+All of these end up jumbled together in the texmf/fonts/tmp/pk directory
+(under either cx or ljfour). I would quite like to shift the common ones
+into the appropriate sub directory (texmf/fonts/public/cm/ ..) so that
+tmp can be deleted on a regular basis. Can someone tell me the standard/most
+common sizes required ?
+
+Thanks.
+- --
+**************************************************************************
+Dr. Matthew Clemence ___ email mclem@medphys.ucl.ac.uk
+University College London
+11-20 Shropshire House,
+London, England
++44 171 387 9300 x 8448/8264
++44 181 442 1832 Home
+**************************************************************************
+
+------------------------------
+
+From: David Carlisle <carlisle@cs.man.ac.uk>
+Date: Wed, 17 Apr 96 17:57:14 BST
+Subject: Re: TeXhax Digest V96 #5
+
+Perhaps the following is a clearer example:
+
+\chardef\xxx`\a
+
+\ifcat a\xxx
+\message{yes}
+\else
+\message{no}
+\fi
+
+\let\xxx=a
+
+\ifcat a\xxx
+\message{yes}
+\else
+\message{no}
+\fi
+
+\bye
+
+A \chardef token is not a character token, and so does not have a
+catcode so as far as \ifcat is concerned it will compare true against
+any other control sequence, but false against any character.
+In your case \next is \let to ! ie it has the definition of
+\futurelet \next \dopling
+because
+!%
+!
+is equivalent to
+
+!!
+
+and \return is a chardef control sequence, so
+
+\ifcat\return\noexpand\next
+
+finds two control sequences and so returns true.
+
+As my example above shows you can get an implicit character token
+for use in \ifcat by using \let (the \bgroup you used is another such
+example) but you can not do this for <return> as it is not possible to
+make any token of catcode 5. input characters of that category always
+produce tokens of catcode 10 (and sometimes also \par tokens)
+You can not `see' the end of line by any method at all if you use the
+normal system, if you make ^^M active then of course you can see a
+ctacode 13 character token, but you disable the automatic system that
+TeX uses for converting end of line to white space, so you need
+arrange that the definition of ^^M does this `by hand'.
+
+David
+
+
+------------------------------
+
+From: Mimi Burbank <mimi@scri.fsu.edu>
+Date: Tue, 23 Apr 1996 11:16:12 -0500 (EDT)
+Subject: TUGboat 17(1) Contents
+
+ TUGboat
+ Volume 17, Number 1 / March 1996
+ ================================
+
+Addresses 3
+Soliciting Bids for TUG'97 4
+
+General Delivery
+ Michel Goossens
+ From the president 5
+ Barbara Beeton
+ Editorial comments 6
+ DEK on tour; TUB: the year ahead 6
+ TUG'95: Questions and answers with Prof. Donald E. Knuth 7
+
+Software & Tools
+ Frank G. Bennett, Jr.
+ Camel: kicking over the bibliographic traces in BibTeX 22
+ Filip Machi, Jerrold E. Marsden and Wendy G. McKay
+ Corrigendum: Introduction to FasTeX: a system of
+ keyboard shortcuts for the fast keying of TeX
+ (Volume 16(4), pp. 358-363) 28
+
+Fonts
+ Donald E. Knuth
+ Important message regarding CM fonts 29
+ Darko Zubrinic
+ Croatian fonts 29
+
+Book Reviews
+ J. Vesely Two new books on TeX in the Czech Republic: 34
+ Petr Olsak, Typograficky system TeX (TeX typesetting system);
+ Jiri Rybicka, LaTeX pro zacatecniky (LaTeX for beginners)
+ Lynne A. Price:
+ Ronald C. Turner, Timothy A. Douglass, and Audrey J. Turner,
+ README.1ST: SGML for Writers and Editors 35
+
+Letters
+ Rama Porrat
+ There's still something missing... 37
+
+Resources
+ Mimi Burbank and Michel Goossens
+ Electronic news from the family 37
+
+Tutorials
+ Keith Reckdahl
+ Using EPS graphics in LaTeXe documents 43
+
+Macros
+ Don Hosek
+ That ol' devil \expandafter 53
+ J. Hagen and A. F. Otten
+ PPCHTeX: typesetting chemical formulas in TeX 54
+
+LaTeX
+ David Carlisle
+ A LaTeX tour, Part 1: The basic distribution 67
+
+Abstracts
+ Les Cahiers GUTenberg, No. 20 73
+
+News & Announcements
+ Calendar 74
+ TUG'96 Preliminary schedule 76
+
+Late-Breaking News
+ Mimi Burbank
+ Production notes 78
+ Future issues 78
+
+TUG Business
+ TUG Bylaws 79
+ 1996 TUG election cancelled 84
+ TUG Board Members 84
+ Institutional members 85
+
+Forms
+ TUG membership application 86
+
+Advertisements
+ TeX consulting and production services 87
+ Index of advertisers 87
+
+
+------------------------------
+
+From: "S. Warde" <sw25@leicester.ac.uk>
+Date: Fri, 26 Apr 1996 12:55:34 +0100 (BST)
+Subject: Creating Adobe pdf files from LaTeX
+
+Looking for help in creating pdf files.
+
+1/
+ We are currently moving from LaTeX 209 to LaTeX2e. We have
+ found the hyper and hyperref packages in the contrib/supported
+ directory.
+ Though both authors acknowledge each other and say it would be
+ a good idea to merge they don't say when. Until they do I would
+ be grateful for peoples experiences with this packages.
+
+2/
+ We have obtained the dvihps source from ftp.tex.ac.uk this seems
+ to be quiet old. Before installing it I was wondering if
+ a - there was a more recent version
+ b - anyone had 'ported' it to work with the kpathsea distribution
+ c - knew of any limitations
+ d - knew of other dvips variants for creating pdf files
+ We run LaTeX etc. on SGI's and HP's
+
+3/
+ We have come across a reference to an archive and mailing list
+ on hypertex hosted at snorri.chem.washington.edu . This machine
+ does not seem to exist. Anyone know if the archive and maillist
+ are hosted elsewhere?
+
+ Thank you
+
+ Hugo Korwaser
+ Femsys Limited
+ Tel: +44 (0) 116 2541475
+ sw25@leiceter.ac.uk
+
+------------------------------
+
+From: sens@sbphy.ucsb.edu (Pierre Sens)
+Date: Wed, 1 May 1996 11:00:31 +0100
+Subject: excalibur
+
+Hi Tex users,
+
+I would like to use the spell checker Excalibur for documents in French on
+my Macintosh. I heard about a french dictionnary for Excalibur. Does
+anybody know where I could download it.
+
+Thanks,
+
+
+ Pierre Sens
+
+Department of Physics
+UCSB
+Santa Barbara, Ca 93106
+U.S.A
+Voice 1 (805) 893-8986
+Fax 1 (805) 893-2902
+email sens@physics.ucsb.edu
+
+
+
+------------------------------
+
+From: Russel Winder <R.Winder@cs.ucl.ac.uk>
+Date: Tue, 07 May 1996 02:50:36 +0100
+Subject: PostScript from other programs
+
+I regularly need to include PostScript figures in documents. I find
+idraw on Unix produces excellent relocatable PostScript descriptions
+but it is a little simplistic. There are excellent command language
+mechanisms for creating PostScript tricks using LaTeX but I really
+want to make use of (essentially) WYSIWYG programs like idraw,
+FrameMaker and Word to create images for inclusion in LaTeX2e
+documents. Furthermore I often have the requirement to use other
+people's PostScript in my LaTeX documents and have no control over the
+programs used to create the PostScript.
+
+In the past I have been able to use PostScript produced by various
+Macintosh programs (though judicious editing out of font information
+is usually required) and also PostScript produced by the PostScript
+driver under Windows3.1.
+
+At the pther extreme, FrameMaker has always been a pain in the !@#$.
+The PostScript produced by this program is totally un-relocatable. It
+assumes total control of the page and I have yet to discover the
+necessary edits to remove it's dictatorial efforts and hence make the
+PostScript relocatable.
+
+Moreover Microsoft appear to have gone down this same road and this is
+my real problem. Word7 with the Apple LaaserWriter plus driver (under
+Windows95) now produces code that it not relocatable. Indeed it has
+this penchant for trying to enquire of the amount of virtual memory on
+a regular basis which makes the PostScript file of a document
+un-previewable with ghostscript. I have found the trick for removing
+this VM quesry problem but have not found the trick for removing the
+manipulations that bind the coordinate system to the page in it's
+dictatorial manner.
+
+Is there a pool of experience on (or even tools to help forcing Word7
+and FrameMaker PostScript into a relocatable and hence usable form?
+
+Thanks for any guidance and/or pointers.
+
+Russel.
+
+=======================================================================
+
+Dr Russel Winder
+
+ Reader in Software Engineering
+ Editor-in-Chief, Object Oriented Systems
+
+Information Systems Research Group
+Department of Computer Science Phone: +44 (0)171 380 7293
+University College London Fax: +44 (0)171 387 1397
+Gower Street EMail: R.Winder@cs.ucl.ac.uk
+London WC1E 6BT
+UK URL: http://www.cs.ucl.ac.uk/staff/R.Winder/
+
+=======================================================================
+
+
+------------------------------
+
+From: Sebastian Rahtz <s.rahtz@elsevier.co.uk>
+Date: Wed, 29 May 1996 13:19:48 +0100
+Subject: [ANNOUNCE] TeX Live CDROM
+
+I am very glad to announce the launch today in Paris of TeX Live, a
+new CD published by the TeX Users Group, the UK TeX Users Group and
+GUIenberg (French TeX Users), with help from NTG (Dutch TeX Users) and
+many individuals from other groups.
+
+TeX Live's 649 megabytes contains:
+
+ - a ready to run Unix TeX setup, Thomas Esser's teTeX (based on Karl
+ Berry's Web2c). It has binaries for:
+ Linux on Intel and m68k platforms;
+ Irix 5.2, 5.3 on MIPS (SGI Indy/Indigo)
+ SunOS 4.1.3 on Sun
+ Solaris 2.3, 2.4, 2.5 on SPARC
+ HPUX 9.01, 10.01 for HP workstations
+ Digital Unix (OSF/1) 2.0 and 3.2 for DEC Alpha machines
+ FreeBSD and NetBSD on Intel platforms
+ Ultrix 4.3, for DEC Decstation machines
+ AIX 3.2, 4.1.1, for IBM RS6000 machines
+ NeXTStep on Intel platforms
+ - a very large support tree of macros, fonts and documentation
+ arranged according to the TeX Directory Structure layout
+ - the GUTenberg Mac, DOS and Windows distributions (archived)
+
+You can use the TeX system by running directly from the CD, installing
+on your hard disk, or by adding packages to your existing system.
+
+This is not a dump of CTAN full of compressed archives. This is a
+*working* system. To make it useable under Unix, it uses the Rock
+Ridge extensions to the ISO9660 file system. Ordinary systems can
+still read it, but will not see the long file names or symbolic links.
+
+If you are on any flavour of Unix, BUY this CD! There is no
+complicated compilation or moving of installed files around, it will
+just *WORK*.
+
+More details, and ordering information, can be found at
+http://www.tug.org/texlive.html. If you dont have WWW access, mail me
+or tug@tug.org for details. Cost is around $20 for members of any TeX
+users group, or $40 for others (the prices vary, depending on
+postage). All profits from sales go back to fund new versions of the
+CD, and to TeX-related development projects.
+
+Sebastian Rahtz
+Secretary, TeX Users Group
+
+------------------------------
+
+End of TeXhax Digest V96 #6
+***************************
+
+
+About TeXhax...
+
+Please send contributions to: TeXhax@tex.ac.uk
+
+Subscription and unsubscription requests:
+ send a one line mail message to TeXhax-Request@tex.ac.uk
+ containing either subscribe texhax
+ or unsubscribe texhax
+If you have problems with un/subscribing, please mail owner-texhax@nott.ac.uk
+
+To obtain the Frequently Asked Questions (FAQ) lists for TeX, send a
+message with no subject to fileserv@shsu.edu, consisting of
+SENDME FAQ
+
+For information on the TeX Users Group, please send a message to
+TUG@TUG.org, or write TeX Users Group, 1850 Union Street, #1637
+San Francisco CA 94123 (phone: 1 415 982 8449, fax: 1 415 982 8559)
+
+Backnumbers of all the digests are stored in the Comprehensive TeX
+Archive Network (CTAN) and can be retrieved on the Internet by
+anonymous ftp. The hosts comprising CTAN include, among others,
+ ftp.dante.de (129.69.1.12) -- Germany
+ ftp.shsu.edu (192.92.115.10) -- USA
+ ftp.tex.ac.uk (128.232.1.87) -- UK
+Please use your nearest server, to keep network load down.
+The file /tex-archive/CTAN.sites on each of these hosts gives a
+list of other sites which maintain full or partial mirrors of the CTAN.
+Alternatively, finger ctan_us@ftp.shsu.edu for full details.
+
+TeXhax Digest back issues are filed below /tex-archive/digests/texhax/
+Keyword-In-Context indexes are filed in /tex-archive/digests/indexes/
+
+A Hypermail version of TeXhax is also available on the World-Wide Web at URL
+http://www.tex.ac.uk/tex-archive/digests/hyper/
+
+\bye
+
diff --git a/info/digests/texhax/96/texhax.07 b/info/digests/texhax/96/texhax.07
new file mode 100644
index 0000000000..8631c3a2e0
--- /dev/null
+++ b/info/digests/texhax/96/texhax.07
@@ -0,0 +1,433 @@
+From texhax-digest-outgoing-request@nottingham.ac.uk Wed May 29 16:20:12 1996
+Received: from jess.ccc.nottingham.ac.uk (jess.ccc.nottingham.ac.uk [128.243.40.193]) by granby.ccc.nottingham.ac.uk (8.6.12/8.6.12) with ESMTP id QAA05215 for <cczdao@unix.ccc.nottingham.ac.uk>; Wed, 29 May 1996 16:20:10 +0100
+Message-Id: <199605291520.QAA05215@granby.ccc.nottingham.ac.uk>
+Received: from nottingham.ac.uk by jess.ccc.nottingham.ac.uk
+ id <25657-0@jess.ccc.nottingham.ac.uk>;
+ Wed, 29 May 1996 16:14:30 +0100
+From: Majordomo list server <owner-texhax-digest@nottingham.ac.uk>
+To: texhax-digest@nottingham.ac.uk
+Subject: TeXhax Digest V96 #7
+Reply-To: TeXhax@tex.ac.uk
+Errors-To: owner-texhax-digest@nottingham.ac.uk
+Precedence: bulk
+Date: Wed, 29 May 1996 16:14:30 +0100
+Sender: owner-texhax-digest@nottingham.ac.uk
+
+
+TeXhax Digest Wednesday, 29 May 1996 Volume 96 : Number 007
+
+(incorporating UKTeX Digest)
+
+Today's Topics:
+
+ Re: TeXhax Digest V96 #6
+ Re: TeXhax Digest V96 #6
+ Redefining maximum number of strings, fonts, etc...
+ margins
+ BibTeX/LaTeX2e question
+ BibTeX/LaTeX2e point -- PS
+ Location of index package (by David Jones)???
+ \def with catcode 1 in parameter text?
+ Re: TeXhax Digest V96 #6
+ Re: TeXhax Digest V96 #6
+
+----------------------------------------------------------------------
+
+From: Sebastian Rahtz <s.rahtz@elsevier.co.uk>
+Date: Wed, 29 May 1996 14:06:40 +0100
+Subject: Re: TeXhax Digest V96 #6
+
+ > From: "S. Warde" <
+ > Date: Fri, 26 Apr 1996 12:55:34 +0100 (BST)
+ > Subject: Creating Adobe pdf files from LaTeX
+ > We are currently moving from LaTeX 209 to LaTeX2e. We have
+ > found the hyper and hyperref packages in the contrib/supported
+ > directory.
+ > Though both authors acknowledge each other and say it would be
+ > a good idea to merge they don't say when. Until they do I would
+tell us if its necessary, and I expect we'll do something about it.
+
+ > We have obtained the dvihps source from ftp.tex.ac.uk this seems
+ > to be quiet old. Before installing it I was wondering if
+ > a - there was a more recent version
+the author is working on it
+
+ > b - anyone had 'ported' it to work with the kpathsea
+ > distribution
+yes, its moderately trivial to do yourself. but i'll be in the next
+release of dvipsk
+
+ > c - knew of any limitations
+yes, you cant vary the height of the anchor box...
+
+ > d - knew of other dvips variants for creating pdf files
+use the nativepdf option of hyperref, and you dont need dvihps at all
+
+sebastian rahtz
+
+------------------------------
+
+From: Sebastian Rahtz <s.rahtz@elsevier.co.uk>
+Date: Wed, 29 May 1996 14:09:02 +0100
+Subject: Re: TeXhax Digest V96 #6
+
+ > From: mclem@medphys.ucl.ac.uk (Matthew Clemence)
+ > From: Russel Winder <R.Winder@cs.ucl.ac.uk>
+ > Date: Tue, 07 May 1996 02:50:36 +0100
+ > Subject: PostScript from other programs
+ > Is there a pool of experience on (or even tools to help forcing Word7
+ > and FrameMaker PostScript into a relocatable and hence usable form?
+
+my generic solution is Acrobat Distiller. its an excellent (obviously)
+PostScript interpreter. Use that to make a PDF (ie clean!) version of
+the file, then use Acrobat Exchange to write out a new .ps version.
+
+i havent tried this per se with Frame files, but the principal should
+be sound.
+
+sebastian
+
+------------------------------
+
+From: mse@sdr.utias.utoronto.ca (Manfred D. M. Sever)
+Date: Mon, 13 May 1996 13:51:46 -0400
+Subject: Redefining maximum number of strings, fonts, etc...
+
+Hello All!
+
+I would like to be able to increase the
+maximum number ``strings'' available to
+TeX for processing documents. It may also
+be usefull to increase the upper limit
+on the number of fonts allowed.
+
+I think this should be possible to do since
+Goossens, Mittelbach and Samarin discuss
+this on page xi of their book. I believe
+that increasing these limits also involves
+recompiling TeX.
+
+However, I can't find the file that these
+limits are set in.
+
+How do I proceed?
+
+Thanks in advance,
+
+Manfred.
+
+
+------------------------------
+
+From: Mr Marcilo Alves <marcilio@liverpool.ac.uk>
+Date: Thu, 16 May 1996 10:11:57 +0100 (BST)
+Subject: margins
+
+Hi,
+
+That's Marcilio from Liverpool.
+
+I am using book class in Latex2e with the option
+[a4paper,twoside].
+
+Yet, my output shows different margins; that is there
+is a horizontal and vertical shift of one side of the output in
+relation to the other side.
+
+Is this a problem with the book class or with
+the printer.
+
+The printer is in the computer centre and I do not
+know how to eventually set margins, etc...
+
+
+Could you help me?
+
+
+- - ---------------------------------
+
+
+I am using fancyheadings to set the head of the page.
+It works fine but it seems that the commands
+
+\listoffigures
+\listoftables
+\tableofcontents
+
+uses its own formatting for the heads.
+
+How can I set my own head in conjunction
+with the above commands?
+
+
+- - ---------------------------------
+
+
+I do appreciate your help in advance!
+
+Thank you
+
+marcilio
+
+
+------------------------------
+
+From: Russel Winder <R.Winder@cs.ucl.ac.uk>
+Date: Thu, 16 May 1996 19:31:37 +0100
+Subject: BibTeX/LaTeX2e question
+
+I am wondering whether there is a solution to this or whether it is "just one
+of those things you have to hack around".
+
+The background: Using the Harvard family of citation styles with BibTeX and
+LaTeX2e. Two references are different papers in the same tome.
+Cross-referencing is used to avoid duplication of information. Both
+references are used in the same document. The tome itself (item C) has no
+author or editor.
+
+@InCollection{A,
+ ...
+ crossref = "C"
+}
+@InCollection{B,
+ ...
+ crossref = "C"
+}
+@Book{C,
+ ...
+}
+
+The problem: Because the tome is cited in two difference references, three
+entries get written to the bbl and auxiliary files. Because the tome has no
+author or editor, the title is used as the citation mark (both short and long
+form) -- and here is the problem -- this is emphasized, i.e. there is a \emph
+in the parameters of the citation entries in the .aux file. When the aux
+file
+is read in, the error:
+
+! Incomplete \iffalse; all text was ignored after line 72.
+<inserted text>
+ \fi
+
+occurs.
+
+There are three hack solutions:
+
+1. Manual edit the .aux file -- totally unacceptable.
+2. Do not use crossref but suffer the replication of information --
+unacceptable but...
+3. Force BibTeX not to generate the third reference item but to replicate
+crossreference information in all entries -- I haven't investigated this and
+therefore do not know how to do it.
+
+Have others seen this and found other solutions?
+
+Russel.
+
+=======================================================================
+
+Dr Russel Winder
+
+ Reader in Software Engineering
+ Editor-in-Chief, Object Oriented Systems
+
+Information Systems Research Group
+Department of Computer Science Phone: +44 (0)171 380 7293
+University College London Fax: +44 (0)171 387 1397
+Gower Street EMail: R.Winder@cs.ucl.ac.uk
+London WC1E 6BT
+UK URL: http://www.cs.ucl.ac.uk/staff/R.Winder/
+
+=======================================================================
+
+
+------------------------------
+
+From: Russel Winder <R.Winder@cs.ucl.ac.uk>
+Date: Thu, 16 May 1996 19:48:19 +0100
+Subject: BibTeX/LaTeX2e point -- PS
+
+I forgot to note the obvious point about using the key field in the
+bibliography item, I suppose that should be solution 0. My point was:
+Has anyone got an answer to the problem of \emph appearing in the .aux
+file, even if accidently?
+
+------------------------------
+
+From: coleman@trillium.phys.uregina.ca (Robert Coleman)
+Date: Fri, 17 May 1996 15:28:01 -0600 (CST)
+Subject: Location of index package (by David Jones)???
+
+On page 367 of the LaTeX Companion, the index
+package written by David Jones is discussed.
+I have, however, not been able to find it on
+CTAN. From where can this package be ftped?
+Thanks for the help. A direct reply would be
+appreciated.
+
+Robert
+
+- --
+Robert Alan Coleman
+Department of Physics
+University of Regina
+Regina, Saskatchewan
+Canada S4S 0A2
+
+Tel: (306) 585-4260
+Fax: (306) 585-4894
+email: coleman@cas.uregina.ca
+
+
+------------------------------
+
+From: Matteo Frigo <athena@glauke.lcs.mit.edu>
+Date: Mon, 27 May 1996 11:41:45 -0400
+Subject: \def with catcode 1 in parameter text?
+
+Abstract: I want to put a token with catcode 1 in the parameter text
+of a macro, and I don't know how.
+
+Problem: I wrote a macro \largeenough which takes one argument, and
+formats it in the largest possible font so that the argument fits
+in one page (I use it for slides). The macro works by trial and
+error, and some other hackery.
+
+Now, I want to define a LaTeX environment
+
+\begin{largeenough}
+ text
+\end{largeenough}
+
+to accomplish the same effect --- since the environment seems more
+elegant to me. The problem is: how do I capture all text preceding the
+\end{largeenough} ? I need it for the trial-and-error procedure. So
+far, I use a simple-minded loop to accomplish this effect, but I
+wonder if there is a better way.
+
+Essentially, I would like to define a macro
+
+\def\foo#1\end{largeenough}{...}
+
+where I have a { *with catcode 1* in the parameter text. Changing the
+catcode doesn't work, because I want to keep the original catcode
+of { in the normal text (inside the environment).
+
+Any idea?
+
+Thanks,
+Matteo
+
+
+------------------------------
+
+From: Michael Doob <mdoob@cc.UManitoba.CA>
+Date: Wed, 29 May 1996 08:39:09 -0500 (CDT)
+Subject: Re: TeXhax Digest V96 #6
+
+> From: mclem@medphys.ucl.ac.uk (Matthew Clemence)
+> Date: Mon, 15 Apr 96 15:52:03 BST
+> Subject: "Standard" Font sizes
+>
+> I am maintaining a TeX system here at ucl, and like many others I suspect
+> am allowing the system to generate all the fonts as needed (via MakeTeXpk).
+> All of these end up jumbled together in the texmf/fonts/tmp/pk directory
+> (under either cx or ljfour). I would quite like to shift the common ones
+> into the appropriate sub directory (texmf/fonts/public/cm/ ..) so that
+> tmp can be deleted on a regular basis. Can someone tell me the
+> standard/most common sizes required ?
+
+You could run a cron job that deletes pk files that haven't been accessed
+recently. What's left over will be (by definition) the popular ones, and they
+could be moved out to a more permanent location periodically.
+
+
+Cheers,
+Michael
+
+
+------------------------------
+
+From: David Carlisle <carlisle@ma.man.ac.uk>
+Date: Wed, 29 May 1996 15:32:12 +0100
+Subject: Re: TeXhax Digest V96 #6
+
+> Date: Fri, 26 Apr 1996 12:55:34 +0100 (BST)
+You'd get a quicker information turn around by asking on comp.text.tex
+newsgroup (or the equivalent info-tex mailing list).
+
+> b - anyone had 'ported' it [dvihps] to work with the kpathsea
+> distribution
+
+I'm not sure, but if you use the [nativepdf] option to the hyperref
+package you can use a standard dvips(k). The pdfmark operators are
+written straight to the PS file via literal postscript specials.
+
+> on hypertex hosted at snorri.chem.washington.edu . This machine
+> does not seem to exist
+the mailing list is now reachable via ....
+
+This mailing list will ensure that you are notified of new releases
+and other major developments to HyperTeX, the hypertext extensions
+to TeX. The list address is:
+
+ hypertex-announce@aps.org
+
+This list is now being managed by majordomo, so subscription requests
+and other information can be obtained by e-mail to
+
+ majordomo@aps.org
+
+Archives of the announcements and an ftp area for the software
+should be announced shortly.
+
+For other information, check out the Web page at Los Alamos:
+
+ http://xxx.lanl.gov/hypertex/
+
+Please let the list owner (apsmith@aps.org) know of any problems.
+
+
+------------------------------
+
+End of TeXhax Digest V96 #7
+***************************
+
+
+About TeXhax...
+
+Please send contributions to: TeXhax@tex.ac.uk
+
+Subscription and unsubscription requests:
+ send a one line mail message to TeXhax-Request@tex.ac.uk
+ containing either subscribe texhax
+ or unsubscribe texhax
+If you have problems with un/subscribing, please mail owner-texhax@nott.ac.uk
+
+To obtain the Frequently Asked Questions (FAQ) lists for TeX, send a
+message with no subject to fileserv@shsu.edu, consisting of
+SENDME FAQ
+
+For information on the TeX Users Group, please send a message to
+TUG@TUG.org, or write TeX Users Group, 1850 Union Street, #1637
+San Francisco CA 94123 (phone: 1 415 982 8449, fax: 1 415 982 8559)
+
+Backnumbers of all the digests are stored in the Comprehensive TeX
+Archive Network (CTAN) and can be retrieved on the Internet by
+anonymous ftp. The hosts comprising CTAN include, among others,
+ ftp.dante.de (129.69.1.12) -- Germany
+ ftp.shsu.edu (192.92.115.10) -- USA
+ ftp.tex.ac.uk (128.232.1.87) -- UK
+Please use your nearest server, to keep network load down.
+The file /tex-archive/CTAN.sites on each of these hosts gives a
+list of other sites which maintain full or partial mirrors of the CTAN.
+Alternatively, finger ctan_us@ftp.shsu.edu for full details.
+
+TeXhax Digest back issues are filed below /tex-archive/digests/texhax/
+Keyword-In-Context indexes are filed in /tex-archive/digests/indexes/
+
+A Hypermail version of TeXhax is also available on the World-Wide Web at URL
+http://www.tex.ac.uk/tex-archive/digests/hyper/
+
+\bye
+
diff --git a/info/digests/texhax/96/texhax.08 b/info/digests/texhax/96/texhax.08
new file mode 100644
index 0000000000..986a3421aa
--- /dev/null
+++ b/info/digests/texhax/96/texhax.08
@@ -0,0 +1,372 @@
+From texhax-digest-outgoing-request@nottingham.ac.uk Tue Jun 18 10:21:36 1996
+Received: from jess.ccc.nottingham.ac.uk (jess.ccc.nottingham.ac.uk [128.243.40.193]) by granby.ccc.nottingham.ac.uk (8.6.12/8.6.12) with ESMTP id KAA23924 for <cczdao@unix.ccc.nottingham.ac.uk>; Tue, 18 Jun 1996 10:21:32 +0100
+Message-Id: <199606180921.KAA23924@granby.ccc.nottingham.ac.uk>
+Received: from nottingham.ac.uk by jess.ccc.nottingham.ac.uk
+ id <14408-0@jess.ccc.nottingham.ac.uk>;
+ Tue, 18 Jun 1996 10:17:25 +0100
+From: Majordomo list server <owner-texhax-digest@nottingham.ac.uk>
+To: texhax-digest@nottingham.ac.uk
+Subject: TeXhax Digest V96 #8
+Reply-To: TeXhax@tex.ac.uk
+Errors-To: owner-texhax-digest@nottingham.ac.uk
+Precedence: bulk
+Date: Tue, 18 Jun 1996 10:17:25 +0100
+Sender: owner-texhax-digest@nottingham.ac.uk
+
+
+TeXhax Digest Tuesday, 18 June 1996 Volume 96 : Number 008
+
+(incorporating UKTeX Digest)
+
+Today's Topics:
+
+ Re: TeXhax Digest V96 #7
+ Re: TeXhax Digest V96 #7
+ Re: TeXhax Digest V96 #7
+ Re: Location of David Jones' index package (TeXhax V96 #7)
+ Re: excalibur
+ TeXhax Digest V96 #6
+ Writing long formulas
+ Fonts in Latex2.09
+ [none]
+ EMTeX no longer available from ftp.shsu.edu
+
+----------------------------------------------------------------------
+
+From: Sebastian Rahtz <s.rahtz@elsevier.co.uk>
+Date: Thu, 30 May 1996 08:22:44 +0100
+Subject: Re: TeXhax Digest V96 #7
+
+ > I would like to be able to increase the
+ > maximum number ``strings'' available to
+ > TeX for processing documents. It may also
+ > be usefull to increase the upper limit
+ > on the number of fonts allowed.
+yes, you can do it. but its not trivial. current releases of TeX for
+DOS (emtex) and Mac (OzTeX) allow you to do it dynamically), while
+Textures and Y&Y TeX claim to just give you unlimited memory (I only say
+claim because i havent tried it myself). If you have Unix TeX, lets
+assume you have the web2c stuff. in that case, find the directory
+web2c/tex and the file ctex.ch. thats the "change file", and near the
+top you find a set of constants which are set to complicated
+values. thats where you play, and thats as far as I am going to give a
+recipe! you have to understand how change files work (read the WEB
+documentation) and what the values do. NOTE that some values are
+already at their upper limit
+
+sebastian rahtz
+
+
+------------------------------
+
+From: David Carlisle <carlisle@ma.man.ac.uk>
+Date: Wed, 29 May 1996 17:03:05 +0100
+Subject: Re: TeXhax Digest V96 #7
+
+> Abstract: I want to put a token with catcode 1 in the parameter text
+> of a macro, and I don't know how.
+
+You can't.
+
+> Essentially, I would like to define a macro
+
+> \def\foo#1\end{largeenough}{...}
+
+you have to go
+
+\def\foo#1\end#2{...
+and then check that #2 is `largeenough' and if not search for the next
+\end. It's a bit of a pain, but that's life. There are examples of this
+in the ams alignment environments, and tabularx and load of other
+places.
+
+David
+
+
+
+------------------------------
+
+From: Matteo Frigo <athena@glauke.lcs.mit.edu>
+Date: Wed, 29 May 1996 12:07:58 -0400
+Subject: Re: TeXhax Digest V96 #7
+
+> \def\foo#1\end#2{...
+> and then check that #2 is `largeenough' and if not search for the next
+> \end. It's a bit of a pain, but that's life. There are examples of this
+> in the ams alignment environments, and tabularx and load of other
+> places.
+>
+> David
+>
+
+Thanks for your answer. In fact, this is what I am doing now. I am
+glad to hear that there is no better way to do the same thing.
+
+Cheers
+Matteo
+
+
+------------------------------
+
+From: Robin Fairbairns <Robin.Fairbairns@cl.cam.ac.uk>
+Date: Wed, 29 May 1996 17:32:15 +0100
+Subject: Re: Location of David Jones' index package (TeXhax V96 #7)
+
+Robert Coleman <coleman@trillium.phys.uregina.ca> writes:
+>On page 367 of the LaTeX Companion, the index
+>package written by David Jones is discussed.
+>I have, however, not been able to find it on
+>CTAN. [...]
+
+I maintain a file (on CTAN, of course) info/companion.ctan; this aims
+to list all packages mentioned in the Companion, and to give their
+location on CTAN.
+
+In this instance, it says
+
+ index
+ macros/latex209/contrib/misc/index.doc
+
+As you will note, this is a 2.09 package. A 2e one is in development,
+and I have a copy of a beta-version. Sadly, I failed to note which
+MIT machine I got it from, and it's pretty old. Perhaps David Jones
+himself will respond... (Or even submit a new version for the
+archive?)
+- --
+Robin (Campaign for Real Radio 3) Fairbairns rf@cl.cam.ac.uk
+U of Cambridge Computer Lab, Pembroke St, Cambridge CB2 3QG, UK
+Home page: http://www.cl.cam.ac.uk/users/rf/robin.html
+
+
+------------------------------
+
+From: Alun.J.Carr@ucd.ie (Dr Alun J. Carr)
+Date: Thu, 30 May 1996 10:55:33 +0100
+Subject: Re: excalibur
+
+On Wed, 1 May 1996 11:00:31 +0100 sens@sbphy.ucsb.edu (Pierre Sens) wrote:
+>Hi Tex users,
+>
+>I would like to use the spell checker Excalibur for documents in French on
+>my Macintosh. I heard about a french dictionnary for Excalibur. Does
+>anybody know where I could download it.
+
+<ftp://ftp.eg.bucknell.edu/pub/mac/Excalibur-dictionaries/Dico-Francais-sea.hq
+x>
+
+Alun
+
+
+- --
+Dr Alun J. CARR Phone: +353-1-7061989
+Mechanical Engineering Dept. +353-1-2693244 x1989
+University College Dublin Fax: +353-1-2830534
+Belfield E-mail: <Alun.J.Carr@ucd.ie>
+Dublin 4 WWW: <http://tizit.ucd.ie/ajcarr/>
+Ireland
+
+
+------------------------------
+
+From: bkph@ai.mit.edu (Berthold K.P. Horn)
+Date: Fri, 31 May 1996 08:34:23 -0400
+Subject: TeXhax Digest V96 #6
+
+> From: Russel Winder <R.Winder@cs.ucl.ac.uk>
+> Date: Tue, 07 May 1996 02:50:36 +0100
+> Subject: PostScript from other programs
+>
+> I regularly need to include PostScript figures in documents. I find
+> idraw on Unix produces excellent relocatable PostScript descriptions
+> but it is a little simplistic. There are excellent command language
+> mechanisms for creating PostScript tricks using LaTeX but I really
+> want to make use of (essentially) WYSIWYG programs like idraw,
+> FrameMaker and Word to create images for inclusion in LaTeX2e
+> documents. Furthermore I often have the requirement to use other
+> people's PostScript in my LaTeX documents and have no control over the
+> programs used to create the PostScript.
+>
+> In the past I have been able to use PostScript produced by various
+> Macintosh programs (though judicious editing out of font information
+> is usually required) and also PostScript produced by the PostScript
+> driver under Windows3.1.
+>
+> At the pther extreme, FrameMaker has always been a pain in the !@#$.
+> The PostScript produced by this program is totally un-relocatable. It
+> assumes total control of the page and I have yet to discover the
+> necessary edits to remove it's dictatorial efforts and hence make the
+> PostScript relocatable.
+>
+> Moreover Microsoft appear to have gone down this same road and this is
+> my real problem. Word7 with the Apple LaaserWriter plus driver (under
+> Windows95) now produces code that it not relocatable. Indeed it has
+> this penchant for trying to enquire of the amount of virtual memory on
+> a regular basis which makes the PostScript file of a document
+> un-previewable with ghostscript. I have found the trick for removing
+> this VM quesry problem but have not found the trick for removing the
+> manipulations that bind the coordinate system to the page in it's
+> dictatorial manner.
+>
+> Is there a pool of experience on (or even tools to help forcing Word7
+> and FrameMaker PostScript into a relocatable and hence usable form?
+>
+> Thanks for any guidance and/or pointers.
+>
+> Russel.
+ >
+> =======================================================================
+>
+> Dr Russel Winder
+>
+> Reader in Software Engineering
+> Editor-in-Chief, Object Oriented Systems
+>
+> Information Systems Research Group
+> Department of Computer Science Phone: +44 (0)171 380 7293
+> University College London Fax: +44 (0)171 387 1397
+> Gower Street EMail: R.Winder@cs.ucl.ac.uk
+> London WC1E 6BT
+> UK URL: http://www.cs.ucl.ac.uk/staff/R.Winder/
+>
+> =======================================================================
+
+You must make sure the applications export in EPS format.
+
+If you use raw PS, you are asking for trouble, because
+PS drivers are allowed to use arbitrary PS code including the
+forbidden-in-EPS operators that are troubling you.
+
+Most printer drivers have a check box this produce EPS output.
+If you can't use EPS because the output is multi-page then you
+are not using EPS, since that by definition is one page only.
+
+Berthold.
+
+
+------------------------------
+
+From: Dominique de Waleffe <ddw@goedel.miscrit.be>
+Date: Wed, 5 Jun 96 15:42:51 +0200
+Subject: Writing long formulas
+
+In Digital SRC report 119, Leslie Lamport presents a nice way to
+format long mathematical formulas so that reading is a lot easier.
+
+Does anyone know a Latex package which implements those conventions?
+
+I'd be very interested.
+
+D.
+- --
+Dominique de Waleffe
+Mission Critical, Wijnegemhofstraat 199, B-3071 Erps-Kwerps (Belgium)
+Phone: +32 2 759 95 60 Fax: +32 2 759 27 60
+email: ddw@acm.org, ddw@miscrit.be
+PGP key fingerprint: F9 CC 23 74 44 62 7C F3 8C 12 DF 71 BB 60 54 98
+
+
+
+------------------------------
+
+From: "Klaus M. Wendel" <a9304890@unet.univie.ac.at>
+Date: Tue, 11 Jun 1996 11:56:13 +0200 (MSZ)
+Subject: Fonts in Latex2.09
+
+Help!
+
+I am using LaTex Version 2.09 and i have to print a document in HELVETICA.
+
+Is there a (easy) way to do this?
+
+Thanks!
+
+- ---------------------------------------------------
+Klaus M. Wendel always :-)
+E-Mail: a9304890@unet.univie.ac.at
+WWW: http://www.wu-wien.ac.at/usr/h93/h9304890/
+- ---------------------------------------------------
+
+
+------------------------------
+
+From: David Simpson <mifads@bure.ntnu.no>
+Date: Mon, 17 Jun 1996 09:53:46 +0100
+Subject: [none]
+
+Does anybody know how to convert LaTeX(2e) files
+to, dare I say, Word or Wordperfect documents?
+Apologies to all tex fans for asking this, but
+unfortunately I can't persuade all my PC-based colleagues
+to switch to LateX, but I need to send them text
+for reports.
+
+Thanks, Dave Simpson (david.simpson@dnmi.no)
+
+
+------------------------------
+
+From: Rainer Schoepf <schoepf@uni-mainz.de>
+Date: Tue, 18 Jun 1996 11:14:13 +0200 (MET DST)
+Subject: EMTeX no longer available from ftp.shsu.edu
+
+As ftp.shsu.edu is not properly managed, and the emTeX files there are
+partially out of date, I removed emTeX at teh request of Eberhard
+Mattes from ftp.shsu.edu.
+
+We are apologize for the inconvenience, but the CTAN team doesn't have
+the resources to manage a site from across the atlantic.
+
+For the CTAN team
+
+- --
+ Rainer Schvpf
+ Zentrum f|r Datenverarbeitung A point of view can be a dangerous
+ der Universitdt Mainz luxury when substituted for insight
+ Anselm-Franz-von-Bentzel-Weg 12 and understanding.
+ D-55099 Mainz
+ Germany Herbert Marshall McLuhan:
+ <Schoepf@Uni-Mainz.DE> The Gutenberg Galaxy
+
+------------------------------
+
+End of TeXhax Digest V96 #8
+***************************
+
+
+About TeXhax...
+
+Please send contributions to: TeXhax@tex.ac.uk
+
+Subscription and unsubscription requests:
+ send a one line mail message to TeXhax-Request@tex.ac.uk
+ containing either subscribe texhax
+ or unsubscribe texhax
+If you have problems with un/subscribing, please mail owner-texhax@nott.ac.uk
+
+To obtain the Frequently Asked Questions (FAQ) lists for TeX, send a
+message with no subject to fileserv@shsu.edu, consisting of
+SENDME FAQ
+
+For information on the TeX Users Group, please send a message to
+TUG@TUG.org, or write TeX Users Group, 1850 Union Street, #1637
+San Francisco CA 94123 (phone: 1 415 982 8449, fax: 1 415 982 8559)
+
+Backnumbers of all the digests are stored in the Comprehensive TeX
+Archive Network (CTAN) and can be retrieved on the Internet by
+anonymous ftp. The hosts comprising CTAN include, among others,
+ ftp.dante.de (129.69.1.12) -- Germany
+ ftp.shsu.edu (192.92.115.10) -- USA
+ ftp.tex.ac.uk (128.232.1.87) -- UK
+Please use your nearest server, to keep network load down.
+The file /tex-archive/CTAN.sites on each of these hosts gives a
+list of other sites which maintain full or partial mirrors of the CTAN.
+Alternatively, finger ctan_us@ftp.shsu.edu for full details.
+
+TeXhax Digest back issues are filed below /tex-archive/digests/texhax/
+Keyword-In-Context indexes are filed in /tex-archive/digests/indexes/
+
+A Hypermail version of TeXhax is also available on the World-Wide Web at URL
+http://www.tex.ac.uk/tex-archive/digests/hyper/
+
+\bye
+
diff --git a/info/digests/texhax/96/texhax.09 b/info/digests/texhax/96/texhax.09
new file mode 100644
index 0000000000..cbc9e90728
--- /dev/null
+++ b/info/digests/texhax/96/texhax.09
@@ -0,0 +1,493 @@
+From texhax-digest-outgoing-request@nottingham.ac.uk Tue Jun 18 18:12:06 1996
+Received: from jess.ccc.nottingham.ac.uk (jess.ccc.nottingham.ac.uk [128.243.40.193]) by granby.ccc.nottingham.ac.uk (8.6.12/8.6.12) with ESMTP id SAA20764 for <cczdao@unix.ccc.nottingham.ac.uk>; Tue, 18 Jun 1996 18:12:05 +0100
+Message-Id: <199606181712.SAA20764@granby.ccc.nottingham.ac.uk>
+Received: from nottingham.ac.uk by jess.ccc.nottingham.ac.uk
+ id <00363-0@jess.ccc.nottingham.ac.uk>;
+ Tue, 18 Jun 1996 18:09:36 +0100
+From: Majordomo list server <owner-texhax-digest@nottingham.ac.uk>
+To: texhax-digest@nottingham.ac.uk
+Subject: TeXhax Digest V96 #9
+Reply-To: TeXhax@tex.ac.uk
+Errors-To: owner-texhax-digest@nottingham.ac.uk
+Precedence: bulk
+Date: Tue, 18 Jun 1996 18:09:36 +0100
+Sender: owner-texhax-digest@nottingham.ac.uk
+
+
+TeXhax Digest Tuesday, 18 June 1996 Volume 96 : Number 009
+
+(incorporating UKTeX Digest)
+
+Today's Topics:
+
+ pagebreaks, &c in indexes (resubmitted)
+ Re: TeXhax Digest V96 #8
+
+----------------------------------------------------------------------
+
+From: David Handscomb <David.Handscomb@comlab.ox.ac.uk>
+Date: Tue, 18 Jun 96 10:45:08 BST
+Subject: pagebreaks, &c in indexes (resubmitted)
+
+As nobody responded when I first put this question, may I have another go?
+Surely there is a `makeindex' guru somewhere out there?
+
+- ----- Begin Included Message -----
+
+From dch Wed Oct 4 17:41:49 1995
+To: TeXhax@tex.ac.uk
+Subject: pagebreaks, &c in indexes
+Content-Length: 869
+X-Lines: 41
+
+
+a) Is there any way of ensuring that an index generated by makeindex
+is not broken between an item and its following subitem? - e.g.:
+
+network 12
+news
+========= pagebreak ============
+ bad 13
+ good 14
+next 15
+
+I guess (see Companion p.359) that it could possibly be done via the
+keyword item_x1 in the *.ist file, if at all. If so, then how?
+\nopagebreak in the *.ind file seems to have no effect.
+
+
+
+b) Furthermore, is there any way of generating the following? My guess
+is `no':
+
+news
+ bad 13
+ good 14
+ \emph{see also} gnus
+
+
+
+David Handscomb
+
+Numerical Analysis Group
+Oxford University Computing Laboratory
+Wolfson Building
+Parks Road
+Oxford OX1 3QD
+ENGLAND
+
+tel. (national) 01865 273891 (international) +44 1865 273891
+FAX (national) 01865 273839 (international) +44 1865 273839
+E-mail dch@comlab.ox.ac.uk / na.handscomb@na-net.ornl.gov
+
+
+
+
+- ----- End Included Message -----
+
+
+------------------------------
+
+From: "e-Floyd B. Hanson, UIC 312-413-2142" <U12688@UICVM.UIC.EDU>
+Date: Tue, 18 Jun 1996 12:08:18 CDT
+Subject: Re: TeXhax Digest V96 #8
+
+> TeXhax Digest Tuesday, 18 June 1996 Volume 96 : Number 008
+>
+> (incorporating UKTeX Digest)
+>
+> Today's Topics:
+>
+> Re: TeXhax Digest V96 #7
+> Re: TeXhax Digest V96 #7
+> Re: TeXhax Digest V96 #7
+> Re: Location of David Jones' index package (TeXhax V96 #7)
+> Re: excalibur
+> TeXhax Digest V96 #6
+> Writing long formulas
+> Fonts in Latex2.09
+> [none]
+> EMTeX no longer available from ftp.shsu.edu
+>
+> ----------------------------------------------------------------------
+>
+> From: Sebastian Rahtz <s.rahtz@elsevier.co.uk>
+> Date: Thu, 30 May 1996 08:22:44 +0100
+> Subject: Re: TeXhax Digest V96 #7
+>
+> > I would like to be able to increase the
+> > maximum number ``strings'' available to
+> > TeX for processing documents. It may also
+> > be usefull to increase the upper limit
+> > on the number of fonts allowed.
+> yes, you can do it. but its not trivial. current releases of TeX for
+> DOS (emtex) and Mac (OzTeX) allow you to do it dynamically), while
+> Textures and Y&Y TeX claim to just give you unlimited memory (I only say
+> claim because i havent tried it myself). If you have Unix TeX, lets
+> assume you have the web2c stuff. in that case, find the directory
+> web2c/tex and the file ctex.ch. thats the "change file", and near the
+> top you find a set of constants which are set to complicated
+> values. thats where you play, and thats as far as I am going to give a
+> recipe! you have to understand how change files work (read the WEB
+> documentation) and what the values do. NOTE that some values are
+> already at their upper limit
+>
+> sebastian rahtz
+>
+>
+> ------------------------------
+>
+> From: David Carlisle <carlisle@ma.man.ac.uk>
+> Date: Wed, 29 May 1996 17:03:05 +0100
+> Subject: Re: TeXhax Digest V96 #7
+>
+>> Abstract: I want to put a token with catcode 1 in the parameter text
+>> of a macro, and I don't know how.
+>
+> You can't.
+>
+>> Essentially, I would like to define a macro
+>
+>> \def\foo#1\end{largeenough}{...}
+>
+> you have to go
+>
+> \def\foo#1\end#2{...
+> and then check that #2 is `largeenough' and if not search for the next
+> \end. It's a bit of a pain, but that's life. There are examples of this
+> in the ams alignment environments, and tabularx and load of other
+> places.
+>
+> David
+>
+>
+>
+> ------------------------------
+>
+> From: Matteo Frigo <athena@glauke.lcs.mit.edu>
+> Date: Wed, 29 May 1996 12:07:58 -0400
+> Subject: Re: TeXhax Digest V96 #7
+>
+>> \def\foo#1\end#2{...
+>> and then check that #2 is `largeenough' and if not search for the next
+>> \end. It's a bit of a pain, but that's life. There are examples of this
+>> in the ams alignment environments, and tabularx and load of other
+>> places.
+>>
+>> David
+>>
+>
+> Thanks for your answer. In fact, this is what I am doing now. I am
+> glad to hear that there is no better way to do the same thing.
+>
+> Cheers
+> Matteo
+>
+>
+> ------------------------------
+>
+> From: Robin Fairbairns <Robin.Fairbairns@cl.cam.ac.uk>
+> Date: Wed, 29 May 1996 17:32:15 +0100
+> Subject: Re: Location of David Jones' index package (TeXhax V96 #7)
+>
+> Robert Coleman <coleman@trillium.phys.uregina.ca> writes:
+>>On page 367 of the LaTeX Companion, the index
+>>package written by David Jones is discussed.
+>>I have, however, not been able to find it on
+>>CTAN. [...]
+>
+> I maintain a file (on CTAN, of course) info/companion.ctan; this aims
+> to list all packages mentioned in the Companion, and to give their
+> location on CTAN.
+>
+> In this instance, it says
+>
+> index
+> macros/latex209/contrib/misc/index.doc
+>
+> As you will note, this is a 2.09 package. A 2e one is in development,
+> and I have a copy of a beta-version. Sadly, I failed to note which
+> MIT machine I got it from, and it's pretty old. Perhaps David Jones
+> himself will respond... (Or even submit a new version for the
+> archive?)
+> - --
+> Robin (Campaign for Real Radio 3) Fairbairns rf@cl.cam.ac.uk
+> U of Cambridge Computer Lab, Pembroke St, Cambridge CB2 3QG, UK
+> Home page: http://www.cl.cam.ac.uk/users/rf/robin.html
+>
+>
+> ------------------------------
+>
+> From: Alun.J.Carr@ucd.ie (Dr Alun J. Carr)
+> Date: Thu, 30 May 1996 10:55:33 +0100
+> Subject: Re: excalibur
+>
+> On Wed, 1 May 1996 11:00:31 +0100 sens@sbphy.ucsb.edu (Pierre Sens) wrote:
+>>Hi Tex users,
+>>
+>>I would like to use the spell checker Excalibur for documents in French on
+>>my Macintosh. I heard about a french dictionnary for Excalibur. Does
+>>anybody know where I could download it.
+>
+> <ftp://ftp.eg.bucknell.edu/pub/mac/Excalibur-dictionaries/Dico-Francais-sea.hq
+> x>
+>
+> Alun
+>
+>
+> - --
+> Dr Alun J. CARR Phone: +353-1-7061989
+> Mechanical Engineering Dept. +353-1-2693244 x1989
+> University College Dublin Fax: +353-1-2830534
+> Belfield E-mail: <Alun.J.Carr@ucd.ie>
+> Dublin 4 WWW: <http://tizit.ucd.ie/ajcarr/>
+> Ireland
+>
+>
+> ------------------------------
+>
+> From: bkph@ai.mit.edu (Berthold K.P. Horn)
+> Date: Fri, 31 May 1996 08:34:23 -0400
+> Subject: TeXhax Digest V96 #6
+>
+>> From: Russel Winder <R.Winder@cs.ucl.ac.uk>
+>> Date: Tue, 07 May 1996 02:50:36 +0100
+>> Subject: PostScript from other programs
+>>
+>> I regularly need to include PostScript figures in documents. I find
+>> idraw on Unix produces excellent relocatable PostScript descriptions
+>> but it is a little simplistic. There are excellent command language
+>> mechanisms for creating PostScript tricks using LaTeX but I really
+>> want to make use of (essentially) WYSIWYG programs like idraw,
+>> FrameMaker and Word to create images for inclusion in LaTeX2e
+>> documents. Furthermore I often have the requirement to use other
+>> people's PostScript in my LaTeX documents and have no control over the
+>> programs used to create the PostScript.
+>>
+>> In the past I have been able to use PostScript produced by various
+>> Macintosh programs (though judicious editing out of font information
+>> is usually required) and also PostScript produced by the PostScript
+>> driver under Windows3.1.
+>>
+>> At the pther extreme, FrameMaker has always been a pain in the !@#$.
+>> The PostScript produced by this program is totally un-relocatable. It
+>> assumes total control of the page and I have yet to discover the
+>> necessary edits to remove it's dictatorial efforts and hence make the
+>> PostScript relocatable.
+>>
+>> Moreover Microsoft appear to have gone down this same road and this is
+>> my real problem. Word7 with the Apple LaaserWriter plus driver (under
+>> Windows95) now produces code that it not relocatable. Indeed it has
+>> this penchant for trying to enquire of the amount of virtual memory on
+>> a regular basis which makes the PostScript file of a document
+>> un-previewable with ghostscript. I have found the trick for removing
+>> this VM quesry problem but have not found the trick for removing the
+>> manipulations that bind the coordinate system to the page in it's
+>> dictatorial manner.
+>>
+>> Is there a pool of experience on (or even tools to help forcing Word7
+>> and FrameMaker PostScript into a relocatable and hence usable form?
+>>
+>> Thanks for any guidance and/or pointers.
+>>
+>> Russel.
+> >
+>> =======================================================================
+>>
+>> Dr Russel Winder
+>>
+>> Reader in Software Engineering
+>> Editor-in-Chief, Object Oriented Systems
+>>
+>> Information Systems Research Group
+>> Department of Computer Science Phone: +44 (0)171 380 7293
+>> University College London Fax: +44 (0)171 387 1397
+>> Gower Street EMail: R.Winder@cs.ucl.ac.uk
+>> London WC1E 6BT
+>> UK URL: http://www.cs.ucl.ac.uk/staff/R.Winder/
+>>
+>> =======================================================================
+>
+> You must make sure the applications export in EPS format.
+>
+> If you use raw PS, you are asking for trouble, because
+> PS drivers are allowed to use arbitrary PS code including the
+> forbidden-in-EPS operators that are troubling you.
+>
+> Most printer drivers have a check box this produce EPS output.
+> If you can't use EPS because the output is multi-page then you
+> are not using EPS, since that by definition is one page only.
+>
+> Berthold.
+>
+>
+> ------------------------------
+>
+> From: Dominique de Waleffe <ddw@goedel.miscrit.be>
+> Date: Wed, 5 Jun 96 15:42:51 +0200
+> Subject: Writing long formulas
+>
+> In Digital SRC report 119, Leslie Lamport presents a nice way to
+> format long mathematical formulas so that reading is a lot easier.
+>
+> Does anyone know a Latex package which implements those conventions?
+>
+> I'd be very interested.
+>
+> D.
+> - --
+> Dominique de Waleffe
+> Mission Critical, Wijnegemhofstraat 199, B-3071 Erps-Kwerps (Belgium)
+> Phone: +32 2 759 95 60 Fax: +32 2 759 27 60
+> email: ddw@acm.org, ddw@miscrit.be
+> PGP key fingerprint: F9 CC 23 74 44 62 7C F3 8C 12 DF 71 BB 60 54 98
+>
+>
+>
+> ------------------------------
+>
+> From: "Klaus M. Wendel" <a9304890@unet.univie.ac.at>
+> Date: Tue, 11 Jun 1996 11:56:13 +0200 (MSZ)
+> Subject: Fonts in Latex2.09
+>
+> Help!
+>
+> I am using LaTex Version 2.09 and i have to print a document in HELVETICA.
+>
+> Is there a (easy) way to do this?
+>
+> Thanks!
+>
+> - ---------------------------------------------------
+> Klaus M. Wendel always :-)
+> E-Mail: a9304890@unet.univie.ac.at
+> WWW: http://www.wu-wien.ac.at/usr/h93/h9304890/
+> - ---------------------------------------------------
+>
+>
+> ------------------------------
+>
+> From: David Simpson <mifads@bure.ntnu.no>
+> Date: Mon, 17 Jun 1996 09:53:46 +0100
+> Subject: [none]
+>
+> Does anybody know how to convert LaTeX(2e) files
+> to, dare I say, Word or Wordperfect documents?
+> Apologies to all tex fans for asking this, but
+> unfortunately I can't persuade all my PC-based colleagues
+> to switch to LateX, but I need to send them text
+> for reports.
+>
+> Thanks, Dave Simpson (david.simpson@dnmi.no)
+>
+>
+> ------------------------------
+>
+> From: Rainer Schoepf <schoepf@uni-mainz.de>
+> Date: Tue, 18 Jun 1996 11:14:13 +0200 (MET DST)
+> Subject: EMTeX no longer available from ftp.shsu.edu
+>
+> As ftp.shsu.edu is not properly managed, and the emTeX files there are
+> partially out of date, I removed emTeX at teh request of Eberhard
+> Mattes from ftp.shsu.edu.
+>
+> We are apologize for the inconvenience, but the CTAN team doesn't have
+> the resources to manage a site from across the atlantic.
+>
+> For the CTAN team
+>
+> - --
+> Rainer Schvpf
+> Zentrum f|r Datenverarbeitung A point of view can be a dangerous
+> der Universitdt Mainz luxury when substituted for insight
+> Anselm-Franz-von-Bentzel-Weg 12 and understanding.
+> D-55099 Mainz
+> Germany Herbert Marshall McLuhan:
+> <Schoepf@Uni-Mainz.DE> The Gutenberg Galaxy
+>
+> ------------------------------
+>
+> End of TeXhax Digest V96 #8
+> ***************************
+>
+>
+> About TeXhax...
+>
+> Please send contributions to: TeXhax@tex.ac.uk
+>
+> Subscription and unsubscription requests:
+> send a one line mail message to TeXhax-Request@tex.ac.uk
+> containing either subscribe texhax
+> or unsubscribe texhax
+> If you have problems with un/subscribing, please mail owner-texhax@nott.ac.uk
+>
+> To obtain the Frequently Asked Questions (FAQ) lists for TeX, send a
+> message with no subject to fileserv@shsu.edu, consisting of
+> SENDME FAQ
+>
+> For information on the TeX Users Group, please send a message to
+> TUG@TUG.org, or write TeX Users Group, 1850 Union Street, #1637
+> San Francisco CA 94123 (phone: 1 415 982 8449, fax: 1 415 982 8559)
+>
+> Backnumbers of all the digests are stored in the Comprehensive TeX
+> Archive Network (CTAN) and can be retrieved on the Internet by
+> anonymous ftp. The hosts comprising CTAN include, among others,
+> ftp.dante.de (129.69.1.12) -- Germany
+> ftp.shsu.edu (192.92.115.10) -- USA
+> ftp.tex.ac.uk (128.232.1.87) -- UK
+> Please use your nearest server, to keep network load down.
+> The file /tex-archive/CTAN.sites on each of these hosts gives a
+> list of other sites which maintain full or partial mirrors of the CTAN.
+> Alternatively, finger ctan_us@ftp.shsu.edu for full details.
+>
+> TeXhax Digest back issues are filed below /tex-archive/digests/texhax/
+> Keyword-In-Context indexes are filed in /tex-archive/digests/indexes/
+>
+> A Hypermail version of TeXhax is also available on the World-Wide Web at URL
+> http://www.tex.ac.uk/tex-archive/digests/hyper/
+>
+> \bye
+
+------------------------------
+
+End of TeXhax Digest V96 #9
+***************************
+
+
+About TeXhax...
+
+Please send contributions to: TeXhax@tex.ac.uk
+
+Subscription and unsubscription requests:
+ send a one line mail message to TeXhax-Request@tex.ac.uk
+ containing either subscribe texhax
+ or unsubscribe texhax
+If you have problems with un/subscribing, please mail owner-texhax@nott.ac.uk
+
+To obtain the Frequently Asked Questions (FAQ) lists for TeX, send a
+message with no subject to fileserv@shsu.edu, consisting of
+SENDME FAQ
+
+For information on the TeX Users Group, please send a message to
+TUG@TUG.org, or write TeX Users Group, 1850 Union Street, #1637
+San Francisco CA 94123 (phone: 1 415 982 8449, fax: 1 415 982 8559)
+
+Backnumbers of all the digests are stored in the Comprehensive TeX
+Archive Network (CTAN) and can be retrieved on the Internet by
+anonymous ftp. The hosts comprising CTAN include, among others,
+ ftp.dante.de (129.69.1.12) -- Germany
+ ftp.shsu.edu (192.92.115.10) -- USA
+ ftp.tex.ac.uk (128.232.1.87) -- UK
+Please use your nearest server, to keep network load down.
+The file /tex-archive/CTAN.sites on each of these hosts gives a
+list of other sites which maintain full or partial mirrors of the CTAN.
+Alternatively, finger ctan_us@ftp.shsu.edu for full details.
+
+TeXhax Digest back issues are filed below /tex-archive/digests/texhax/
+Keyword-In-Context indexes are filed in /tex-archive/digests/indexes/
+
+A Hypermail version of TeXhax is also available on the World-Wide Web at URL
+http://www.tex.ac.uk/tex-archive/digests/hyper/
+
+\bye
+
diff --git a/info/digests/texhax/96/texhax.10 b/info/digests/texhax/96/texhax.10
new file mode 100644
index 0000000000..49f492d9ad
--- /dev/null
+++ b/info/digests/texhax/96/texhax.10
@@ -0,0 +1,366 @@
+From: owner-texhax-digest@nottingham.ac.UK (Majordomo list server)
+Newsgroups: ucam.mlist.texhax
+Subject: TeXhax Digest V96 #10
+Message-ID: <E0ueW35-0001y9-00@heaton.cl.cam.ac.ukD>
+Date: 12 Jul 96 01:14:05 GMT
+Sender: owner-texhax-digest@nottingham.ac.uk
+Reply-To: TeXhax@tex.ac.uk
+Lines: 355
+
+
+TeXhax Digest Friday, 12 July 1996 Volume 96 : Number 010
+
+(incorporating UKTeX Digest)
+
+Today's Topics:
+
+ Re: TeXhax Digest V96 #8 -- Helvetica Fonts in Latex2.09
+ wysiwyg
+ Announcement NTG TeX & Graphics course
+ Re: TeXhax postings to the TEX-L list
+ New address
+ METAFONT for Macintosh?
+ TUGBoat 17(3)
+ Chancellor Group (symbol = CHAG)
+
+----------------------------------------------------------------------
+
+From: "e-Floyd B. Hanson, UIC 312-413-2142" <U12688@UICVM.UIC.EDU>
+Date: Tue, 18 Jun 1996 12:09:45 CDT
+Subject: Re: TeXhax Digest V96 #8 -- Helvetica Fonts in Latex2.09
+
+If you have psfonts, then a simple way to use helvetica is to use the
+style file helv.sty:
+- ---cut---
+% latex style with helvetica text.
+\def\@mrm{phvr}% Helvetica - roman
+\def\@mit{phvro}% Helvetica -oblique
+\def\@msl{phvbo}% Helvetica - bold - oblique
+\def\@mbf{phvb}% Helvetica - bold
+\def\@mcsc{phvrc}% Helvetica - small caps
+\def\@mtt{pcrr}% courier
+\def\@mss{phvr}% helvetica
+\input psfonts.sty
+- ---cut---
+> ------------------------------
+.
+> From: "Klaus M. Wendel" <a9304890@unet.univie.ac.at>
+> Date: Tue, 11 Jun 1996 11:56:13 +0200 (MSZ)
+> Subject: Fonts in Latex2.09
+>
+> Help!
+>
+> I am using LaTex Version 2.09 and i have to print a document in HELVETICA.
+>
+> Is there a (easy) way to do this?
+>
+> Thanks!
+>
+> - ---------------------------------------------------
+
+------------------------------
+
+From: Hartmut Peters <peters@sunhp.msrc.sunysb.edu>
+Date: Tue, 18 Jun 1996 18:34:47 -0400 (EDT)
+Subject: wysiwyg
+
+Dear fellow TeXers,
+
+I am having a paper in the mill at Academic Press in the UK, and they only
+do wysiwyg programs like word, yuk! My manuscript is in LaTeX both on Unix
+and on a Mac under Textures. Does anybody know of a way to export the text
+to wysiwyg programs? It wouldn't be a problem if the formatting
+information got lost as long as the text came across. (Excalibur has a
+parser that throws out the LaTeX/TeX commands...)
+
+The opposite way apparently works, but I haven't seen TeX --> wysiwyg,
+yet. Well, if not, the Academic Press folks may have to digest the LaTeX
+files or retype it (in which case I get to find all the typos...) I sure
+wish they could handle something more professional than "Word"...
+
+Best regards from Lawn Guyland in the strange State of New York -->
+::::::::::::::::::::::::::::::::::::::::::::::::::::::::::::::::::::::::::::::::
+ HARTMUT PETERS | E-MAIL: hartmut.peters@sunysb.edu
+ (Assistant Professor) | OR: peters@sunhp.msrc.sunysb.edu
+ | BITNET: hpeters@sbccmail.bitnet
+ Marine Sciences Research Center | All-In-One/Stony Brook: hpeters
+ State University of New York at ... |
+ Stony Brook, NY 11794-5000 | TELEPHONE: +516 632-8682
+ U.S.A. | TELEFAX: +516 632-8820
+::::::::::::::::::::::::::::::::::::::::::::::::::::::::::::::::::::::::::::::::
+
+
+
+------------------------------
+
+From: cgl@rc.service.rug.nl (Kees van der Laan)
+Date: Mon, 24 Jun 1996 20:34:55 +0200
+Subject: Announcement NTG TeX & Graphics course
+
+ ----------------------------------------------------------
+ |Announcement NTG's one-day low-budget no-nonsense course|
+ | |
+ | (La)TeX and Graphics |
+ ----------------------------------------------------------
+
+What? MetaPost, PostScript, mftoeps... (no font design)
+
+Language? English
+
+Teacher? Boguslaw Jackowksi
+
+Where? University of Utrecht
+
+When? The day before *or* after NTG's fall meeting of 24 Oct.
+ (To be announced late August)
+
+Costs? Fl 50 members of NTG or other LUGs; non-members Fl 150
+
+Subsription? Treasurer NTG, Giro 1306238, Eindhoven
+ (Do mention TeX and Graphics course)
+
+Information? Kees van der Laan, cgl@rc.service.rug.nl
+
+Literature? Jackowksi's METAFONT booklet will be in English available.
+ Have a look at
+ Hobby's CSTR 162, A user manual for MetaPost,
+ from
+ netlib@research.att.com
+ with message
+ send 162 from research/cstr
+ or copy it from NTG's 4AllTeX CD-ROM
+
+When the number of subscriptions is insufficient of
+
+ 7 September---ultimate date for registration---
+
+the course will be cancelled (and money refunded).
+
+- ---Kees---
+
+Sent to: TeXhax@tex.ac.uk (earlier Dutch announcement to tex-nl@nic.surfnet.nl)
+cc: TeX-D-L@vm.gmd.de, gut@ens.fr, uktex@tex.ac.uk, metafont@ens.fr, TUG@TUG.org, info-tex@shsu.edu
+
+------------------------------
+
+From: Melvin Fitting <MLFLC@CUNYVM.CUNY.EDU>
+Date: Fri, 05 Jul 96 09:53:55 EDT
+Subject: Re: TeXhax postings to the TEX-L list
+
+- ---------------------------------
+| New e-mail address |
+| fitting@alpha.lehman.cuny.edu |
+| Please use from now on |
+- ---------------------------------
+
+------------------------------
+
+From: Melvin Fitting <MLFLC@CUNYVM.CUNY.EDU>
+Date: Fri, 05 Jul 96 09:54:34 EDT
+Subject: New address
+
+- ---------------------------------
+| New e-mail address |
+| fitting@alpha.lehman.cuny.edu |
+| Please use from now on |
+- ---------------------------------
+
+------------------------------
+
+From: Tom Bryan <tbryan@Thorin>
+Date: Fri, 5 Jul 96 10:29:29 PDT
+Subject: METAFONT for Macintosh?
+
+Has anyone had success in running METAFONT on a Macintosh?
+
+I have a Macintosh PowerBook 5300c, system 7.5.
+I am using the public domain METAFONT version 0.66 from the
+Bluesky ftp site, and it is not fully operational.
+Is there a better METAFONT for Macintosh available?
+
+Once the MF file is written, how does one install a font on the
+Macintosh so that Bluesky's Textures can display the font on the
+screen and print?
+
+Has anyone had success getting "penlabels" to work?
+
+Does anyone know how to produce output with grid lines and
+penlabels like those in Donald Knuth's "Computer Modern
+Typefaces, Computers & Typesetting volume E"?
+
+For some reason, I have not received a TeXhax digest in a while
+even though I am still on the list, so please reply directly to
+me in addition to posting to the list.
+
+Sincerely,
+Tom Bryan
+
+E-Systems, Goleta Division
+One South Los Carneros, Goleta, CA 93117-3197, USA
+Email: tbryan@esd.ray.com Tel: (805) 967-5511 ext. 2974
+FAX: (805) 964-0470
+
+
+------------------------------
+
+From: Mimi Burbank <mimi@scri.fsu.edu>
+Date: Fri, 5 Jul 1996 16:14:04 -0500 (EDT)
+Subject: TUGBoat 17(3)
+
+Dear colleagues,
+
+Below are the contents of the Proceedings Issue of TUGboat for 1996.
+
+ TUGboat
+ Volume 17, Number 2 / June 1996
+ ================================
+
+Opening Address
+ Michel Goossens
+ Opening Words by the President 91
+
+%%%%%%%%%%%
+Fonts
+ Karel Piska
+ Cyrillic alphabets 92
+ J\"org Knappen
+ The dc fonts 1.3: Move towards stability and
+ completeness 99
+ Fukui Rei
+ TIPA: A system for processing phonetic symbols in
+ LaTeX
+ A.S. Berdnikov
+ Computer Modern Typefaces as Multiple Master Fonts 115
+ A.S. Berdnikov
+ VFComb 1.3 --- the program which simplifies
+ virtual font management 120
+
+%%%%%%%%%%%
+Encoding and Multilingual Support
+ Yannis Haralambous
+ \Omega Times and \Omega Helvetica fonts under
+ development: Step ~One 126
+ Richard J.~Kinch
+ Extending TeX for Unicode 147
+ L.N. Znamenskaya and S.V. Znamenskii
+ Russian encoding plurality problem
+ and a new Cyrillic font set 161
+ Peter~A.~Ovchenkov
+ Cyrillic TeX files: interplatform portability 166
+ Michael M. Vinogradov
+ A user-friendly multi-function TeX interface
+ based on Multi-Edit 172
+ Olga~G. Lapko
+ Full Cyrillic: How many languages? 174
+
+
+%%%%%%%%%%%
+TeX Systems
+ John Plaice and Yannis Haralambous
+ The latest developments in \Omega 181
+ Dag Langmyhr
+ StarTeX --- a TeX for beginners 184
+ Gabriel Valiente Feruglio
+ Do journals honor LaTeX submissions? 191
+ Sergei V. Znamenskii and Denis E. Leinartas
+ A new approach to the TeX-related programs:
+ A user-friendly interface 200
+ Ivan G. Vsesvetsky
+ The strait gate to TeX 204
+ Laurent Siebenmann
+ DVI-based electronic publication 206
+ Kees~van~der~Laan
+ BLUe's format --- the off-off alternative 215
+
+%%%%%%%%%%%
+Graphics
+ Kees~van~der~Laan
+ Turtle graphics and TeX -- a child can do it 222
+ A.S. Berdnikov, O.A. Grineva and S.B. Turtia
+ Some useful macros which extend the LaTeX
+ picture environment 229
+
+%%%%%%%%%%%
+News & Announcements
+ Mimi Burbank
+ Production notes 90
+ Calendar 233
+
+%%%%%%%%%%%
+TUG Business<
+ Institutional members 234
+
+%%%%%%%%%%%
+Advertisements
+ TeX consulting and production services 235
+====== end of file
+
+
+
+
+------------------------------
+
+From: chag@moneyworld.com
+Date: Fri, 12 Jul 1996 01:06:33 +0100
+Subject: Chancellor Group (symbol = CHAG)
+
+http://chancellor.stockpick.com
+
+Chancellor Group, Inc. (symbol CHAG) just reported big quarterly earnings.
+SGA Goldstar issued a "buy" recommendation. I understand other investment
+advisors are looking to recommend CHAG. The company has a strong book value.
+The short sellers need to cover. This looks like a good situation to me.
+What do you think? They are located at:
+
+ http://chancellor.stockpick.com
+
+Bob Williams, 206-269-0846
+
+To terminate from my Investment Opportunities, Reply to
+chag@moneyworld.com with "remove" in the subject field.
+
+------------------------------
+
+End of TeXhax Digest V96 #10
+****************************
+
+
+About TeXhax...
+
+Please send contributions to: TeXhax@tex.ac.uk
+
+Subscription and unsubscription requests:
+ send a one line mail message to TeXhax-Request@tex.ac.uk
+ containing either subscribe texhax
+ or unsubscribe texhax
+If you have problems with un/subscribing, please mail owner-texhax@nott.ac.uk
+
+To obtain the Frequently Asked Questions (FAQ) lists for TeX, send a
+message with no subject to fileserv@shsu.edu, consisting of
+SENDME FAQ
+
+For information on the TeX Users Group, please send a message to
+TUG@TUG.org, or write TeX Users Group, 1850 Union Street, #1637
+San Francisco CA 94123 (phone: 1 415 982 8449, fax: 1 415 982 8559)
+
+Backnumbers of all the digests are stored in the Comprehensive TeX
+Archive Network (CTAN) and can be retrieved on the Internet by
+anonymous ftp. The hosts comprising CTAN include, among others,
+ ftp.dante.de (129.69.1.12) -- Germany
+ ftp.shsu.edu (192.92.115.10) -- USA
+ ftp.tex.ac.uk (128.232.1.87) -- UK
+Please use your nearest server, to keep network load down.
+The file /tex-archive/CTAN.sites on each of these hosts gives a
+list of other sites which maintain full or partial mirrors of the CTAN.
+Alternatively, finger ctan_us@ftp.shsu.edu for full details.
+
+TeXhax Digest back issues are filed below /tex-archive/digests/texhax/
+Keyword-In-Context indexes are filed in /tex-archive/digests/indexes/
+
+A Hypermail version of TeXhax is also available on the World-Wide Web at URL
+http://www.tex.ac.uk/tex-archive/digests/hyper/
+
+\bye
+
+
+
diff --git a/info/digests/texhax/96/texhax.11 b/info/digests/texhax/96/texhax.11
new file mode 100644
index 0000000000..aab4ff8edf
--- /dev/null
+++ b/info/digests/texhax/96/texhax.11
@@ -0,0 +1,416 @@
+From: owner-texhax-digest@nottingham.ac.UK (Majordomo list server)
+Subject: TeXhax Digest V96 #11
+Message-ID: <E0ugfTd-0004lZ-00@heaton.cl.cam.ac.ukD>
+Date: 17 Jul 96 23:44:30 GMT
+
+
+TeXhax Digest Wednesday, 17 July 1996 Volume 96 : Number 011
+
+(incorporating UKTeX Digest)
+
+Today's Topics:
+
+ Re: METAFONT for Macintosh?
+ Re: TeXhax Digest V96 #10
+ Re: wysiwyg (TeXhax Digest V96 #10)
+ Re: TeXhax Digest V96 #10
+ Metafont on Mac
+ Re: Metafont on Mac
+
+----------------------------------------------------------------------
+
+From: phadke@oceaneng.eng.hawaii.edu (Amal Phadke)
+Date: Thu, 11 Jul 1996 16:23:54 -1000
+Subject: Re: METAFONT for Macintosh?
+
+> From: Tom Bryan <tbryan@Thorin>
+> Date: Fri, 5 Jul 96 10:29:29 PDT
+> Subject: METAFONT for Macintosh?
+.
+> Has anyone had success in running METAFONT on a Macintosh?
+>
+> I have a Macintosh PowerBook 5300c, system 7.5.
+> I am using the public domain METAFONT version 0.66 from the
+> Bluesky ftp site, and it is not fully operational.
+> Is there a better METAFONT for Macintosh available?
+>
+> Sincerely,
+> Tom Bryan
+>
+> E-Systems, Goleta Division
+> One South Los Carneros, Goleta, CA 93117-3197, USA
+> Email: tbryan@esd.ray.com Tel: (805) 967-5511 ext. 2974
+> FAX: (805) 964-0470
+>
+
+Tom:
+
+ OzTeX 2.0.1, Andrew Trevorrow's Macintosh shareware version of TeX
+now has its own METAFONT program OzMF. It has worked flawlessly for me so
+far. OzTeX is available at all CTAN sites under
+"/tex-archive/systems/mac/oztex" directory.
+
+Cheers
+
+- ---
+Amal Phadke
+Department of Ocean Engineering,
+School of Ocean and Earth Sciences & Technology
+University of Hawaii at Manoa
+2540 Dole street , Holmes Hall 408B
+Honolulu, HI 96822, USA
+Tel: (808) 956-8198 Fax: (808) 956-3498
+
+e-mail: phadke@oceaneng.eng.hawaii.edu
+WWW Page : http://oceaneng.eng.hawaii.edu/~phadke
+
+
+
+------------------------------
+
+From: Piet van Oostrum <piet@cs.ruu.nl>
+Date: Fri, 12 Jul 1996 11:56:52 +0200
+Subject: Re: TeXhax Digest V96 #10
+
+The following message is a courtesy copy of an article
+that has been posted as well.
+
+>>>>> Hartmut Peters <peters@sunhp.msrc.sunysb.edu> (HP) writes:
+
+HP> Dear fellow TeXers,
+HP> I am having a paper in the mill at Academic Press in the UK, and they only
+HP> do wysiwyg programs like word, yuk! My manuscript is in LaTeX both on Unix
+HP> and on a Mac under Textures. Does anybody know of a way to export the text
+HP> to wysiwyg programs? It wouldn't be a problem if the formatting
+HP> information got lost as long as the text came across. (Excalibur has a
+HP> parser that throws out the LaTeX/TeX commands...)
+
+HP> The opposite way apparently works, but I haven't seen TeX --> wysiwyg,
+HP> yet. Well, if not, the Academic Press folks may have to digest the LaTeX
+HP> files or retype it (in which case I get to find all the typos...) I sure
+HP> wish they could handle something more professional than "Word"...
+
+Try tex2rtf (not latex2rtf), from the support directory on CTAN. It
+produces RTF which can be read both by Wordpervert and Microsloth Word, and
+probably plenty of others, too.
+
+If that doesn't work you could try detex, but it may throw away too much.
+Or latex it, and push it through dvi2tty.
+- --
+Piet van Oostrum <piet@cs.ruu.nl>
+URL: http://www.cs.ruu.nl/~piet [PGP]
+
+------------------------------
+
+From: piet@cs.ruu.nl (Piet van Oostrum)
+Date: Fri, 12 Jul 1996 12:09:21 +0200
+Subject: Re: wysiwyg (TeXhax Digest V96 #10)
+
+The following message is a courtesy copy of an article
+that has been posted as well.
+
+>>>>> Hartmut Peters <peters@sunhp.msrc.sunysb.edu> (HP) writes:
+
+HP> Dear fellow TeXers,
+HP> I am having a paper in the mill at Academic Press in the UK, and they only
+HP> do wysiwyg programs like word, yuk! My manuscript is in LaTeX both on Unix
+HP> and on a Mac under Textures. Does anybody know of a way to export the text
+HP> to wysiwyg programs? It wouldn't be a problem if the formatting
+HP> information got lost as long as the text came across. (Excalibur has a
+HP> parser that throws out the LaTeX/TeX commands...)
+
+HP> The opposite way apparently works, but I haven't seen TeX --> wysiwyg,
+HP> yet. Well, if not, the Academic Press folks may have to digest the LaTeX
+HP> files or retype it (in which case I get to find all the typos...) I sure
+HP> wish they could handle something more professional than "Word"...
+
+Try tex2rtf (not latex2rtf), from the support directory on CTAN. It
+produces RTF which can be read both by Wordpervert and Microsloth Word, and
+probably plenty of others, too.
+
+If that doesn't work you could try detex, but it may throw away too much.
+Or latex it, and push it through dvi2tty.
+- --
+Piet van Oostrum <piet@cs.ruu.nl>
+URL: http://www.cs.ruu.nl/~piet [PGP]
+
+------------------------------
+
+From: John Burt <BURT@BINAH.CC.BRANDEIS.EDU>
+Date: Fri, 12 Jul 1996 11:18:02 -0500 (EST)
+Subject: Re: TeXhax Digest V96 #10
+
+Yes, I have had success running Metafont on a Mac. The one I use is
+the Metafont that is a part of the CMacTeX package. It makes the standard
+.tfm and .gf files, and you run gftopk on the .gf file to generate a .pk
+file. Running Metafont is not trivial, but this Metafont behaves
+exactly like the metafonts in the unix and vms worlds. It is shareware, but
+you get the whole CMacTeX package, which is great, for your money. You
+can download the most recent version from the author's home page. I
+don't have his URL handy, but I think it's
+http://math.tamu.edu/~tkiffe/cmactex.html
+The author's name is Tom Kiffe.
+
+One note: your TeX must use standard .pk files (CMacTeX and OzTeX do---in
+fact the latter uses Kiffe's Metafont as well). Some systems---I think
+Te
+textures among them---don't use standard .pk files.
+John Burt
+burt@binah.cc.brandeis.edu
+
+------------------------------
+
+From: campbell@beloit.edu (Paul J. Campbell)
+Date: Fri, 12 Jul 1996 13:19:18 -0600
+Subject: Metafont on Mac
+
+I too have been experimenting with Metafont 0.66 on the Mac (acutally, when
+it runs, it says it is 0.65).
+
+I'm glad you asked your question, so I can describe for my own
+benefit---before I forget!---what works.
+
+There is a file metafont-for-beginners on CTAN (I think it is in /doc), but
+I have not seen anything specific to the Mac.
+
+Last week I had my first success with Metafont. I was able to take a
+Metafont description of an .mf file and turn it into a font suitcase that
+works with any application.
+
+The key problem is that .mf files cannot be processed without some additions.
+
+I used the program shell
+
+mode=laserwriter; % Metafont checks mfinputs:local.mf for all mode_def's;
+ % put here the option that fits your printer
+mag=magstep0; % could be magstep1, or 0.5,2,3,4,5 as well
+%screenstrokes; % could be screenchars, for per character viewing,
+but works faster this way
+input hcyr % this is the .mf file to process; add more
+input lines as needed
+bye
+
+To the original .mf file I added the following introductory lines:
+
+font_size 10pt#; % the "design size" of this font; can be whatever you like
+ht#:=10pt#; % height of characters
+xgap#:=0.6pt#; % horizontal adjustment (I just guessed at this)
+
+If you don't put in lines like those, everything appears to work, Metafont
+says that it writes the fonts to files, and indeed it creates files---empty
+ones!
+
+You process the program shell file with Metafont with the settings Make
+Suitcase and (for me, by default) Plain, by selecting Metafont on the
+Metafont menu.
+
+Metafont adds the font to the MFfonts suitcase and creates an MFmetrics
+suitcase (you can see the latter and you can double-click the former to
+examine the contents---that's how I found out I wasn't getting anything).
+You need the latter for use with TeX but the former will work with any
+program (Word, etc.) if you put it into the System folder. The application
+program (e.g., Word) will scale the font (with jaggies) to any size; if you
+want nice looking characters, process the .mf file to make fonts at all the
+sizes you will use.
+
+I was just making a font of ancient Croatian script for a couple of
+students here, but part of my reason for exploring all this is an old
+desire (and a new need) to be able to take fonts for IBM (.pfb, .pfa, .pfm
+files) and make them into Mac bitmapped (or preferably PS) fonts. After
+exploring lots of software on CTAN last week, I am convinced that this can
+be done but (as far as I know) only by making a detour to using some Unix
+programs along the way.
+
+A promising new port of TeX, LaTeX, Metafont (0.66), BibTex, and
+Metapost(!) for the Mac was promulgated this April. Called cmactex25, it is
+available at CTAN (it's shareware, $35). CMacTeX handles afm, pfa, pfb, and
+tfm files directly and there is no need to convert them into a Mac-specific
+format. I have explored only its font utilities so far.
+
+
+
+ Paul J. Campbell
+ Mathematics and Computer Science
+ Beloit College
+ 700 College St.
+ Beloit, WI 53511
+ (608) 363-2007 (ofc)
+ 362-2805 (res)
+ 363-2718 (fax)
+ campbell@beloit.edu
+
+
+
+------------------------------
+
+From: Tom Bryan <tbryan@Thorin>
+Date: Wed, 17 Jul 96 15:41:45 PDT
+Subject: Re: Metafont on Mac
+
+Dear Paul,
+
+Thank you for your advice on using Metafont 0.66 on the Mac. I
+tried your advice and got the Blue Sky version to work for the
+first time. However, I was still dissatisfied with the
+deficiencies of the Blue Sky Metafont. I received two other
+replies (enclosed) that led me to download and try OzTeX,
+DirectTeX, and CMacTeX. I only have a few days of experience
+with these other packages, but thus far I like the OzTeX the best
+and it replicates the examples in Knuth's MetaFont book without
+any problems (something I couldn't do in the Blue Sky version).
+
+Sincerely,
+Tom
+
+- --
+Dr Thomas A Bryan
+E-Systems, Goleta Division
+One South Los Carneros, Goleta, California, USA 93117-3197, USA
+Email: tbryan@esd.ray.com Tel: (805) 967-5511 ext. 2974
+FAX: (805) 964-0470
+
+Home: 275 Savona Avenue, Goleta, California, 93117, USA (805) 968-1264
+
+Return-Path: <@ESDSVR.esd.ray.com:Alun.J.Carr@ucd.ie>
+Date: Fri, 12 Jul 1996 18:22:47 +0100
+From: Alun.J.Carr@ucd.ie (Dr Alun J. Carr)
+Subject: Re: METAFONT for Macintosh?
+X-Sender: ajcarr@pop3.ucd.ie
+To: tbryan@esd.ray.com
+Mime-Version: 1.0
+Content-Type: text/plain; charset="us-ascii"
+Content-Transfer-Encoding: 7BIT
+
+Tom,
+
+>Has anyone had success in running METAFONT on a Macintosh?
+
+Well, I haven't even tried the Bluesky one, which is obsolete, to say the
+least. All the `mainstream' TeX implementations for the Mac (OzTeX,
+DirectTeX Pro, and CMacTeX) come with their own METAFONT implementations,
+which work extremely well, with the DVI previewers/printer drivers
+automatically calling METAFONT to generate missing fonts. Textures seems to
+be primarily Postscript font based, rather than bitmap-based, so METAFONT
+is a bit of an anomaly in a Textures system. I know only one Textures user
+here at UCD, and even he abandoned it a while back, and started using
+DirectTeX Pro (which works like a dream in conjunction with the Alpha text
+editor and the Excalibur spelling-checker).
+
+You can find the latest releases of OzTeX, CMacTeX and DirectTeX Pro, with
+their METAFONTs at the UK or German CTAN sites (the US CTAN is pretty-much
+defunct):
+
+<ftp://ftp.tex.ac.uk/tex-archive/systems/mac/>
+<ftp://ftp.dante.de/tex-archive/systems/mac/>
+
+Hope this helps.
+
+Alun
+
+
+- --
+Dr Alun J. CARR Phone: +353-1-7061989
+Mechanical Engineering Dept. +353-1-2693244 x1989
+University College Dublin Fax: +353-1-2830534
+Belfield E-mail: <Alun.J.Carr@ucd.ie>
+Dublin 4 WWW: <http://tizit.ucd.ie/ajcarr/>
+Ireland
+
+
+
+Return-Path: <@ESDSVR.esd.ray.com:phadke@oceaneng.eng.hawaii.edu>
+Mime-Version: 1.0
+Content-Type: text/plain; charset="us-ascii"
+Date: Thu, 11 Jul 1996 16:23:54 -1000
+To: TeXhax@tex.ac.uk
+From: phadke@oceaneng.eng.hawaii.edu (Amal Phadke)
+Subject: Re: METAFONT for Macintosh?
+Cc: tbryan@esd.ray.com
+
+> From: Tom Bryan <tbryan@Thorin>
+> Date: Fri, 5 Jul 96 10:29:29 PDT
+> Subject: METAFONT for Macintosh?
+>
+> Has anyone had success in running METAFONT on a Macintosh?
+>
+> I have a Macintosh PowerBook 5300c, system 7.5.
+> I am using the public domain METAFONT version 0.66 from the
+> Bluesky ftp site, and it is not fully operational.
+> Is there a better METAFONT for Macintosh available?
+>
+> Sincerely,
+> Tom Bryan
+>
+> E-Systems, Goleta Division
+> One South Los Carneros, Goleta, CA 93117-3197, USA
+> Email: tbryan@esd.ray.com Tel: (805) 967-5511 ext. 2974
+> FAX: (805) 964-0470
+>
+
+Tom:
+
+ OzTeX 2.0.1, Andrew Trevorrow's Macintosh shareware version of TeX
+now has its own METAFONT program OzMF. It has worked flawlessly for me so
+far. OzTeX is available at all CTAN sites under
+"/tex-archive/systems/mac/oztex" directory.
+
+Cheers
+
+- ---
+Amal Phadke
+Department of Ocean Engineering,
+School of Ocean and Earth Sciences & Technology
+University of Hawaii at Manoa
+2540 Dole street , Holmes Hall 408B
+Honolulu, HI 96822, USA
+Tel: (808) 956-8198 Fax: (808) 956-3498
+
+e-mail: phadke@oceaneng.eng.hawaii.edu
+WWW Page : http://oceaneng.eng.hawaii.edu/~phadke
+
+
+
+
+------------------------------
+
+End of TeXhax Digest V96 #11
+****************************
+
+
+About TeXhax...
+
+Please send contributions to: TeXhax@tex.ac.uk
+
+Subscription and unsubscription requests:
+ send a one line mail message to TeXhax-Request@tex.ac.uk
+ containing either subscribe texhax
+ or unsubscribe texhax
+If you have problems with un/subscribing, please mail owner-texhax@nott.ac.uk
+
+To obtain the Frequently Asked Questions (FAQ) lists for TeX, send a
+message with no subject to fileserv@shsu.edu, consisting of
+SENDME FAQ
+
+For information on the TeX Users Group, please send a message to
+TUG@TUG.org, or write TeX Users Group, 1850 Union Street, #1637
+San Francisco CA 94123 (phone: 1 415 982 8449, fax: 1 415 982 8559)
+
+Backnumbers of all the digests are stored in the Comprehensive TeX
+Archive Network (CTAN) and can be retrieved on the Internet by
+anonymous ftp. The hosts comprising CTAN include, among others,
+ ftp.dante.de (129.69.1.12) -- Germany
+ ftp.shsu.edu (192.92.115.10) -- USA
+ ftp.tex.ac.uk (128.232.1.87) -- UK
+Please use your nearest server, to keep network load down.
+The file /tex-archive/CTAN.sites on each of these hosts gives a
+list of other sites which maintain full or partial mirrors of the CTAN.
+Alternatively, finger ctan_us@ftp.shsu.edu for full details.
+
+TeXhax Digest back issues are filed below /tex-archive/digests/texhax/
+Keyword-In-Context indexes are filed in /tex-archive/digests/indexes/
+
+A Hypermail version of TeXhax is also available on the World-Wide Web at URL
+http://www.tex.ac.uk/tex-archive/digests/hyper/
+
+\bye
+
diff --git a/info/digests/texhax/96/texhax.12 b/info/digests/texhax/96/texhax.12
new file mode 100644
index 0000000000..0a7ab8617b
--- /dev/null
+++ b/info/digests/texhax/96/texhax.12
@@ -0,0 +1,352 @@
+From: owner-texhax-digest@nottingham.ac.UK (Majordomo list server)
+Newsgroups: ucam.mlist.texhax
+Subject: TeXhax Digest V96 #12
+Message-ID: <E0utVVW-0006lb-00@heaton.cl.cam.ac.ukD>
+Date: 22 Aug 96 09:44:45 GMT
+Sender: owner-texhax-digest@nottingham.ac.uk
+Reply-To: TeXhax@tex.ac.uk
+Lines: 341
+
+
+TeXhax Digest Thursday, 22 August 1996 Volume 96 : Number 012
+
+(incorporating UKTeX Digest)
+
+Today's Topics:
+
+ Re: METAFONT for Macintosh?
+ Migration of CTAN-related mailing lists
+ Matrices
+ T.V. Raman in the news
+ ANNOUCEMENT: Sauter-2.4
+ single quotes using the times postscript fonts???
+ How do you get equal variable-sized delimiters?
+ Coversion of DVI file to ASCII File
+ DEK in the news
+
+----------------------------------------------------------------------
+
+From: Tom Bryan <tbryan@Thorin>
+Date: Wed, 17 Jul 96 15:50:23 PDT
+Subject: Re: METAFONT for Macintosh?
+
+Amal:
+
+Thank you for your advice on running METAFONT on the Macintosh.
+I downloaded OzTeX and it is working very well for me. I had
+been beating myself over the head with the Blue Sky version for
+weeks trying to get it to work properly. You saved me a great
+deal of time.
+
+Cheers,
+Tom
+
+- --
+Dr Thomas A Bryan
+E-Systems, Goleta Division
+One South Los Carneros, Goleta, California, USA 93117-3197, USA
+Email: tbryan@esd.ray.com Tel: (805) 967-5511 ext. 2974
+FAX: (805) 964-0470
+
+Home: 275 Savona Avenue, Goleta, California, 93117, USA (805) 968-1264
+
+------------------------------
+
+From: Rainer Schoepf <schoepf@uni-mainz.de>
+Date: Thu, 18 Jul 1996 19:01:13 +0200 (MET DST)
+Subject: Migration of CTAN-related mailing lists
+
+The CTAN-related mailing lists
+
+ CTAN@shsu.edu
+ CTAN-ANN@shsu.edu
+
+have now been migrated to a new mailing list server, located at
+Heidelberg, Germany. These lists can now be reached as
+
+ CTAN@urz.Uni-Heidelberg.de
+ CTAN-ANN@urz.Uni-Heidelberg.de
+
+Requests for subscription/deletion must be sent to
+
+ LISTSERV@urz.Uni-Heidelberg.de
+
+
+
+For the CTAN team
+
+- --
+ Rainer Schvpf
+ Zentrum f|r Datenverarbeitung A point of view can be a dangerous
+ der Universitdt Mainz luxury when substituted for insight
+ Anselm-Franz-von-Bentzel-Weg 12 and understanding.
+ D-55099 Mainz
+ Germany Herbert Marshall McLuhan:
+ <Schoepf@Uni-Mainz.DE> The Gutenberg Galaxy
+
+------------------------------
+
+From: Barry Tesman <tesman@dickinson.edu>
+Date: Fri, 26 Jul 1996 09:37:43 -0400 (EDT)
+Subject: Matrices
+
+I am trying to produce the following "labelled" matrix with latex:
+
+ B1 B2 B3
+ / \
+ a1 | 8 10 4 |
+A = a2 | 11 5 18 |
+ a3 | 27 6 18 |
+ \ /
+
+I tried to use array/tabbing/tabular but am having trouble placing
+the left and right large parentheses around the matrix but not the labels.
+Any ideas would be most helpful.
+
+Thanks,
+
+Barry Tesman < tesman@dickinson.edu >
+Dickinson College
+Carlisle, PA, USA
+
+
+
+------------------------------
+
+From: "Nelson H. F. Beebe" <beebe@math.utah.edu>
+Date: Mon, 19 Aug 1996 08:53:20 -0600 (MDT)
+Subject: T.V. Raman in the news
+
+You might care to look at this just-appeared profile of TUGboat
+author and TUG conference participant T. V. Raman:
+
+@String{j-SCI-AMER = "Scientific American"}
+
+
+@Article{Gibbs:1996:PVR,
+ author = "W. Wayt Gibbs",
+ title = "Profile: T. V. Raman: Envisioning Speech",
+ journal = j-SCI-AMER,
+ volume = "275",
+ number = "3",
+ pages = "52, 54",
+ month = nov,
+ year = "1996",
+ acknowledgement = ack-nhfb,
+ bibdate = "Mon Aug 19 08:48:09 1996",
+.
+
+Curiously, this article shows a single equation, which seems
+to violate the Scientific American practice of eschewing
+equations.
+
+========================================================================
+Nelson H. F. Beebe Tel: +1 801 581 5254
+Center for Scientific Computing FAX: +1 801 581 4148
+Department of Mathematics, 105 JWB Internet: beebe@math.utah.edu
+University of Utah URL: http://www.math.utah.edu/~beebe
+Salt Lake City, UT 84112, USA
+========================================================================
+
+------------------------------
+
+From: KNAPPEN@VKPMZD.kph.Uni-Mainz.DE
+Date: Sun, 21 Jul 1996 16:31:56 +0100
+Subject: ANNOUCEMENT: Sauter-2.4
+
+I have uploaded a new Sauter release to the CTAN archives in directory
+tex-archive/fonts/cm/sauter24/
+and it will propagate to the mirrors soon.
+
+Attached is the release file.
+
+J"org Knappen.
+
+%*%*%*%*%*%*%*%*%*%*%*%*%*%*%*%*%*%*%*%*%*%*%*%*%*%*%*%*%*%*%*%*%*%*%*%*%*%*%*%*
+
+The release 2.4 of the Sauter tools to generate true sized fonts contains
+the following news:
+
+* Support for LaTeX2e 1996/06/01
+* Fixes to cmcsc fonts
+* Fix to the capital S in cmmi fonts
+* Sauterised versions of AMS symbol fonts, bbm fonts and rsfs fonts
+ contributed by Hubert Holin
+
+If you already have version 2.3 of the Sauter fonts, please regenerate
+the cmcsc fonts for sizes => 10pt, there was a bug affecting the depth of
+the letter `q'.
+
+Please recalculate the cmmi fonts for sizes >12pt, the capital S will look
+much better afterwards.
+
+Send any questions, comments and suggestions to
+
+J"org Knappen knappen@vkpmzd.kph.uni-mainz.de
+
+------------------------------
+
+From: coleman@natasha.phil.uregina.ca (Robert Coleman)
+Date: Mon, 22 Jul 1996 16:09:17 -0600 (CST)
+Subject: single quotes using the times postscript fonts???
+
+I have installed the times.sty package that uses
+the psfonts courier, helvetica and times in LaTeX2e.
+In the times font ptmr8r, the usual positions for
+the left single quote "60 and for the right single
+quote "27 contain instead of the symbols found in
+the font cmr a symbol that looks like a grave accent
+and a symbol that looks like a vertical prime,
+respectively. How is one to get matched single quotes?
+
+There is an older font rptmr that has the usual
+single quotes in those positions. Is there
+some way to use the font ptmr8r and still get the
+usual single quotes, for example, by means of some
+trick using virtual fonts and rptmr?
+
+Thanks,
+Robert
+- --
+Robert Alan Coleman
+Department of Physics
+University of Regina
+Regina, Saskatchewan
+Canada S4S 0A2
+
+Tel: (306) 585-4260
+Fax: (306) 585-4894
+email: coleman@cas.uregina.ca
+
+********end message***********************
+
+------------------------------
+
+From: holliegp@eee.bham.ac.uk (Garry Hollier)
+Date: Wed, 21 Aug 1996 12:21:42 +0100
+Subject: How do you get equal variable-sized delimiters?
+
+In a LaTeX (I'm using LaTeX2e <1994/06/01> patch level 2) expression like
+
+\begin{eqnarray}
+ \left<delimiter>
+ %some stuff
+ \right.\\
+ \left.
+ %some more stuff
+ \right<delimiter>
+\end{eqnarray}
+
+how do you ensure that the resulting <delimiter>'s are the same size?
+
+
+------------------------------
+
+From: "ABDOL S. SOOFI, ECONOMICS, UW-PLATTEVILLE; HTTP://VMS.WWW.UWPLATT.EDU/~SOOFI" <SOOFI@uwplatt.edu>
+Date: Wed, 21 Aug 1996 11:00:59 -0500 (CDT)
+Subject: Coversion of DVI file to ASCII File
+
+Greetings to all,
+
+Is there a way to convert a filename.dvi to its LaTeX source file (ASCII
+file)? I am in a peculiar situation. I have the dvi version of a revised
+file, but its revised LaTeX source file is missing?
+
+Also, what is the meaning of ``dvi file is corrupt''? What are the
+possible sources of this problem?
+
+Thanks in advance for any insights.
+
+- --Abdol
+
+------------------------------
+
+From: "Nelson H. F. Beebe" <beebe@math.utah.edu>
+Date: Fri, 16 Aug 1996 09:54:23 -0600 (MDT)
+Subject: DEK in the news
+
+The September 1996 issue of Dr. Dobb's Journal has an editorial on p. 6
+which begins:
+
+ It was a pleasant surprise to open the mail and read that Donald
+ Knuth, the author of the three-volume {\em The Art of Computer
+ Programming}, is the recipient of the Inamori Foundation's
+ 1996 Kyoto Prize in the category of Advanced Technology.
+ Considering that a check for $460,000 accompanies his wall
+ plaque, Dr. Knuth is probably more pleased than I.
+
+ According to Kazuo Inamori, president of the Inamori Foundation
+ and chairman of Kyocera, Knuth was given Japan's highest
+ private award for his contributions to the betterment of
+ humankind. In addition to {\em The Art of Computer Programming},
+ Knuth (who was featured in an April 1996 DDJ interview) is
+ the creator of the TeX document-preparation system, Metafont
+ font-design system, and LR parser and attribute grammar.
+ TeX has been described as the most important achievement
+ in publishing since Gutenberg's moveable type.
+
+ Previous Kyoto Advanced Technology laureates include John
+ McCarthy of artificial-intelligence fame, and George Gray,
+ developer of the liquid-crystal display. Please join me
+ in hearty congratulations to Dr. Knuth.
+
+ ...
+
+ Jonathan Erickson
+ Editor-in-chief
+
+========================================================================
+Nelson H. F. Beebe Tel: +1 801 581 5254
+Center for Scientific Computing FAX: +1 801 581 4148
+Department of Mathematics, 105 JWB Internet: beebe@math.utah.edu
+University of Utah URL: http://www.math.utah.edu/~beebe
+Salt Lake City, UT 84112, USA
+========================================================================
+
+
+
+------------------------------
+
+End of TeXhax Digest V96 #12
+****************************
+
+
+About TeXhax...
+
+Please send contributions to: TeXhax@tex.ac.uk
+
+Subscription and unsubscription requests:
+ send a one line mail message to TeXhax-Request@tex.ac.uk
+ containing either subscribe texhax
+ or unsubscribe texhax
+If you have problems with un/subscribing, please mail owner-texhax@nott.ac.uk
+
+To obtain the Frequently Asked Questions (FAQ) lists for TeX, send a
+message with no subject to fileserv@shsu.edu, consisting of
+SENDME FAQ
+
+For information on the TeX Users Group, please send a message to
+TUG@TUG.org, or write TeX Users Group, 1850 Union Street, #1637
+San Francisco CA 94123 (phone: 1 415 982 8449, fax: 1 415 982 8559)
+
+Backnumbers of all the digests are stored in the Comprehensive TeX
+Archive Network (CTAN) and can be retrieved on the Internet by
+anonymous ftp. The hosts comprising CTAN include, among others,
+ ftp.dante.de (129.69.1.12) -- Germany
+ ftp.shsu.edu (192.92.115.10) -- USA
+ ftp.tex.ac.uk (128.232.1.87) -- UK
+Please use your nearest server, to keep network load down.
+The file /tex-archive/CTAN.sites on each of these hosts gives a
+list of other sites which maintain full or partial mirrors of the CTAN.
+Alternatively, finger ctan_us@ftp.shsu.edu for full details.
+
+TeXhax Digest back issues are filed below /tex-archive/digests/texhax/
+Keyword-In-Context indexes are filed in /tex-archive/digests/indexes/
+
+A Hypermail version of TeXhax is also available on the World-Wide Web at URL
+http://www.tex.ac.uk/tex-archive/digests/hyper/
+
+\bye
+
+
+
diff --git a/info/digests/texhax/96/texhax.13 b/info/digests/texhax/96/texhax.13
new file mode 100644
index 0000000000..a5a88f113c
--- /dev/null
+++ b/info/digests/texhax/96/texhax.13
@@ -0,0 +1,429 @@
+From texhax-digest-outgoing-request@nottingham.ac.uk Tue Oct 15 11:48:14 1996
+Received: from jess.ccc.nottingham.ac.uk (jess.ccc.nottingham.ac.uk [128.243.40.193]) by granby.ccc.nottingham.ac.uk (8.6.12/8.6.12) with ESMTP id LAA27207 for <cczdao@unix.ccc.nottingham.ac.uk>; Tue, 15 Oct 1996 11:48:12 +0100
+Message-Id: <199610151048.LAA27207@granby.ccc.nottingham.ac.uk>
+Received: from nottingham.ac.uk by jess.ccc.nottingham.ac.uk
+ id <14694-0@jess.ccc.nottingham.ac.uk>;
+ Tue, 15 Oct 1996 11:42:19 +0100
+From: Majordomo list server <owner-texhax-digest@nottingham.ac.uk>
+To: texhax-digest@nottingham.ac.uk
+Subject: TeXhax Digest V96 #13
+Reply-To: TeXhax@tex.ac.uk
+Errors-To: owner-texhax-digest@nottingham.ac.uk
+Precedence: bulk
+Date: Tue, 15 Oct 1996 11:42:19 +0100
+Sender: owner-texhax-digest@nottingham.ac.uk
+
+
+TeXhax Digest Tuesday, 15 October 1996 Volume 96 : Number 013
+
+(incorporating UKTeX Digest)
+
+Today's Topics:
+
+ Answers
+ Re:TeXhax Digest V96 #12
+ Re: TeXhax Digest V96 #12
+ (Fwd) Repeated announcement T&G course NTG
+ Memory question
+ ispell problems
+ Re: word to latex converter
+ Announce: xtem X11-TeX-Menu 5.23
+
+----------------------------------------------------------------------
+
+From: Jose Manuel Souto Menendez <mtpsomej@lg.ehu.es>
+Date: Fri, 4 Oct 1996 19:14:51 +0100 (WET DST)
+Subject: Answers
+
+I am writing a book with Latex and I would like to know how it is posible
+to write the answers of some of the exercises in such a way that that
+they appear at the end of the book. Are there any macros to solve
+this problem? (I am using AMSLaTeX, and with LaTeX2e).
+ Thank you very much,
+ Jose M. Souto
+
+------------------------------
+
+From: kletzing@totcon.com (Dennis Kletzing)
+Date: Fri, 23 Aug 1996 12:47:39 -0400
+Subject: Re:TeXhax Digest V96 #12
+
+Barry:
+
+I just noticed your post to TeXhax. Did you ever get a response? The
+\bordermatrix macro in the Texbook will do just what you want. If you don't
+have it I'll be glad to send it along.
+
+
+
+
+>In article <199608220854.JAA28160@paperboy.ccc.nottingham.ac.uk>, you write:
+>From: Barry Tesman <tesman@dickinson.edu>
+>Date: Fri, 26 Jul 1996 09:37:43 -0400 (EDT)
+>Subject: Matrices
+>
+>I am trying to produce the following "labelled" matrix with latex:
+>
+> B1 B2 B3
+> / \
+> a1 | 8 10 4 |
+>A = a2 | 11 5 18 |
+> a3 | 27 6 18 |
+> \ /
+>
+>I tried to use array/tabbing/tabular but am having trouble placing
+>the left and right large parentheses around the matrix but not the labels.
+>Any ideas would be most helpful.
+
+
+
+_______________________________________________________________________________
+Dennis Kletzing
+Department of Mathematics & Computer Science
+Stetson University
+DeLand, Florida
+kletzing@totcon.com
+
+
+
+------------------------------
+
+From: "K. Berry" <kb@cs.umb.edu>
+Date: Fri, 23 Aug 1996 15:56:09 -0400
+Subject: Re: TeXhax Digest V96 #12
+
+ In the times font ptmr8r, the usual positions for
+ the left single quote "60 and for the right single
+ quote "27 contain instead of the symbols found in
+ the font cmr a symbol that looks like a grave accent
+ and a symbol that looks like a vertical prime,
+ respectively. How is one to get matched single quotes?
+
+ There is an older font rptmr that has the usual
+ single quotes in those positions. Is there
+ some way to use the font ptmr8r and still get the
+ usual single quotes, for example, by means of some
+ trick using virtual fonts and rptmr?
+
+Use ptmr (close to plain-compatible) or ptmr8q (Cork), which have the
+usual quotes in the usual positions. Those are virtual fonts based on 8r.
+I wouldn't call them tricks, exactly.
+
+ptmr8r is close to Windows-compatible, for reasons explained in the
+8r.enc source file. Its primary purpose is to make all characters
+commonly present in Type 1 fonts available for typesetting (i.e., with
+virtual fonts), not to be the One True Encoding for typesetting itself.
+
+If you want to use 8r itself, then you'll have to make ` and ' active
+and have them typeset the right character. You probably don't want to do this.
+
+------------------------------
+
+From: Erik Frambach <E.H.M.Frambach@eco.rug.nl>
+Date: Thu, 29 Aug 1996 10:33:26 GMT+0200
+Subject: (Fwd) Repeated announcement T&G course NTG
+
+- ------- Forwarded Message Follows -------
+Date: Mon, 26 Aug 1996 13:03:16 +0200
+Reply-to: Netherlands Tex users-Group <NTG@NIC.SURFNET.NL>,
+ Kees van der Laan <cgl@RC.SERVICE.RUG.NL>
+From: Kees van der Laan <cgl@RC.SERVICE.RUG.NL>
+Subject: Repeated announcement T&G course NTG
+
+ ----------------------------------------------------------
+ |Announcement NTG's one-day low-budget no-nonsense course|
+ | |
+ | (La)TeX and Graphics |
+ ----------------------------------------------------------
+
+What? MetaPost, PostScript, mftoeps... (no font design)
+
+Language? English
+
+Teacher? Boguslaw Jackowksi
+
+Where? University of Utrecht
+
+When? The day before *or* after NTG's fall meeting of 24 Oct.
+ (To be announced late August)
+
+Costs? Fl 50 members of NTG or other LUGs; non-members Fl 150
+
+Subsription? Treasurer NTG, Giro 1306238, Eindhoven
+ (Do mention TeX and Graphics course)
+
+Information? Kees van der Laan, cgl@rc.service.rug.nl
+
+Literature? Jackowksi's METAFONT booklet will be in English available.
+ Have a look at
+ Hobby's CSTR 162, A user manual for MetaPost,
+ from
+ netlib@research.att.com
+ with message
+ send 162 from research/cstr
+ or copy it from NTG's 4AllTeX CD-ROM
+
+When the number of subscriptions is insufficient of
+
+ 7 September---ultimate date for registration---
+
+the course will be cancelled (and money refunded).
+
+- ---Kees---
+
+------------------------------
+
+From: Mark Freeman <AFINMF@razor.wbs.warwick.ac.uk>
+Date: Thu, 29 Aug 1996 10:00:09 GMT
+Subject: Memory question
+
+Sorry if this is a FAQ or a newbie question, but I am in need of some
+help. I have tried crawling the web pages but with little success...
+
+I am running LaTeX2.09 (dated 9/1/90) under DOS. I am towards the end
+of writing my thesis and am currently trying to compile it as one
+document (rather than individual chapters) using \include
+instructions. On first compilation it runs fine, and will continue
+to compile on each subsequent compilation UNLESS I have run BibTeX
+first. In this case, I can only get it to recompile if I instruct
+"del *.aux" prior to recompilation - obviously not very satisfactory.
+The number of different references is around 180 at present
+(although there are many more \citeasnoun instructions).
+
+The error message I am getting on recompiling after running BibTeX is
+insufficient memory. On looking at the .log file it appears to be
+"save size" that is crashing. After first compilation the memory
+counter is "362s" and then on second compilation I get "601s ... out
+of ... 600s". Other sections of memory appear to be fine.
+
+I have tried to reduce the number of \citeasnoun, but this appears to
+make no difference. If I edit my .aux file to manually remove about
+50 of the 180 citations it will recompile with no problem.
+
+Any ideas about what I should do to get it to recompile with no
+crashes after running BibTeX? I would be very grateful if you would
+email me direct with any potential solutions.
+
+Thanking you in advance for your help.
+
+Mark Freeman.
+
+------------------------------
+
+From: Clinton Arokianathan <clint@elec.gla.ac.uk>
+Date: Fri, 30 Aug 1996 15:02:23 +0100
+Subject: ispell problems
+
+I have recently downloaded ispell version 3.1.
+
+Unfortunately it is still the US dictionary despite using the Makefile in
+/languages/british. We also have the /langages/english folder.
+What lables need to be changed in local.h?
+
+
+I would be grateful if you can help
+
+Clint
+
+- ---------------------------------------------------------------------
+ Clinton R Arokianathan
+ Nanoelectronics Research Centre
+ Department of Electronics and Electrical Engineering
+ University of Glasgow
+ Glasgow G12 8QQ
+ Scotland
+
+ phone: +44-(0)141-339-8855 Ext 6024 (office)
+ 8349 (laboratory)
+ fax: +44-(0)141-330-4907
+
+ e-mail: clint@elec.gla.ac.uk
+- ---------------------------------------------------------------------
+
+
+------------------------------
+
+From: cross@seraph1.sewanee.edu (Clay C. Ross)
+Date: Fri, 11 Oct 1996 09:30:23 +0100
+Subject: Re: word to latex converter
+
+At 12:24 PM 10/10/96, Kevin R. Vixie wrote:
+
+ * I am interested in your web site dealing with word to tex conv. but it
+ * seems to be offline right now.
+ *
+ * Kevin R. Vixie
+
+This was my reply:
+
+I do not now have, nor ever have had such a site. I HAVE answered *many*
+questions since spring about rtf2latex and latex2rtf, two programs that
+others wrote. To find the flavor you need, look them up using a web browser.
+
+Also, rtf2tex and tex2rtf probably exist. Find them by searching.
+
+- -- I add this for this transmission: --
+
+There seems to be a great deal of interest in converters to- and from-
+Word (on many platforms). All I know about them is that the conversions
+work as follows:
+
+Word Doc. -> Word(Save As) -> RTF(text)Doc.-> rtf2latex -> LaTeX Doc. -> LaTeX
+ or
+Latex Document -> latex2rtf -> Word(read and convert RTF) -> Word Document
+
+Some TeX/LaTeX expert could provide a valuable service by setting up a
+site dedicated to such converters. I am almost totally ignorant of the
+issues; for me to become involved would be foolhardy. Please, would an
+informed person step up to provide this service (a Converter Site)?
+
+CCR
+
++++++++++++++++++++++++++++++++
+ Clay C. Ross (615) 598-1301
+ http://cross.sewanee.edu/
+ Mathematics & Computer Science
+ The University of the South
+ Sewanee, Tennessee 37383-1000
+
+
+
+------------------------------
+
+From: l44@iwd.uni-bremen.de (Roland Weibezahn)
+Date: Tue, 15 Oct 1996 09:23:56 +0200
+Subject: Announce: xtem X11-TeX-Menu 5.23
+
+Dear xtem users,
+
+We have finished the new version (xtem_TeXMenu.5.23) of
+
+ "xtem", an X11-TeX-menu
+
+which runs with the new Tcl/Tk/TclX (Tcl7.5/ and Tcl7.6-beta1)
+as well as with the old version (Tcl7.4).
+
+In addition to adapting xtem to the new Tcl/Tk, we have done
+errror corrections and added new features, such as:
+
+ - installation (and maintenance of the logfiles) is much easier with
+ the (new) installation procedure,
+
+ - (optional) logfile analyze after TeX run, you can then click at each
+ error message (incl. overfull/underfull boxes) in order to edit
+ the texfile at the corresponding position,
+
+ - "quick&dirty" button for quick TeXing and previewing of a small text
+ fragment (including preamble),
+
+ - the edit file can be selected from other directories than the main file,
+
+ - (optional) vertical scrollbars for the text widgets,
+
+ - revision of all bindings (conforming behaviour: actions are done
+ at button release),
+
+ - LaTeX syntax got it's own button now.
+
+Most setting files from older versions of xtem may be kept unchanged:
+
+ - mkcommand.vst must be updated
+ - texfmt.vst should be updated to enable "quick&dirty"
+ - logform.vst one line concerning logfile analyze can/should be added
+
+
+
+You will find all files on our file server:
+
+ http://ftp.iwd.uni-bremen.de/xtem/xtem_texmenu.html
+or
+ ftp://ftp.iwd.uni-bremen.de/pub/tex/xtem.v5/xtem_texmenu.5.23.tar.gz
+ ftp://ftp.iwd.uni-bremen.de/pub/tex/xtem.v5/xtem_texmenu_5eng.ps.gz
+ ftp://ftp.iwd.uni-bremen.de/pub/tex/xtem.v5/xtem_texmenu_5ger.ps.gz
+
+Here you may also find the sources: Tcl7.5, Tk4.1, TclX7.5.2
+
+ ftp://ftp.iwd.uni-bremen.de/pub/tcl/tcl7.4p2.tar.gz
+ ftp://ftp.iwd.uni-bremen.de/pub/tcl/tk4.0p2.tar.g
+ ftp://ftp.iwd.uni-bremen.de/pub/tcl/tclX7.4a-p1.tar.gz
+
+We have uploaded the new xtem version to the
+neosoft server into "/pub/tcl/incoming/",
+and it's already mirrored to the CTAN servers,
+so you also may get all the material from
+ ftp://ftp.neosoft.com/pub/tcl/NEW/xtem_*
+and ftp://ftp.dante.de/tex-archive/support/xtem_texmenu/xtem.v5/*
+and the other CTAN servers.
+
+
+For those who don't know xtem_TeXMenu up to now, we give a short overview:
+
+xtem provides for a simple and comfortable graphical user interface to control
+the following facilities:
+
+ - file and directory selection, directory creation,
+ - editor (vi, emacs, ...) including additional windows for the
+ LaTeX-syntax (using hypertext) and examples,
+ - TeX, LaTeX, ...,
+ - previewer (ghostview, xdvi, TkDvi, ...),
+ - printing (including comfortable printer selection, ...),
+ - syntax and spelling check,
+ - makeindex,
+ - bibtex,
+ - additional programs as required,
+ .
+ .
+ .
+Online help is available for all the buttons and windows by simple mouse click.
+xtem is written for Unix platforms and has been tested on many systems.
+
+
+
+Thanks,
+ Roland Weibezahn
+- ---
+Dr. Roland Weibezahn
+weibezahn@iwd.uni-bremen.de phone: +49-421-218-3532
+University Bremen, IWD, postbox: 330440, 28334 Bremen, Germany
+http://ftp.iwd.uni-bremen.de/xtem/xtem_texmenu.html (the xtem_TeXMenu project)
+
+------------------------------
+
+End of TeXhax Digest V96 #13
+****************************
+
+
+About TeXhax...
+
+Please send contributions to: TeXhax@tex.ac.uk
+
+Subscription and unsubscription requests:
+ send a one line mail message to TeXhax-Request@tex.ac.uk
+ containing only the line
+ subscribe texhax
+ or
+ unsubscribe texhax
+If you have problems with un/subscribing,
+please mail texhax-owner@nottingham.ac.uk
+
+For information on the TeX Users Group, please send a message to
+TUG@TUG.org, or write TeX Users Group, 1850 Union Street, #1637
+San Francisco CA 94123 (phone: 1 415 982 8449, fax: 1 415 982 8559)
+
+Backnumbers of all the digests are stored in the Comprehensive TeX
+Archive Network (CTAN) and can be retrieved on the Internet by
+anonymous ftp. The hosts comprising CTAN include, among others,
+ ftp.dante.de (129.69.1.12) -- Germany
+ ftp.tex.ac.uk (128.232.1.87) -- UK
+Please use your nearest server, to keep network load down.
+The file /tex-archive/CTAN.sites on each of these hosts gives a
+list of other sites which maintain full or partial mirrors of the CTAN.
+Alternatively, finger ctan_us@ftp.shsu.edu for full details.
+
+TeXhax Digest back issues are filed below /tex-archive/digests/texhax/
+Keyword-In-Context indexes are filed in /tex-archive/digests/indexes/
+
+A Hypermail version of TeXhax is also available on the World-Wide Web at URL
+http://www.tex.ac.uk/tex-archive/digests/hyper/
+
+\bye
diff --git a/info/digests/texhax/96/texhax.14 b/info/digests/texhax/96/texhax.14
new file mode 100644
index 0000000000..45b951e71b
--- /dev/null
+++ b/info/digests/texhax/96/texhax.14
@@ -0,0 +1,344 @@
+(Message texhax/v96:14)
+From: owner-texhax-digest
+To: texhax-digest
+Subject: TeXhax Digest V96 #14
+Reply-To: TeXhax@tex.ac.uk
+Errors-To: owner-texhax-digest
+Precedence: bulk
+
+
+TeXhax Digest Friday, 1 November 1996 Volume 96 : Number 014
+
+(incorporating UKTeX Digest)
+
+Today's Topics:
+
+ unsuscribe
+ Phone: +44 171 380 7293
+ Converting LaTex2e to HTML
+ Re: TeXhax Digest V96 #13
+ Downloading TeX & LaTeX
+ Plain TeX query about \csname ... \endcsname
+ Memory question
+ 'none'
+
+----------------------------------------------------------------------
+
+From: Dave Elliman <dge@Cs.Nott.AC.UK>
+Date: Tue, 15 Oct 96 12:07:16 +0100
+Subject: unsuscribe
+
+
+------------------------------
+
+From: Russel Winder <R.Winder@cs.ucl.ac.uk>
+Date: Tue, 15 Oct 1996 14:11:08 +0100
+Subject: Phone: +44 171 380 7293
+
+Jose,
+
+> From: Jose Manuel Souto Menendez <mtpsomej@lg.ehu.es>
+> Date: Fri, 4 Oct 1996 19:14:51 +0100 (WET DST)
+> Subject: Answers
+>
+> I am writing a book with Latex and I would like to know how it is posible
+> to write the answers of some of the exercises in such a way that that
+> they appear at the end of the book. Are there any macros to solve
+> this problem? (I am using AMSLaTeX, and with LaTeX2e).
+> Thank you very much,
+> Jose M. Souto
+
+I do not know of any macros for doing this, I have to admit I didn't look that
+hard. I wrote my own macros for doing this, initially as TeX macros then as
+LaTeX(209|2e) environment(s).
+
+My basic algorithm was to write the exercise and answers out to a file and
+then read them in at the appropriate place in the text. This is not entirely
+trivial as the writing has to be of the original code; the verbatim stuff from
+the standard TeX/LaTeX(209|2e) came in very handy at this point.
+
+I suppose I should have turned these macros into a separate package and
+submitted it, if there is enough interest I can do this. Otherwise I could
+let you have a copy of my macros.
+
+Russel.
+=======================================================================
+
+Dr Russel Winder
+
+ Reader in Software Engineering
+ Editor-in-Chief, Object Oriented Systems
+
+Information Systems Research Group
+Department of Computer Science Phone: +44 (0)171 380 7293
+University College London Fax: +44 (0)171 387 1397
+Gower Street EMail: R.Winder@cs.ucl.ac.uk
+London WC1E 6BT
+UK URL: http://www.cs.ucl.ac.uk/staff/R.Winder/
+
+=======================================================================
+
+
+
+------------------------------
+
+From: reynoldd@ccmail.dcu.ie
+Date: Wed, 16 Oct 1996 08:53:11 +0000 (GMT)
+Subject: Converting LaTex2e to HTML
+
+ Dear TeXhax Digest,
+
+ Presumably there's alot of work being done on how to convert LaTex2e
+ to HTML. I want to put lecture notes with equations onto our intranet.
+ Can you point me to where I might find relevant advice.
+
+ Latex2html seems a good answer for unix users. However I use DOS, WIN
+ 3.11 and OS2 Warp. I did find correspondence on a newsgroup about
+ compiling a DOS version of LaTex2HTML, but there was much discussion
+ about perl and fork, about which I am ignorant.
+
+ Any advice would be gratefully received.
+
+ Regards,
+
+ David Reynolds
+
+------------------------------
+
+From: Andreas Schlechte <inas@mexico.mib.harz.de>
+Date: Thu, 17 Oct 1996 18:48:23 +0100 (MET)
+Subject: Re: TeXhax Digest V96 #13
+
+Hi,
+
+In reply to
+
+> From: Jose Manuel Souto Menendez <mtpsomej@lg.ehu.es>
+> Date: Fri, 4 Oct 1996 19:14:51 +0100 (WET DST)
+> Subject: Answers
+
+> I am writing a book with Latex and I would like to know how it is posible
+> to write the answers of some of the exercises in such a way that that
+> they appear at the end of the book. Are there any macros to solve
+> this problem? (I am using AMSLaTeX, and with LaTeX2e).
+> Thank you very much,
+> Jose M. Souto
+
+Well, you can either use the macros, Knuth is using in his TeXbook. But it's
+(a bit) difficult to implement these. I use some other macros, based on the
+package "moreverb.sty", which must be included.
+
+ ---- solution.sty ----
+ %
+ % open new file solution.tex for writing
+ %
+ \newwrite\solutionout
+ \immediate\openout\solutionout=solution.tex
+ %
+ % define environment solutionwrite
+ %
+ \def\solutionwrite{%
+ \@bsphack
+ \let\do\@makeother\dospecials
+ \catcode`\^^M\active \catcode`\^^I=12
+ \def\verbatim@processline{\immediate\write\solutionout{\the\verbatim@line}}%
+ \verbatim@start}
+ \def\endsolutionwrite{\@esphack}
+ %
+ % Use this command to close file
+ %
+ \def\closesolutionfile{\immediate\closeout\solutionout}
+ ---- end ----
+
+With these package you will be able to write any string into the file. Here
+is an example:
+
+ \documentclass{book}
+ \usepackage{moreverb,solution}
+ \begin{document}
+ \begin{solutionwrite}
+ You may use any character in this environment. its although possible to
+ write TeX-command into the file:
+ \begin{quote}
+ Like this quoted block.
+ \end{quote}
+ \end{solutionwrite}
+ %...
+ \closesolutionfile
+ \input solution.tex
+ \end{document}
+
+The only restriction is, that you might not write the string
+\end{solutionwrite} into the file. In my opinion there is rather
+no need for this.
+
+Hope, this helps a little bit at least,
+
+ Andi
+
+- --
+>>>>>>>>>>>>>>>>>>>>>>>>>>>>>>>>>>>>><<<<<<<<<<<<<<<<<<<<<<<<<<<<<<<<<<<<<<
+> Andreas Schlechte Tel.: +49-5323-1289 Q <
+> Am Klepperberg 2 +49-5323-9499-7 <
+> 38678 Clausthal-Zellerfeld Fax.: +49-5323-9499-3 <
+> Andreas.Schlechte@tu-clausthal.de http://www.tu-clausthal.de/~inas <
+>>>>>>>>>>>>>>>>>>>>>>>>>>>>>>>>>>>>><<<<<<<<<<<<<<<<<<<<<<<<<<<<<<<<<<<<<<
+
+------------------------------
+
+From: "P. WISUTHSERIWONG" <fe32@dial.pipex.com>
+Date: Fri, 18 Oct 1996 15:07:26 +0000
+Subject: Downloading TeX & LaTeX
+
+Hi there,
+
+I'm wondering if you could help. I'm interested in using LaTeX but have
+no idea where & how I can get the programmes. Could you tell me the
+actual file names I need to download from the ctan site please? Oh, do I
+have to have TeX in order to run LaTeX?
+
+Thanks very much for your time.
+
+P. Wisuthseriwong
+
+
+------------------------------
+
+From: cgm@ssci.liv.ac.uk (Colin Mason)
+Date: Tue, 22 Oct 1996 15:08:54 GMT
+Subject: Plain TeX query about \csname ... \endcsname
+
+If you do:
+
+ \tracingrestores=1
+ {\expandafter\xdef\csname 777\endcsname{555}}
+ \bye
+
+you get the message:
+
+ {retaining \777=macro:->555}
+
+in the log file. Page 301 of the TeXbook says that nothing needs to go on the
+save stack at the time of a global assignment so I was initially surprised to
+get the message. However, page 213 says that \csname ... \endcsname defines the
+control sequence to be like \relax if it is not currently defined. So, it would
+appear that \csname ... \endcsname first locally defines \777 to be like \relax
+and the \xdef then globally redefines it to be 555. If you then do:
+
+ \tracingrestores=1
+ {\globaldefs=1 \expandafter\xdef\csname 777\endcsname{555}}
+ \bye
+
+it makes no difference. Why? Shouldn't the \csname ... \endcsname globally
+define \777 to be like \relax? Or is this some strange exception to the rules?
+
+Thanks in advance for any help.
+
+Colin Mason.
+
+------------------------------
+
+From: Mark Freeman <AFINMF@razor.wbs.warwick.ac.uk>
+Date: Mon, 28 Oct 1996 14:29:49 GMT
+Subject: Memory question
+
+On 29/8/96 I posted a question to TeXhax on a memory problem that I
+was encountering running LaTeX2.09 under Dos on a document with many
+references. The following reply was sent to me by Jonathan Fine:
+
+> So far as I can tell, the problem is with the lines of code
+> > \def\bibcite#1#2{\global\@namedef{b@#1}{#2}}
+> > \def\@namedef#1{\expandafter\def\csname #1\endcsname}
+> in the main source file for latex, version 2.09. (Comments on LaTeX
+> 2e appear later.)
+>
+> Please place the definition
+> > \def\bibcite#1#2{\begingroup\expandafter\endgroup\expandafter
+> > \gdef\csname b@#1\endcsname{#2}}
+> somewhere in the preamble to your document. If I am correct, then
+> the problem will go away.
+>
+> Explanation. When \csname ... \endcsname produces an undefined
+> control sequence, it is as a local assignment given the value \relax.
+> This has the side effect of occupying space in the save stack. The
+> replacement definition does not remove this side effect, but makes it
+> harmless. The \expandafter's function so that the local definition
+> is valid only within the
+> > \begingroup\expandafter\endgroup
+> group of \bibcite. At the end of this group, the previous value of
+> \undefined is restored for \b@#1, and this frees up the space in the
+> save stack. Use \tracingall to get the gory details.
+>
+> For another explanation, see my article in Baskerville (4)1,
+> published 1994. In my article I wrote "This [side effect] can cause
+> problems in processing LaTeX documents which have a lot of
+> cross-references." So far as I can tell, this problem remains with
+> LaTeX 2e.
+
+As I am running \harvardcite as opposed to \bibcite I made the
+following minor adjustment to Jonathan's proposed solution:
+
+ \def\harvardcite#1#2#3#4{\begingroup\expandafter\endgroup
+ \expandafter\gdef\csname b@#1\endcsname{#2}
+ \begingroup\expandafter\endgroup
+ \expandafter\gdef\csname bhf@#1\endcsname{#2}
+ \begingroup\expandafter\endgroup
+ \expandafter\gdef\csname bha@#1\endcsname{#3}
+ \begingroup\expandafter\endgroup
+ \expandafter\gdef\csname bhy@#1\endcsname{#4}}
+
+This appears to have fixed my problem. I hope that others might
+find this correspondance useful.
+
+Regards,
+
+Mark Freeman.
+
+------------------------------
+
+From: "Robert Greer c/o CUNY 212-346-8450 (fax 346-8453)" <GREER%BMACADM.bitnet@CUNYVM.CUNY.EDU>
+Date: Fri, 01 Nov 96 10:43:13 EST
+Subject: 'none'
+
+Acknowledge-To: <GREER@BMACADM>
+
+------------------------------
+
+End of TeXhax Digest V96 #14
+****************************
+
+
+About TeXhax...
+
+Please send contributions to: TeXhax@tex.ac.uk
+
+Subscription and unsubscription requests:
+ send a one line mail message to TeXhax-Request@tex.ac.uk
+ containing only the line
+ subscribe texhax
+ or
+ unsubscribe texhax
+If you have problems with un/subscribing,
+please mail texhax-owner@nottingham.ac.uk
+
+For information on the TeX Users Group, please send a message to
+TUG@TUG.org, or write TeX Users Group, 1850 Union Street, #1637
+San Francisco CA 94123 (phone: 1 415 982 8449, fax: 1 415 982 8559)
+
+Backnumbers of all the digests are stored in the Comprehensive TeX
+Archive Network (CTAN) and can be retrieved on the Internet by
+anonymous ftp. The hosts comprising CTAN include, among others,
+ ftp.dante.de (129.69.1.12) -- Germany
+ ftp.tex.ac.uk (128.232.1.87) -- UK
+Please use your nearest server, to keep network load down.
+The file /tex-archive/CTAN.sites on each of these hosts gives a
+list of other sites which maintain full or partial mirrors of the CTAN.
+Alternatively, finger ctan_us@ftp.shsu.edu for full details.
+
+TeXhax Digest back issues are filed below /tex-archive/digests/texhax/
+Keyword-In-Context indexes are filed in /tex-archive/digests/indexes/
+
+A Hypermail version of TeXhax is also available on the World-Wide Web at URL
+http://www.tex.ac.uk/tex-archive/digests/hyper/
+
+\bye \ No newline at end of file
diff --git a/info/digests/texhax/96/texhax.15 b/info/digests/texhax/96/texhax.15
new file mode 100644
index 0000000000..4798e59c21
--- /dev/null
+++ b/info/digests/texhax/96/texhax.15
@@ -0,0 +1,233 @@
+(Message texhax/v96:15)
+Resent: Thu, 19 Dec 1996 18:18:37 +0000
+Resent: texhax-outgoing@nottingham.ac.uk
+Resent: Thu, 19 Dec 1996 18:16:56 +0000
+Resent: texhax@nottingham.ac.uk
+From texhax-digest-owner@nottingham.ac.uk Thu Dec 19 18:12:44 1996
+Received: from nottingham.ac.uk (jess.ccc.nottingham.ac.uk [128.243.40.193]) by granby.ccc.nottingham.ac.uk (8.6.12/8.6.12) with SMTP id SAA20933 for <cczdao@unix.ccc.nottingham.ac.uk>; Thu, 19 Dec 1996 18:12:43 GMT
+Received: from majordom by nottingham.ac.uk with local (Exim 1.58 #10)
+ id 0vamxI-0004Im-00; Thu, 19 Dec 1996 18:12:28 +0000
+From: owner-texhax-digest@nottingham.ac.uk
+To: texhax-digest@nottingham.ac.uk
+Subject: TeXhax Digest V96 #15
+Reply-To: TeXhax@tex.ac.uk
+Errors-To: owner-texhax-digest
+Precedence: bulk
+Message-Id: <E0vamxI-0004Im-00@nottingham.ac.uk>
+Date: Thu, 19 Dec 1996 18:12:28 +0000
+
+
+TeXhax Digest Thursday, 19 December 1996 Volume 96 : Number 015
+
+(incorporating UKTeX Digest)
+
+Today's Topics:
+
+ New version of AMS-LaTeX released
+ Latex
+ e-TeX is released
+ Plain TeX query about \font
+ correct way to write Makefile?
+
+----------------------------------------------------------------------
+
+From: bbeeton <BNB@MATH.AMS.ORG>
+Date: Fri, 08 Nov 1996 08:24:45 -0500 (EST)
+Subject: New version of AMS-LaTeX released
+
+An interim release of the AMS-LaTeX version 1.2 macro collection has
+been posted.
+
+This release fixes a number of bugs, and documents others that have
+not yet been fixed, but are known and are on the list for attention
+when time becomes available. The documentation has been extensively
+revised and users are requested to pay careful attention to the
+READ.ME file. In particular, AMS-LaTeX 1.2 now requires a version
+of LaTeX2e no earlier than December 1994.
+
+The canonical home site for this collection is e-math.ams.org for ftp,
+or at http://www.ams.org/tex for those who prefer a Web connection.
+The collection has also been mirrored onto CTAN (locations shown are
+relative to the root of the tex archive at the particular site):
+
+ location on e-math: location on CTAN:
+
+ /pub/tex/amslatex under macros/latex/packages/amslatex
+
+The other AMS collections are available from the same AMS servers
+and have now been installed on CTAN in these new locations:
+
+ location on e-math: location on CTAN:
+
+ /pub/tex/amsfonts under fonts/amsfonts
+ /pub/tex/amsltx11 under macros/latex209/contrib/amslatex
+ /pub/tex/amstex under macros/amstex
+
+At CTAN, the changes have been made at ftp.tex.ac.uk and ftp.dante.de;
+they will propagate to the mirrors in due course.
+
+Only the AMS-LaTeX collection has been updated; some minor changes
+are under construction in the AMSFonts and AMS-TeX collections, and
+a separate announcement will be made when these are posted.
+
+
+------------------------------
+
+From: zadokh@rafael.co.il (Zadok Hougui)
+Date: Mon, 11 Nov 1996 22:23:22 +0200
+Subject: Latex
+
+Hi
+Do you know where I could download a copy of Latex for use on an IBM PC
+compatible? (for a windows 3.1 or windows 95 environment)
+Thanks
+Zadok
+==========================================================================
+Sadok Hougui
+32 Lehi St.
+Kiryat Bialik, 27000
+Israel
+
+Home phone: (972)-4-8740167
+e-mail: zadokh@rafael.co.il
+
+
+------------------------------
+
+From: Philip Taylor <CHAA006@vms.rhbnc.ac.uk>
+Date: Wed, 13 Nov 1996 17:55:17 GMT
+Subject: e-TeX is released
+
+Dear Colleagues --
+
+I am delighted to be able to tell you that after a protracted period of
+design and development, e-TeX is now formally released. In summary, e-TeX
+is an evolutionary (rather that revolutionary) development of TeX, and is
+100%-compatible, including compatibility at the level of the TRIP test.
+However, it is also capable of operating in "extended" mode, in which case
+it adds approximately 30 new primitives to the TeX language whilst remaining
+completely compatible with TeX if none of the new primitives are used, and in
+"enhanced" mode, in which case it adds functionality (for the first release,
+TeX--XeT) at the expense of compatibility.
+
+Reference implementations are available for VMS (both VAX & AXP) and MS/DOS,
+the former implementation being primarily the work of Christian Spieler whilst
+the latter is the work of Peter Breitenlohner. As this message is being
+sent to TeX-Implementors as well as to the NTS and TeX lists, we hope that new
+implementations will become available in the near future.
+
+For further information on e-TeX, and for access to the sources and reference
+implementations, please see:
+
+ http://www.rhbnc.ac.uk/e-TeX/
+
+Philip Taylor,
+for and on behalf of the e-TeX team / NTS group.
+
+------------------------------
+
+From: cgm@ssci.liv.ac.uk (Colin Mason)
+Date: Wed, 27 Nov 1996 14:45:13 GMT
+Subject: Plain TeX query about \font
+
+If you do:
+
+ \font\titlefont=cmssdc10
+ \ifx\titlefont\nullfont
+ \wlog{Using \string\nullfont}%
+ \fi
+ \bye
+
+You get the following message in the log file:
+
+ Using \nullfont
+
+But if there is a blank line between the \font... and the \ifx... this does not
+happen. I would not expect to get the message anyway so why does it happen and
+why does inserting a blank line cure it?
+
+Thanks in advance for any help.
+
+Colin Mason.
+
+------------------------------
+
+From: Steve Kelem <steve.kelem@xilinx.com>
+Date: Fri, 06 Dec 1996 09:30:11 -0800
+Subject: correct way to write Makefile?
+
+What's the correct way to create a Makefile for LaTeX?
+I'm not sure how to capture the business about having to run:
+ latex (Extract citations.)
+ bibtex (Create the bibliography.)
+ latex (Incorporate bibliography.)
+ latex (Resolve references.)
+
+Thanks,
+/7\'7 Steve Kelem (408)879-5347 Steve.Kelem@xilinx.com
+\\ ` Xilinx FAX: (408)377-3259
+// 2100 Logic Drive
+\\/.\ San Jose, California 95124
+
+So far, I have the following, but it doesn't capture the above requirements.
+
+.SUFFIXES : $(LATEX_SUFFIXES) $(SUFFIXES)
+
+LATEX_SUFFIXES = .aux .bbl .blg .dvi .ilg .idx .ind .lof .log .lot .toc
+
+all : pop.ps
+
+%.ps : %.dvi
+ dvips OPTIONS $*.dvi
+
+
+%.ind : %.idx
+ makeindex $*
+
+%.bbl : %.aux
+ bibtex $*
+
+%.dvi %.aux : %.tex
+ latex $*.tex
+
+------------------------------
+
+End of TeXhax Digest V96 #15
+****************************
+
+
+About TeXhax...
+
+Please send contributions to: TeXhax@tex.ac.uk
+
+Subscription and unsubscription requests:
+ send a one line mail message to TeXhax-Request@tex.ac.uk
+ containing only the line
+ subscribe texhax
+ or
+ unsubscribe texhax
+If you have problems with un/subscribing,
+please mail texhax-owner@nottingham.ac.uk
+
+For information on the TeX Users Group, please send a message to
+TUG@TUG.org, or write TeX Users Group, 1850 Union Street, #1637
+San Francisco CA 94123 (phone: 1 415 982 8449, fax: 1 415 982 8559)
+
+Backnumbers of all the digests are stored in the Comprehensive TeX
+Archive Network (CTAN) and can be retrieved on the Internet by
+anonymous ftp. The hosts comprising CTAN include, among others,
+ ftp.dante.de (129.69.1.12) -- Germany
+ ftp.tex.ac.uk (128.232.1.87) -- UK
+Please use your nearest server, to keep network load down.
+The file /tex-archive/CTAN.sites on each of these hosts gives a
+list of other sites which maintain full or partial mirrors of the CTAN.
+Alternatively, finger ctan_us@ftp.shsu.edu for full details.
+
+TeXhax Digest back issues are filed below /tex-archive/digests/texhax/
+Keyword-In-Context indexes are filed in /tex-archive/digests/indexes/
+
+A Hypermail version of TeXhax is also available on the World-Wide Web at URL
+http://www.tex.ac.uk/tex-archive/digests/hyper/
+
+\bye
+